Anda di halaman 1dari 124

Capa

1
1103362 E-book gerado especialmente para MARCUS ICARO CRUZ DA SILVA
Inglês ........................................................................................................................................... 01

Espanhol ...................................................................................................................................... 52

Candidatos ao Concurso Público,

O Instituto Maximize Educação disponibiliza o e-mail professores@maxieduca.com.br para dúvidas


relacionadas ao conteúdo desta apostila como forma de auxiliá-los nos estudos para um bom
desempenho na prova.

As dúvidas serão encaminhadas para os professores responsáveis pela matéria, portanto, ao entrar
em contato, informe:

- Apostila (concurso e cargo);

- Disciplina (matéria);

- Número da página onde se encontra a dúvida; e

- Qual a dúvida.

Caso existam dúvidas em disciplinas diferentes, por favor, encaminhá-las em e-mails separados. O
professor terá até cinco dias úteis para respondê-la.

Bons estudos!

2
1103362 E-book gerado especialmente para MARCUS ICARO CRUZ DA SILVA
Serão apresentados textos autênticos selecionados de livros, jornais ou revistas, para avaliar o
domínio de vocabulário, compreensão das ideias expostas e a correspondência de sentido com a
Língua Portuguesa. O desempenho adequado na compreensão de textos exige o conhecimento básico
dos seguintes tópicos gramaticais:

Inglês
1. Estrutura dos sintagmas nominal, adjetival e adverbial.
2. O verbo e o sintagma verbal: tempo, aspecto e modo; voz; modais.
3. A estrutura de frases simples e complexa:
coordenação e subordinação; orações reduzidas.
4. Conectivos.

Serão, ainda, apresentadas oportunidades de interação comunicativa.

TÉCNICA DE LEITURA DE TEXTO DE LÍNGUA INGLESA

No Brasil, de um modo geral, o inglês instrumental é uma das abordagens do ensino do Inglês que
centraliza a língua técnica e científica focalizando o emprego de estratégias específicas, em geral,
voltadas à leitura. Seu foco é desenvolver a capacidade de compreensão de textos de diversas áreas
do conhecimento. O estudo da gramática restringe-se a um mínimo necessário normalmente associado
a um texto atual ou similar que foi veiculado em periódicos. O conhecimento de uma boa quantidade de
palavras também faz parte das técnicas que serão relacionadas abaixo.

Background knowledge (conhecimento prévio): para que um leitor consiga identificar e entender
certas informações em qualquer tipo de texto, torna-se extremamente importante que ele possua algum
conhecimento prévio sobre seu assunto. Podemos comparar esta situação com a de um estudante
tentando fazer uma prova de redação. Se ele nunca tiver lido, discutido, estudado ou ouvido falar do
tema daquela redação, como poderá dissertar? Suas ideias podem até ir para o papel, mas correrá um
grande risco de não ter o vocabulário necessário, consistência, profundidade, argumentos,
conhecimento de causa, exemplos a citar, etc. sua redação será pobre. Da mesma maneira, se o leitor
de um texto técnico em língua inglesa não tiver conhecimento de mundo, vivência, experiências
variadas de vida, conhecimento prévio sobre o assunto, seu nível de compreensão será mais
superficial. Por isso, o ponto de partida para uma leitura eficiente está sempre em você. Mas também
não adianta buscar apenas informação de coisas que te atraem, coisas que você gosta de saber. É
preciso ampliar sua visão de mundo. Se você for mulher, busque saber algo sobre futebol também,
sobre carros, sobre coisas do mundo masculino. Se você for homem, busque também conhecer
assuntos do mundo feminino como cosméticos e vestuário. Busquem ambos interessar-se por assuntos
relacionados a crianças, idosos, povos diferentes do seu, países variados, regiões do mundo sobre as
quais que você normalmente não sabe nada. Leia jornais, revistas, sites da internet, pesquise coisas
curiosas, assista a programas de TV jornalísticos, de variedades, de humor, de esportes, de ciência, de
religião, de saúde, de entretenimento, converse com pessoas de opiniões, idades e classes sociais
diferentes da sua, dê valor a todos os assuntos porque você nunca sabe qual tema será abordado num
texto de uma prova. Esteja preparado para todos eles. Desta forma podemos agilizar sua compreensão
acerca de um texto. Desta forma você terá mais prazer ao ler, pois compreenderá os mais variados
textos. Desta forma você verá que é capaz de adquirir conhecimento em uma língua estrangeira. Desta
forma poderemos minimizar seus problemas e aumentar suas chances de obter o sucesso.

Skimming (ler ou examinar superficialmente; desnatar; retirar aquilo de maior peso ou importância):
é uma técnica que permite rapidez e eficiência na busca de algum direcionamento inicial acerca do
texto. Realizar o skimming significa ler rapidamente o texto para saber o assunto principal trabalhado
pelo autor. Esta atividade de leitura nos proporciona um nível de compreensão geral, visando nos dar

1
1103362 E-book gerado especialmente para MARCUS ICARO CRUZ DA SILVA
uma visão global, aberta e ampla do texto. Ao realizarmos o skimming, não podemos nos deter em
detalhes como palavras novas nem palavras das quais nos esquecemos. Estamos em busca do
assunto principal e do sentido geral do texto.

Prediction: Com esta estratégia o leitor lança mão do seu próprio conhecimento, através das
experiências de vida que possui, e da informação linguística e contextual. Após realizar o skimming, o
leitor precisa concentrar-se para tentar ativar as informações que já possui sobre o tema e prever que
tipos de palavras, frases ou argumentos podem estar presentes naquele texto. É um momento de
reflexão. É a hora de buscar na memória tudo o que foi lido, estudado, discutido, e visto na mídia a
respeito daquele tema. Além do mais, esta é uma estratégia de leitura que também permite ao leitor
prever o que vem a seguir em um texto. Trata-se do desenvolvimento sequenciado do pensamento. Isso
só é possível porque quem escreve, o faz de maneira organizada, porque as pessoas pensam de
maneira semelhante e porque alguns tipos de textos possuem estruturas previsíveis levando nós
leitores a atingir certas formas de compreensão. Quanto mais experiente for o leitor, maior será sua
capacidade de prever. Nesta etapa, passamos a associar o assunto do texto com as dicas tipográficas
usadas pelo autor para transmitir significados.

Grifo de palavras cognatas, das palavras já conhecidas pelo leitor e das repetidas: Muito comuns
entre as línguas inglesa e portuguesa, os cognatos são termos bastante parecidos tanto na escrita
como no significado em ambas as línguas. Grifar todas estas palavras em um texto é um recurso
psicológico e técnico que visa mostrar e provar visualmente para o leitor que ele tem conhecimento de
muitas das palavras daquele texto e de que, assim, ele é capaz de fazer uso dessas informações para
responder às questões propostas. Trata-se de um recurso que usamos para dar mais relevância e
importância às palavras que já sabemos em um texto, pois é nelas que nos apoiaremos para resolver
exercícios e para entender os textos. É muito mais inteligente voltar nosso foco para as palavras que
têm algum significado para nós do que destacar aquelas que não conhecemos. Além disso, ao grifar,
você acaba relendo as informações de uma maneira mais lenta, o que faz com que perceba certos
detalhes que não havia percebido antes. É uma forma de quantificar em porcentagem aproximada o
quanto se sabe daquele texto. É preciso lembrar que há um número muito grande de palavras repetidas
nos textos e isso facilita para o estudante, pois ele poderá grifar mais de uma vez a mesma palavra.

Scanning: esta técnica de leitura visa dar agilidade na busca por informações específicas. Muitas
vezes, após ler um texto, nós queremos reencontrar alguma frase ou alguma palavra já lida
anteriormente. Para efetuar esta busca não precisamos ler o texto inteiro de novo, podemos
simplesmente ir direto ao ponto aonde podemos encontrar tal informação. Isso é o scanning, significa
encontrar respostas de uma forma rápida e direta sem perder tempo relendo o texto todo. Esta técnica
em geral deve ser aplicada após uma ou mais leituras completas do texto em questão. Assim o leitor
diminuirá o risco de confundir informações, perder tempo ou de dar respostas erradas. Se desejar, o
estudante pode ler o que os exercícios pedirão antes de fazer o scanning, pois assim ele irá selecionar
mais facilmente o que for mais importante para responder àquelas questões direcionando-se melhor.

Lexical Inference (inferência lexical): Inferir significa deduzir. Às vezes será preciso deduzir o
sentido de um termo, decifrando o que ele quer dizer. Mas isso não pode ser feito de qualquer maneira.
Para inferirmos bem, é necessário entender o significado daquela palavra desconhecida através do
contexto no qual ela está inserida, observando as palavras vizinhas, as frases anteriores e posteriores,
o parágrafo onde ela está, as noções gerais que temos do texto, etc. Precisamos observar o meio no
qual a palavra está posta. Neste caso teremos de nos fazer valer de nossos conhecimentos de classes
gramaticais (substantivos, adjetivos, preposições, verbo, etc.), de afixos, de singular e plural,
conhecimento sobre a estrutura de textos, etc. Tudo isso em conjunto pode ajudar numa
aproximação do sentido real daquele termo que não sabemos.

Há estudos que relacionaram as palavras que mais aparecem em textos e livros técnicos em língua
inglesa. Desses estudos foram feitas diferentes listas com as 500 palavras mais comuns, ou as 700
palavras mais comuns. Para facilitar seu estudo, incluímos aqui as 318 mais comuns para serem
estudadas. Ao memorizar estas palavras você obterá um magnífico subsídio preparando-se para
enfrentar qualquer texto. Você verá que várias destas palavras já são conhecidas por você, assim, na
verdade, terá que memorizar bem menos destas. Um número bem significativo delas está presente em
qualquer tipo de texto. Quanto mais palavras você souber, mais poderá grifar! Apoie-se nelas e bom
estudo!

2
1103362 E-book gerado especialmente para MARCUS ICARO CRUZ DA SILVA
001 although embora 060 common comum
002 able capaz 061 country país, zona rural
003 about sobre, aproximadamente 062 course curso
004 above acima 063 day dia
005 according to de acordo com 064 dead morto
006 after depois, após 065 death morte
007 again novamente, de novo 066 different diferente
008 against contra 067 door porta
009 age idade 068 down para baixo
010 air ar 069 during durante
011 all tudo 070 each cada
012 almost quase 071 earth terra (planeta)
013 alone só, sozinho 072 either... or ou... ou
014 along ao longo de 073 emperor imperador
015 already já 074 empire império
016 also também 075 end fim
017 always sempre 076 enemy inimigo
018 among entre (3 ou mais coisas) 077 England Inglaterra
019 an um, uma 078 enough suficiente
020 ancient antigo 079 even mesmo
021 and e 080 ever em qualquer momento, já
022 another um outro 081 every cada, todo
023 any algum(a), qualquer 082 eye olho
024 anything qualquer coisa 083 fact fato
025 arm braço 084 family família
026 army exército 085 far distanste, longe
027 around em torno de, perto de 086 father pai
028 art arte 087 fear medo
029 as como, assim como 088 few poucos(as)
030 at em, às 089 fire fogo
031 authority autoridade 090 first primeiro
032 away distante, longe 091 five cinco
033 back de volta, atrás 092 foot/feet pé/pés
034 because porque 093 footnote notas de rodapé
035 before antes 094 for para, por
036 behind atrás 095 force força, forçar
037 best melhor (superlativo) 096 four quatro
038 better melhor (comparativo) 097 France França
039 between entre (2 coisas) 098 free livre, grátis
040 beyond além 099 French francês
041 big grande 100 friend amigo(a)
042 black preto(a) 101 from de (origem)
043 blood sangue 102 full completo, cheio
044 body corpo 103 general geral
045 both ambos(as) 104 girl menina, garota
046 boy menino, garoto 105 God Deus
047 brother irmão 106 gold ouro
048 but mas, porém, exceto 107 good bom(ns), boa(s)
049 by próximo a, perto de, por 108 government governo
050 captain capitão 109 great grande, maravilhoso
051 care cuidado 110 ground chão
052 case caso 111 half metade
053 certain certo 112 hand mão/entregar
054 chapter capítulo 113 he ele (pessoa)
055 character caráter, personalidade 114 head cabeça, líder
056 child criança 115 heart coração
057 children crianças 116 her dela (pessoa)
058 church igreja 117 here aqui
059 city cidade 118 high alto

3
1103362 E-book gerado especialmente para MARCUS ICARO CRUZ DA SILVA
119 him ele, o (pessoa) 178 natural natural
120 himself ele mesmo (pessoa) 179 nature natureza
121 his dele (pessoa) 180 near próximo, perto
122 history história 181 neither...nor nem...nem
123 home casa, lar 182 never nunca
124 horse cavalo 183 new novo(a)(s)
125 hour hora 184 next próximo, a seguir
126 house casa 185 night noite
127 how como 186 no não
128 however entretanto 187 non não
129 human humano 188 not não
130 hundred cem, centena 189 nothing nada
131 idea idéia 190 now agora
132 if se 191 number número
133 ill doente 192 of de
134 in em, dentro (de) 193 off afastado, desligado
135 indeed de fato, realmente 194 often frequentemente
136 into para dentro de 195 old velho(s), velha(s)
137 it ele(a) (coisa, animal) 196 on sobre, em cima
138 its seu, sua, (coisa, animal) 197 once uma vez
139 itself a si mesmo (coisa, animal) 198 one um, uma
140 just apenas, justo 199 only apenas, único, somente
141 kind tipo, gentil 200 or ou
142 king rei 201 other outro(a)
143 knowledge conhecimento 202 our nosso(a), nossos(as)
144 land terra 203 out fora
145 large largo, amplo, grande 204 over acima, encerrado
146 law lei 205 part parte
147 (at) least (pelo) menos 206 peace paz
148 left esquerdo(a) 207 people pessoas
149 less menos 208 perhaps talvez
150 life vida 209 period período
151 light luz, leve 210 person pessoa
152 little pouco(a) 211 place lugar
153 long longo 212 point ponto
154 longer mais longo 213 poor pobre
155 love amor 214 power poder, força
156 man/men homem/homens 215 present presente
157 manner maneira 216 prince príncipe
158 many muitos (as) 217 public público
159 master mestre 218 quite completamente, muito
160 matter matéria 219 rather preferencialmente
161 me me, mim 220 reason razão
162 miles milhas 221 reign reino
163 mind mente 222 religion religião
164 mine meu(s), minha(s) 223 room cômodo, quarto
165 moment momento 224 round redondo
166 money dinheiro 225 same mesmo(a)
167 more mais 226 sea mar
168 morning manhã 227 second segundo
169 most mais 228 set conjunto
170 mother mãe 229 seven sete
171 Mr. senhor 230 several vários(as)
172 Mrs. senhora 231 she ela (pessoa)
173 much muito(a) 232 short pequeno(a), curto(a)(s)
174 my meu(s), minha(s) 233 side lado
175 myself eu mesmo 234 sight vista, visão
176 name nome 235 since desde
177 nation nação 236 sir senhor

4
1103362 E-book gerado especialmente para MARCUS ICARO CRUZ DA SILVA
237 six seis 278 town cidade
238 small pequeno(s), pequena(s) 279 true verdade
239 so então 280 truth verdade
240 some algum(a), alguns(mas) 281 twenty vinte
241 something algo, alguma coisa 282 two dois
242 sometimes algumas vezes 283 under sob
243 son filho 284 until/till até (que)
244 soon logo, em breve 285 up para cima
245 spirit espírito 286 upon sobre
246 state estado, situação 287 us nos, a nós
247 still ainda 288 very muito
248 street rua 289 voice voz
249 strength força 290 war guerra
250 strong forte 291 water água
251 subject assunto, sujeito 292 way caminho, maneira, jeito
252 such tão 293 we nós
253 sure certo (certeza) 294 well bem
254 ten dez 295 what o que, qual, quais
255 than do que 296 when quando
256 that aquele(a), esse(a) 297 where onde
257 the o, a, os, as 298 whether se
258 their deles, delas 299 which (o,a) qual, (os, as) quais
259 them eles, os 300 while enquanto
260 themselves eles mesmos 301 white branco
261 then então, em seguida 302 who/whom quem, a quem
262 there lá 303 whole complete, inteiro
263 therefore por esta razão 304 whose de quem, cujo(a)(s)
264 these estes(as) 305 why por que?
265 they eles, elas 306 wife esposa
266 thing coisa 307 with com
267 thirty trinta 308 within dentro de
268 this este(a), isto 309 without sem
269 those aquele(as), esses(as) 310 woman/women mulher/mulheres
270 thousand mil, milhar 311 word palavra
271 three três 312 world mundo
272 through através 313 year ano
273 time tempo, momento, vez 314 yes sim
274 to para, em direção a 315 yet ainda, já
275 together junto(a)(s) 316 you você(s)
276 too também 317 young jovem
277 towards na direção de 318 yours seu(s), sua(s)

Vamos a um exemplo:

Fonte: http://goo.gl/R7fIJz

5
1103362 E-book gerado especialmente para MARCUS ICARO CRUZ DA SILVA
Neste caso temos uma antiga propaganda da cola cola que demonstra exatamente os passos da
leitura lexical. Primeiro vamos analisar as palavras grandes, o skimming. “Beverage”, “Men” e “Ladies”.
Beverage não é uma palavra tão comum, mas pelo contexto da Coca Cola talvez o estudante chegue a
sua tradução que é bebida. Já “men” e “ladies” a maioria das pessoas já conhece que são homens e
mulheres. Aqui já temos o contexto da propaganda, que novamente desde o começo era obvio por ser
uma propaganda da Coca Cola. Vamos as palavras menores, porém iremos citar apenas as cogntas ou
as palavras que a maioria das pessoas conheça.
Do lado esquerdo: “business and professional” Negócios e profissionais.
Do lado direito: “Students ... Athletes”. Estudantes e atletas.
No centro: “Favorite drink for ladies”. A bebida favorita das mulheres.
E nos cantos inferiores 5c que se refece a 5 centavos, ou seja, o preço.

Vamos estudar agora a gramática da Língua Inglesa.

SUBSTANTIVOS

Substantivos, que no inglês são conhecidos como nouns, são palavras que dão nome a pessoas,
lugares, coisas, conceitos, ações, sentimentos, etc. Também chamados de nomes, eles funcionam de
muitas maneiras nas sentenças. Na maioria das vezes, posicionam-se como o sujeito de um verbo,
funcionando como o ator ou agente dele. Os nomes também podem receber uma ação quando
funcionam como objeto do verbo. Quando atuam como sujeitos ou objetos, os substantivos podem ser
apenas uma palavra, frases, ou cláusulas.

Exemplos:

The plane crashed. (substantivo como sujeito da frase)


He kicked the dog. (substantivo como objeto direto do verbo)

A maioria dos substantivos forma o plural com o acréscimo de -s. Por exemplo:

Singular Plural
dog dogs
cat cats

Quando o substantivo termina em -y e é precedido por consoante, faz-se o plural com -ies.

a canary canaries
a library libraries
a pony ponies
a story stories

Se o substantivo termina em -s, -ss, -z, -sh, -ch, -x (exceção: ox => oxen), acrescentamos -es para
formar o plural:

A beach two beaches


A church two churches
A dish two dishes
A fox two foxes

Existem algumas formas irregulares de plural. Alguns exemplos comuns são:

Woman women
Man men
Child children
Tooth teeth
Foot feet
Goose geese
Mouse mice
Person people

6
1103362 E-book gerado especialmente para MARCUS ICARO CRUZ DA SILVA
Para alguns terminados em -f ou -fe, trocamos estas letras por -ves. Para outros, apenas usamos -s:

Knife knives
Wife wives
Life lives

SUBSTANTIVOS CONTÁVEIS E NÃO CONTÁVEIS

Fonte: http://goo.gl/oiXKLN

Na tabela acima nós temos os exemplos de alguns alimentos divididos nas duas categorias que
iremos explicar abaixo, contáveis e incontáveis. Aqui iremos também traduzir todos os alimentos da
lista, assim o estudante não precisa ficar procurando em um dicionário um por um.

Countables – Contáveis Uncountables – Incontáveis


Bun – Bolinho Bread – Pão
Sandwich – Sanduiche Fruit – Fruta
Apple – Maça Juice – Suco
Orange – Laranja Meat – Carne
Burguer – Hamburguer Rice – Arroz
Fries – Batata frita Cereal – Cereal
Eggs – Ovos Jam – Geléia
Salad – Salada Milk – Leite
Vegetables – Vegetais Coffee – Café
Cookies – Biscoitos Sugar – Açucar
Potatoes – Batatas Flour – Farinha
Tomato – Tomates Oil – Óleo

7
1103362 E-book gerado especialmente para MARCUS ICARO CRUZ DA SILVA
Carrot – Cenoura Salt – Sal
Hot Dog – Cachorro quente Soup – Sopa
Candies – Doces Tea – Chá
Olives – Azeitonas Cottage Cheese – Coalhada
Peanuts – Amedoins Pasta – Massa
Pancakes – Panquecas Honey – Mel
Onion – Cebola Water – Água
Watermelon – Melancia Cheese – Quejo
Pea – Ervilha Butter – Queijo
Grapes – Uvas Seafood – Frutos do mar
Cherries – Cerejas Mustard – Mostarda

Contáveis são aqueles substantivos que podemos enumerar e contar, ou seja, que podem possuir
tanto forma singular quanto plural. Eles são chamados de countable nouns ou de count nouns, em
inglês.

Por exemplo, podemos contar pencil. Podemos dizer one pencil, two pencils, three pencils, etc.

Incontáveis são os substantivos que não possuem forma no plural. Eles são chamados de
uncountable nouns, de non-countable nouns, ou até de non-count nouns, em inglês. Podem ser
precedidos por alguma unidade de medida ou quantificador. Em geral, eles indicam substâncias,
líquidos, pós, conceitos, etc., que não podemos dividir em elementos separados. Por exemplo, não
podemos contar “water” em por exemplo one water ou two waters. Podemos, sim, contar "bottles of
water" ou "liters of water", mas não podemos contar “water” em sua forma líquida.

Outros exemplos de substantivos incontáveis são: music, art, love, happiness, advice, information,
news, furniture, luggage, rice, sugar, butter, water, milk, coffee, electricity, gas, power, m oney, etc.

Em geral, estudantes de língua inglesa têm dificuldade de saber quando um substantivo é contável e
quando é não-contável. As dicas são sempre conferir a informação num bom dicionário e também tentar
memorizar alguns dos mais comuns para agilizar o seu estudo. Nos dicionários, normalmente você
encontra o símbolo [U] para identificar os uncountable nouns e [C] para os countable nouns.

Em várias situações necessitamos de fazer o uso de determinantes/quantificadores em conjunto com


substantivos contáveis e incontáveis.

Há determinantes específicos para os incontáveis: a little, little, less, much.

Exemplos:

I have little time to study today.


Eu tenho pouco tempo para estudar hoje.

She has little patience with her kids.


Ela tem pouca paciência com seus filhos.

He demonstrates less aptitude.


Ele demonstra menos aptidão.

Judy and her husband have much money.


Judy e seu marido têm bastante dinheiro.

E há alguns específicos para uso com substantivos contáveis: a few, few, fewer, many.

Exemplos:

There are a few coins in my wallet.


Há algumas moedas na minha carteira.

8
1103362 E-book gerado especialmente para MARCUS ICARO CRUZ DA SILVA
Few people went to the show.
Poucas pessoas foram ao show.

We can see fewer cars on the streets today.


Nós podemos ver menos carros nas ruas hoje.

He has many friends.


Ele tem muitos amigos.

Existe ainda o determinante a lot of que pode ser utilizado tanto para substantivos contáveis como
incontáveis. Ele é apelidade de “coringa” porque serve para ambas as categorias. Mas lembre-se de
focar os estudos nos demais principalmente no much e many. Os concursos sempre focm mais no
much e many na tentativa de confundir o candidato.

Exemplo:

I have a lot of money.


Eu tenho um monte de dinheiro.

I have much money.


Eu tenho muito dinheiro.

There are a lot of cars in the street tonight.


Tem um monte de carros na rua esta noite.

There are many cars in the street tonight.


Tem muitos carros na rua esta noite.

MODIFICADORES DE SUBSTANTIVOS

Modifiers são palavras, locuções, frases, ou cláusulas que qualificam o significado de outras
palavras. O termo é bem genérico: qualquer parte da fala que funciona como um adjetivo ou advérbio é
um modificador.

Nos exemplos abaixo, o modifier está em negrito e a palavra que ele modifica está sublinhada; a
função do modificador está descrita abaixo.

Adjetivos — descrevem ou modificam nomes. Uma locução adjetiva ou cláusula adjetiva funciona da
mesma maneira que uma simples palavra funcionaria.

Exemplos:

The yellow balloon flew away over the crying child.


O balão amarelo voou sobre a criança chorona.

O adjetivo yellow modifica o substantivo balloon; crying modifica child.

Artigos — são palavras que acompanham os substantivos e tem função de classifica-los.

Exemplos:

The killer selected a knife from an antique collection.


O assassino escolheu uma faca de uma antiga coleção.

The, a, e an são artigos que especificam ou delimitam seus respectivos substantivos.

Advérbios — descrevem verbos, adjetivos, ou outros advérbios, completando a ideia de como,


quanto ou quando. Uma locução adverbial ou cláusula adverbial funciona da mesma forma que um
único advérbio funcionaria.

9
1103362 E-book gerado especialmente para MARCUS ICARO CRUZ DA SILVA
Exemplos:

The woman carefully selected her best dress for the party.
A mulher cuidadosamente escolheu seu melhor vestido para a festa.

Carefully é um advérbio que modifica o verbo selected.

CASOS POSSESSIVOS COM ‘S

Quando falamos de posse, geralmente em inglês se usa os pronomes adjetivos ou possessivos.


Porém em algumas situações nós queremos relacionar o objeto em questão diretamente ao nome de
seu proprietário.

Exemplos:

The house Anna.


A casa de Anna.

The book of Steve.


O livro de Steve.

Em Inglês, existe um "atalho" para este tipo de situação usando o 's.

Exemplos:

Anna's house.
Steve's book.

ATENÇAO: Não podemos confundir este 's possessivo com o 's (abreviação to verbo TO BE “is”).

Exemplos:

Wilson's a teacher.
Wilson é um professor.

Wilson's teacher.
O professor de Wilson.

Outros detalhes quanto ao 's.


Caso existam múltiplos "donos" o 's vai apenas no último.

Exemplo:

Kate and Cindy's parents.


Os pais de Kate e Cindy.

Caso o "dono" seja terminado em S por conta de sua pluralidade, fazemos apenas o acréscimo do '
já no S existente ficando s’.

Exemplos:

My brother's house.
A casa do meu irmão.
(Apenas um irmão)

My brothers' house.
A casa dos meus irmãos.
(Mais de um irmão – plural)

10
1103362 E-book gerado especialmente para MARCUS ICARO CRUZ DA SILVA
O 's é muito usado para informação de grau de parentesco o que pode confundir um pouco o
estudante, portanto faça a leitura com calma deste tipo de estrutura.

Exemplos:

Jack is my father's brother.


Jack é o irmão de meu pai.

Peter is her brother's best friend.


Peter é o melhor amigo do irmão dela.

William is David's last name.


William é o sobre nome do David.

ARTIGOS

Fonte: http://goo.gl/UbQWRi

E geral, emprega-se o artigo definido the antes de substantivos com a finalidade de especificá-los.

Exemplo:

The boy is late.


O menino está atrasado.

Às vezes, pode ocorrer a presença de um ou mais adjetivos entre o artigo the e o substantivo.

Exemplos:

The little boy is late.


O pequeno menino está atrasado.

The little good boy is late.


O pequeno bom menino está atrasado.

Na língua inglesa, os artigos indefinidos são: a e an. Ambos são traduzidos como: um ou uma. O
artigo indefinido no inglês não tem plural. Só podemos usar a/an antes de substantivos que estejam no
singular.

Exemplos:

A car.
Um carro.

11
1103362 E-book gerado especialmente para MARCUS ICARO CRUZ DA SILVA
A house.
Uma casa.

Assim como no artigo definido the pode existir um ou mais adjetivos entre o artigo e o
substantivo, o mesmo pode acontecer com os artigos indefinidos a/an.

Exemplos:

A beautiful day.
Um lindo dia.

A hot summer.
Um verão quente.

A diferença entre o artigo a para o artigo an é a palavra que vem após estes. Se a próxima palavra
(substantivo ou adjetivo) tiver o som de consoante em sua pronuncia, utilizamos a. Se o som for de
vogal em sua pronuncia, utilizamos an,

Exemplos:

A cow.
Uma vaca.

A desk.
Uma carteira.

An elephant.
Um elefante.

An envelope.
Um envelope.

I have an English dictionary.


Eu tenho um dicionário de Inglês.

A função do an é acelerar a pronuncia uma vez que o an já se junta na pronuncia da próxima


palavra.

Exemplo:

This is an American car.


Este é um carro americano.

A pronuncia da frase acima não é:


This is an (pausa) American car.

A pronunca correta é:
This is anAmerican car.

Determinantes, também conhecidos como quantificadores, são usados antes de substantivos para
fazer referência a algo específico ou a um grupo em geral. São palavras ou expressões usadas para
indicar e fornecer informações a respeito da quantidade de algo.

Os determinantes específicos são:

O artigo definido: the


Os pronomes demonstrativos: this, that, these, those
Os pronomes adjetivos possessivos: my, your, his, her, its, our, their

12
1103362 E-book gerado especialmente para MARCUS ICARO CRUZ DA SILVA
Exemplos:

The dog barked at the boy.


O cachorro latiu para o garoto.

These apples are not good to eat.


Estas maçãs não estão boas para comer.

Their train was early.


O trem deles estava adiantado.

Você usa quantificadores mais gerais para falar sobre pessoas ou coisas sem dizer exatamente
quem ou o quê eles são.

Os determinantes/quantificadores gerais são:

a, an, a few, little, all, another, any, both, each, either, enough, every, few, fewer, less, little,
many, more, most, much, neither, no, other, several, some.

Exemplos:

A woman sat under an umbrella.


Uma mulher sentou-se embaixo de um guarda-chuva.

Have you got any literature books?


Você tem algum livro de literatura?

There is not enough food for everyone.


Não há comida suficiente para todos.

I have no idea to give.


Eu tenho nenhuma ideia para dar.

She has little money in her purse.


Ela tem pouco dinheiro em sua bolsa.

There are fewer students in class today.


Há menos alunos na classe hoje.

Os pronomes demonstrativos servem para apontar, demonstrar, indicar algum animal, objeto ou
pessoa. São quatro: this, these, that e those. No inglês não existem pronomes demonstrativos
masculinos ou femininos como temos no Português.

Singular Plural

Perto This That

Longe These Those

Usa-se this para referir-se a algo no singular e que está perto de quem fala.
Usa-se that para referir-se a algo no singular e que está longe de quem fala.

Usa-se these para referir-se a algo no plural e que está perto de quem fala.
Usa-se those para referir-se a algo no plural e que está longe de quem fala.

Exemplos:

13
1103362 E-book gerado especialmente para MARCUS ICARO CRUZ DA SILVA
This car is modern.
Este carro é moderno.

These clothes are very cheap.


Estas roupas estão muito baratas.

That is my best friend.


Aquele é meu melhor amigo.

Those are the new doctors.


Aqueles são os novos médicos.

Resumindo:

This (singular, perto) - Este, esta, isto.


That (singular, longe) - Aquele, aquela, aquilo.

These (plural, perto) - Estes, estas.


Those (plural, longe) - Aqueles, aquelas.

ADJETIVOS E COMPARATIVOS

Fonte: http://goo.gl/lSsQ0l

14
1103362 E-book gerado especialmente para MARCUS ICARO CRUZ DA SILVA
Na ilustração:
Sleepwalker – sonâmbulo.
Ugly – feio.
Surprised – surpreso.
Angry – bravo.
Hungry – com fome.
Tired – cansado.
In a hurry – com pressa.

Adjetivos são palavras ou grupo de palavras que indicam características dos substantivos, definindo-
os, delimitando-os ou modificando-os.

Ao contrário do que ocorre na língua portuguesa, os adjetivos em inglês não possuem forma singular,
plural, masculina nem feminina. Existe apenas a forma singular para ambos os sexos.

She is beautiful.
Ela é linda.

They are beautiful.


Elas (ou eles) são lindos.

His car is red


O carro dele é vermelho.

Their cars are red.


O carro deles é vermelho.

Anna is intelligent. Jack is intelligent.


Anna é inteligente. Jack é inteligente.

Quando o(s) adjetivo(s) aparece(m) junto a um substantivo, tal abjetivo(s) deve(m) vir antes do
substantivo:

Exemplos:

This is a big city.


Esta é uma grande cidade.

They live in a huge white house.


Eles moram em uma enorme casa branca.

Marcos is a soccer player.


Marcos é um jogador de futebol.

Os adjetivos em inglês também possuem graus diversos, assim como ocorre em português.

Não existe uma regra para determinar-nos quando um adjetivo é curto ou longo, por exemplo se
baseando no número de letras ou algo do tipo. O estudante deve se familiarizar com os adjetivos já os
classificando entre longos e curtos.

Grau Comparativo de Igualdade (as + adjetivo + as) = (tão/tanto... quanto)

Exemplos:

Dereck is as short as Fred.


Dereck é tão baixo quanto Fred.

15
1103362 E-book gerado especialmente para MARCUS ICARO CRUZ DA SILVA
That motorcycle is as fast as this one.
Aquela moto é tão rápida quanto esta.

Julie is as beautiful as Sharon.


Julie é tão bela quanto Sharon.

Grau Comparativo de Superioridade (adjetivo curto + er + than) = (mais... do que..)

Fonte: http://goo.gl/GnJiiN

Na ilustração:

More beautiful – mais bonita.


Uglier – mais feia.

Exemplos:

Adjetivo: Strong (forte)


Tim is stronger than Peter.
Tim é mais forte do que Peter.

Adjetivo: Tall (alto)


An elephant is taller than a lion.
Um elefante é mais alto que um leão.

Adjetivo: Thin (magro)


Nancy is thinner than Sue.
Nancy é mais magra do que Sue.

Grau Comparativo de Superioridade (more + adjetivo longo + than) = (mais... do que..)

Exemplos:

Adjetivo: Intelligent (Inteligente)


Dave is more intelligent than his brother.
Dave é mais inteligente que seu irmão.

Adjetivo: Careful (cuidadoso)


He is more careful than his father when driving.
Ele é mais cuidadoso que seu pai quando está dirigindo.

Adjetivo: Comfortable (confortável)


This house is more comfortable than the other.
Esta casa é mais confortável que a outra.

Grau Comparativo de Inferioridade (less + adjetivo + than) = (menos... do que...)

16
1103362 E-book gerado especialmente para MARCUS ICARO CRUZ DA SILVA
Exemplos:

Adjetivo: Famous (famoso)


Christopher is less famous than Brad.
Christopher é menos famoso do que Brad.

Adjetivo: Hot (quente)


Your city is less hot than mine.
Sua cidade é menos quente do que a minha.

Adjetivo: Difficult (difícil)


This language is less difficult than the others.
Esta língua é menos difícil do que as outras.

Os graus de comparativo devem ser utilizados apenas quando estamos comparando duas
pessoas ou duas coisas. Por outro lado os graus de superlativo (como veremos abaixo) são
utilizados quando estamos comparando três ou mais pessoas ou coisas. Geralmente as frases se
referem a uma totalidade (da classe, da cidade, etc.).

Passemos então a estudar, agora, o grau superlativo:

Grau Superlativo de Superioridade (the + adjetivo curto + est) = (o mais...)

Exemplos:

Adjetivo: Cheap (barato)


This is the cheapest restaurant in town.
Este é o restaurante mais barato da cidade.

Adjetivo: Tall (alto)


Jennifer is the tallest girl in the group.
Jennifer é a garota mais alta do grupo.

Adjetivo: Dry (seco)


This is the driest region of the state.
Esta é a região mais seca do estado.

Grau Superlativo de Superioridade (the most + adjetivo longo) = (o mais...)

Exemplos:

Adjetivo: Modern (moderno)


This is the most modern TV set nowadays.
Este é o aparelho de TV mais moderno do momento.

Adjetivo: Handsome (bonito)


He is the most handsome actor in the movies.
Ele é o ator mais bonito do cinema.

Adjetivo: Famous (famoso)


Messy is the most famous soccer player now.
Messy é o jogador de futebol mais famoso agora.

Grau Superlativo de Inferioridade (the least + adjetivo) = (o menos...)

Adjetivo: Important (importante)


This is the least important detail.
Este é o detalhe menos importante.

17
1103362 E-book gerado especialmente para MARCUS ICARO CRUZ DA SILVA
Adjetivo: Nervous (nervoso)
I’m always the least nervous during the tests.
Sempre sou o menos nervoso durante as provas.

Adjetivo: Safe (seguro)


That region is the least safe of the city.
Aquela região é a menos segura da cidade.

Adjetivos irregulares. Aqueles que sua forma no comparativo e superlativo mudam totalmente sem
seguir qualquer regra pré-definida.

Exemplos:

Adjetivo Comparativo Superlativo


Bad (mau) worse the worst
Good (bom) better the best
Far (longe) farther the farthest
Far (mais/complementar) further the furthest
Little (pouco) less the least
Many (muitos/as) more the most
Much (muito/a) more the most

PRONOMES PESSOAIS

Há dois tipos de pronomes pessoais: sujeitos e objetos.

Pronome Pessoal Sujeito Tradução Pronome Pessoal Objeto

I eu Me

You você You

He ele Him

She ela Her

It ele/ela (para coisas ou animais) It

We nós Us

You vocês You

They eles/elas Them

Os pronomes pessoais sujeitos vêm antes do verbo, como sujeito da frase.


Os pronomes pessoais objetos vêm depois de verbo ou de preposição. Além de virem depois, o
verbo principal da frase está fazendo uma ação relacionada ao pronome pessoal objeto em questão.
A tabela criada acima já trás os sujeitos do lado esquerdo e os objetos do lado direto justamente
para fazer a representação descrita acima, facilitando assim o entendimento por parte do candidato.

Exemplos:

She loves him a lot.


Ela ama ele muito.

I saw her at the party yesterday.

18
1103362 E-book gerado especialmente para MARCUS ICARO CRUZ DA SILVA
Eu vi ela na festa ontem.

We are going to meet them in front of the stadium.


Nós vamos encontrar eles na frente do estádio.

They waited for us for two hours.


Eles esperaram por nós por duas horas.

Can you send this e-mail for me, please?


Você pode enviar este e-mail para mim, por favor?

PRONOMES POSSESSIVOS

Na tirinha:
Ok, Ok I’ll put mine down if you put yours down first.
Ok, Ok, eu irei abaixar o meu se você abaixar o seu primeiro.

Há dois tipos de pronomes possessivos: adjetivos e substantivos.

Pron. Possessivo Adjetivo Tradução Pron. Possessivo Substantivo

My meu(s)/minha(s) Mine

Your seu/sua Yours

His dele His

Her dela Hers

Its dele/dela (coisas ou animais) Its

Our nosso(s)/ nossa(s) Ours

Your seus/suas Yours

Their deles/delas Theirs

Os pronomes possessivos adjetivos vem antes do substantivo.


Os pronomes possessivos substantivos podem vir após o substantivo ou podem substituir o
substantivo a qual se referem assim reduzindo a frase.
Para facilitar o entendimento do candidato, nos exemplos abaixos os substantivos ficarão
sublinhados.

19
1103362 E-book gerado especialmente para MARCUS ICARO CRUZ DA SILVA
Exemplos:

His kid is playing with hers.


O filho dele está brincando com o dela.

No exemplo acima:
His – pronome possessivo adjetivo, antes do substantivo “kid”.
Hers – pronome possessivo substantivo, substituindo o substantivo “kid”, para evitar a repetição da
mesma palavra várias vezes na mesma frase.

Exemplos:

My friends went to the club with yours.


Meus amigos foram ao clube com os seus.

Our mother likes pizza.


Nossa mãe gosta de pizza.

Did you prefer his presentation or hers?


Você preferiu a apresentação dele ou a dela?

PRONOMES INTERROGATIVOS

Fonte: http://goo.gl/y7dkBK

Os Pronomes Interrogativos também chamados de Question Words, são utilizados para obtermos
informações mais específicas a respeito de algo ou alguém. As perguntas formuladas com eles são
conhecidas também como wh-questions porque todos estes pronomes interrogativos possuem as
letras wh. Na grande maioria das vezes, os pronomes interrogativos são posicionados antes de verbos
auxiliares ou modais, no início de frases.

What – O que, que, qual - usado para questões com opções mais amplas de resposta.

Exemplos:

What time is it now?


Que horas são agora?

20
1103362 E-book gerado especialmente para MARCUS ICARO CRUZ DA SILVA
What are you doing here?
O que você está fazendo aqui?

Where – Onde

Exemplos:

Where do you work?


Onde você trabalha?

Where do your kids study?


Onde seus filhos estudam?

When – Quando

Exemplos:

When did they move?


Quando eles se mudaram?

When did you travel to Europe?


Quando você viajou para a Europa?

Who – Quem

Exemplos:

Who is that girl?


Quem é aquela garota?

Who arrived first in the race?


Quem chegou primeiro na corrida?

Why – Por que

Exemplos:

Why did you cry?


Por que você chorou?

Why are you late for class?


Por que você está atrasado para a aula?

Whom – Quem – mais formal que who e geralmente vem após uma preposição.

Exemplos:

With whom did you go to the park?


Com quem você foi ao parque?

To whom were you speaking last night?


Com quem você estava falando ontem à noite?
Whose – De quem

Exemplos:

Whose pen is this?


De quem é esta caneta?

21
1103362 E-book gerado especialmente para MARCUS ICARO CRUZ DA SILVA
Whose mansion is that?
De quem é aquela mansão?

Which – Qual, quais - usado para questões com opções limitadas de resposta.

Exemplos:

Which of those girls is your sister?


Qual daquelas meninas é a sua irmã?

Which color do you prefer: yellow or blue?


Qual cor você prefere: amarelo ou azul?

Existem diversas formas compostas dos pronomes interrogativos. Podemos juntar outras palavras
a estes antes dos verbos auxiliares, para especificar alguma informação.

Exemplos:

What kind of movies do you like?


Que tipo de filmes você gosta?

What sports do you practice?


Que esportes você pratica?

What soccer team are you a fan of?


Para que time de futebol você torce?

How often do you go to the gym?


Com que frequência você vai à academia?

How long is the Amazon river?


Qual o comprimento do rio Amazonas?

How much does this newspaper cost?


Quanto custa este jornal?

How many brothers do you have?


Quantos irmãos você tem?

How good are you at tennis?


O quanto você é bom em tênis?

How old are you?


Quantos anos você tem?

How far is São Paulo from Rio?


Qual a distância entre São Paulo e Rio?

How deep is this river?


Quão profundo é este rio?

Quando uma pergunta questiona sobre o sujeito da oração, não se usa verbo auxiliar. Assim, o
pronome interrogativo inicia a pergunta seguido das outras palavras na ordem afirmativa. Observe:

Exemplos:

Who likes to eat vegetables?


Quem gosta de comer vegetais?

22
1103362 E-book gerado especialmente para MARCUS ICARO CRUZ DA SILVA
What broke the window?
O que quebrou a janela?

Who speaks English in this room?


Quem fala inglês nesta sala?

How many people survived the accident?


Quantas pessoas sobreviveram ao acidente?

Em muitos casos, as perguntas são finalizadas por preposições que complementam seu sentido:

Exemplos:

Where are you from?


De onde você é?

What is your city like?


Como é a sua cidade?

Who did you play against?


Contra quem você jogou?

Where did you send the letter to?


Para onde você enviou a carta?

What is this for?


Para que é isto?

VERBOS

Quanto à forma, podemos classificar os verbos ingleses em Regulares, Irregulares e Modais.

São chamados de regulares os verbos que geralmente seguem a mesma regra.


No caso do presente, verbos regulares são aqueles que recebem -s:

Exemplo:

Play – plays, sing – sings

No caso do passado, verbos regulares são aqueles que recebem -ed:

23
1103362 E-book gerado especialmente para MARCUS ICARO CRUZ DA SILVA
Exemplo:

Play – played, cook – cooked

Verbos irregulares são aqueles que não seguem uma mesma regra.
Tanto no caso do presente ou do passado, os verbos sofrem modificações individuais.

Exemplos:

Presente:
have – has, do – does

Passado:
Sing – sang, eat – ate

Os verbos irregulares não têm uniformidade quanto à escrita do passado simples e do particípio.
Confira os três últimos exemplos na tabela abaixo.

Infinitivo Simple Past tense Past Participle Tradução

to accept accepted accepted aceitar


to add added added adicionar, somar
to arrive arrived arrived chegar
to be was, were been ser, estar
to begin began begun começar, iniciar
to buy bought bought comprar

Abaixo segue uma tabela dos verbos mais utilizados na língua inglesa. Assim como as palavras mais
comuns (aquela lista não possui verbos) os verbos também são parte fundamental das frases. Quanto
mais verbos o estudante souber – mais facilmente ele entenderá todas as frases de um texto.

# Infinitive Simple Past Tradução


1 Accept Accepted Aceitar
2 Agree Agreed Concordar
3 Answer Answered Responder
4 Appear Appeared Aparecer
5 Arrive Arrived Chegar
6 Ask Asked Perguntar
7 Attack Attacked Atacar
8 Bake Baked Assar
9 Be Was, were Ser, estar
10 Become Became Tornar-se
11 Begin Began Começar
12 Believe Believed Acreditar, crer
13 Bet Bet Apostar
14 Bite Bit Morder, picar
15 Bleed Bled Sangrar
16 Borrow Borrowed Pedir emprestado
17 Break Broke Quebrar, interromper
18 Bring Brought Trazer
19 Build Built Construir
20 Burn Burned, burnt Queimar
21 Buy Bought Comprar
22 Call Called Ligar, chamar

24
1103362 E-book gerado especialmente para MARCUS ICARO CRUZ DA SILVA
23 Cancel Canceled Cancelar
24 Carry Carried Carregar
25 Celebrate Celebrated Celebrar, comemorar
26 Change Changed Trocar, mudar
27 Chat Chatted Bater papo
28 Clap Clapped, clapt Bater palma
29 Clean Cleaned Limpar
30 Climb Climbed Subir, escalar
31 Close Closed Fechar
32 Come Came Vir, chegar
33 Complain Complained Reclamar
34 Cook Cooked Cozinhar
35 Cost Cost Custar
36 Broadcast Broadcast Transmitir
37 Create Created Criar
38 Cry Cried Chorar
39 Cut Cut Cortar
40 Damage Damaged Danificar, estragar
41 Dance Danced Dançar
42 Date Dated Sair para um encontro, namorar
43 Decide Decided Decidir
44 Deliver Delivered Entregar
45 Depend Depended Depender
46 Dive Dived, dove Mergulhar
47 Do Did Fazer, executar
48 Draw Drew Desenhar
49 Dream Dreamt, dreamed Sonhar
50 Drink Drank Beber
51 Drive Drove Dririgir (4 rodas)
52 Eat Ate Comer
53 End Ended Terminar
54 Enjoy Enjoyed Apreciar, desfrutar, gostar
55 Exercise Exercised Exercitar-se, fazer exercícios
56 Fall Fell Cair
57 Feed Fed Alimentar(se), alguém
58 Fight Fought Lutar
59 Find Found Encontrar
60 Finish Finished Terminar
61 Fish Fished Pescar
62 Fix Fixed Consertar, arrumar
63 Fly Flew Voar
64 Follow Followed Seguir
65 Forget Forgot Esquecer(se)
66 Fry Fried Fritar
67 Get Got Conseguir, ganhar
68 Get up Got up Levantar-se
69 Give Gave Dar, conceder
70 Go Went Ir

25
1103362 E-book gerado especialmente para MARCUS ICARO CRUZ DA SILVA
71 Grow Grew Crescer, cultivar
72 Guess Guessed Adivinhar, supor
73 Happen Happened Acontecer
74 Hate Hated Odiar
75 Have Had Ter, possuir
76 Hear Heard Ouvir
77 Help Helped Ajudar
78 Hide Hid Esconder, ocultar(se)
79 Hit Hit Bater
80 Hunt Hunted Caçar
81 Hurt Hurt Machucar
82 Improve Improved Melhorar, aperfeiçoar
83 Interview Interviewed Entrevistar
84 Jog Jog Caminhar (exercício físico)
85 Jump Jumped Pular, saltar
86 Keep Kept Guardar, manter, permanecer
87 Kiss Kissed Beijar
88 Know Knew Saber, conhecer
89 Listen Listened Escutar
90 Live Lived Viver, ao vivo
91 Look Looked Olhar, parecer
92 Lose Lost Perder
93 Love Loved Amar
94 Make Made Fazer, produzir, fabricar
95 Marry Married Casar
96 Meet Met Encontrar-se com
97 Miss Missed Sentir saudades, perder a hora
98 Move Moved Mexer, mudar-se
99 Need Needed Precisar, necessitar
100 Offer Offered Oferecer
101 Open Opened Abrir
102 Paint Painted Pintar
103 Park Parked Estacionar
104 Pay Paid Pagar
105 Plant Planted Plantar
106 Play Played Tocar instrumento, brincar
107 Practice Practiced Praticar, treinar
108 Prefer Prefered Preferir
109 Pull Pulled Puxar
110 Push Pushed Empurrar
111 Quit Quit Desistir, sair, abandonar
112 Rain Rained Chover
113 Read Read Ler
114 Relax Relaxed Relaxar, descansar
115 Remember Remembered Lembrar, recordar
116 Repair Repaired Reparar, consertar
117 Repeat Repeated Repetir
118 Rescue Rescued Resgatar, socorrer

26
1103362 E-book gerado especialmente para MARCUS ICARO CRUZ DA SILVA
119 Respond Responded Responder
120 Rest Rested Relaxar, descansar
121 Review Reviewd Revisar
122 Ride Rode Cavalgar (2 rodas)
123 Run Run Correr, administrar
124 Save Saved Salvar, economizar (dinheiro)
125 Say Said Dizer
126 Search Seached Pesquisar, procurer, buscar.
127 See Saw Ver
128 Sell Sold Vender
129 Send Sent Enviar
130 Sing Sang Cantar
131 Sink Sank Afundar, naufragar
132 Sit Sat Sentar
133 Skate Skated Patinar, andar de skate
134 Ski Skied Esquiar
135 Sleep Slept Dormir
136 Smell Smelt Cheirar
137 Snow Snowed Nevar
138 Speak Spoke Falar
139 Spell Spelled Soletrar
140 Spend Spent Gastar tempo ou dinheiro
141 Spill Spilled, spilt Derramar liquido
142 Start Started Iniciar, começar
143 Steal Stole Roubar
144 Stop Stopped Parar, deter
145 Study Studied Estudar
146 Suggest Suggested Sugerir
147 Swear Swore Jurar, falar palavrão
148 Sweat Sweat, sweated Suar
149 Sweep Swept Varrer
150 Swim Swam Nadar
151 Take Took Tomar, pegar
152 Talk Talked Falar
153 Teach Taught Ensinar
154 Tell Told Contar, dizer
155 Thank Thanked Agradecer
156 Think Thought Pensar, achar (opnião)
157 Throw Threw Jogar, arremessar.
158 Touch Touched Tocar
159 Travel Traveled Viajar
160 Try Tried Tentar
161 Turn Turn Girar, rodar, virar
162 Understand Understood Entender, compreender
163 Upset Upset Ficar nervoso, com raiva
164 Use Used Usar
165 Visit Visited Visitar
166 Wait Waited Esperar

27
1103362 E-book gerado especialmente para MARCUS ICARO CRUZ DA SILVA
167 Wake up Waked up, woke up Acordar
168 Walk Walked Caminhar, andar
169 Want Wanted Querer
170 Wash Washed Lavar
171 Watch Watched Assistir, vigiar
172 Water Watered Regar
173 Wear Wore Vestir
174 Welcome Welcomed Dar boas vindas
175 Win Won Ganhar, vencer
176 Wish Wished Desejar
177 Work Worked Trabalhar, funcionar
178 Worry Worried Preocupar-se
179 Write Wrote Escrever

TEMPOS VERBAIS

PRESENTE CONTÍNUO: indica algo que acontece no exato momento da fala. As frases neste tempo
verbal mostram o que alguém está fazendo (gerúndio). Necessita do verbo to be (am, is, are) e mais
algum outro verbo com terminação -ing (-ando, endo, -indo, -ondo):

Exemplos:

I am writing a book.
Eu estou escrevendo um livro.

You are reading.


Você está lendo.

He is listening to music.
Ele está escutando música.

She is making lunch.


Ela está fazendo o almoço.

28
1103362 E-book gerado especialmente para MARCUS ICARO CRUZ DA SILVA
It is playing with a ball.
Ele/Ela está brincando com uma bola.

We are learning together.


Nós estamos aprendendo juntos.

You are studying English.


Vocês estão estudando Inglês.

They are traveling.


Eles estão viajando.

*O pronome it é usado para coisas e animais. Pode referir-se a pessoas quando não se sabe o sexo.

Tudo o que foi descrito nestas frases está acontecendo agora, neste exato momento. Por isso
usamos o presente contínuo. Para tornar todas estas frases negativas, basta posicionar a palavra not
após o to be, ou fazer uma contração ente eles (am not, isn’t, aren’t).

Exemplos:

I am not writing a book. (O to be am negativo não possui forma contraida)

You aren’t reading.


Você não está lendo.

He isn’t listening to music.


Ele não está escutando música.

She isn’t making lunch.


Ela não está fazendo o almoço.

It isn’t playing with a ball.


Ele/Ela não está brincando com uma bola.

We aren’t learning together.


Nós não estamos aprendendo juntos.

You aren’t studying English.


Vocês não estão estudando Inglês.

They aren’t traveling.


Eles não estão viajando.

Agora, para transformarmos as frases em interrogações, devemos mudar a posição do to be.


Precisamos posicioná-lo (am, is, are) antes dos sujeitos das frases. As outras palavras permanecem em
suas posições originais. Claro que não podemos esquecer do ponto de interrogação. Veja:

Exemplos:

Am I writing a book?
Eu estou escrevendo um livro?

Are you reading?


Você está lendo?

Is he listening to music?
Ele está ouvindo música?

29
1103362 E-book gerado especialmente para MARCUS ICARO CRUZ DA SILVA
Is she making lunch?
Ela está fazendo o almoço?

Is It playing with a ball?


Ele/ela (animal) está brincando com a bola?

Are we learning together?


Nós estamos aprendendo juntos?

Are you studying English?


Você está estudando Inglês?

Are they traveling?


Eles estão viajando?

PASSADO CONTÍNUO: se você quiser colocar todas as frases que acabamos de estudar no
passado, para relatar o que alguém estava fazendo, é muito simples. Basta trocar verbo to be que
estava no presente pelo to be no passado (was, were). Apenas tenha atenção na hora de saber qual
pessoa usará was e qual usará were. Exemplos:

Exemplos:

I was writing a book.


Eu estava escrevendo um livro.

You were reading.


Você estava lendo.

He was listening to music.


Ele estava ouvindo musica.

She was making lunch.


Ela estava fazendo o almoço.

It was playing with a ball.


Ele/ela (animal) estava brincando com a bola.

We were learning together.


Nós estávamos aprendendo juntos.

You were studying English.


Você estava estudando Inglês.

They were traveling.


Eles estavam viajando.

Perceba que usamos was com I/He/She/It, e que usamos were com You/We/They. Agora, para
formar a negativa (wasn’t, weren’t) e a interrogativa (Was I...?, Were you...?), basta proceder da mesma
forma que vimos no caso do Presente Contínuo.

FUTURO CONTÍNUO: para relatar aquilo que alguém estará fazendo em um determinado momento
no futuro, é só utilizar will be e mais qualquer outro verbo terminado em -ing.

I will be writing a book tomorrow night.


Eu estarei escrevendo um livro amanhã a noite.

You will be reading when she arrives.


Você estará lendo quando ela chegar.

30
1103362 E-book gerado especialmente para MARCUS ICARO CRUZ DA SILVA
He will be listening to music this Saturday.
Ele estará ouvindo música este sábado.

She will be making lunch tomorrow at noon.


Ela estará fazendo o almoço amanhã ao meio dia.

It will be playing with a ball Monday.


Ele/ela (animal) estará brincando com a bola segunda-feira.

We will be learning together during the trip to Spain.


Nós estaremos aprendendo juntos durante a viagem para a Espanha.

You will be studying English next semester.


Você estará estudando Inglês durante o próximo semestre.

They will be traveling to Germany next summer.


Eles estarão viajando para a Alemanha no próximo verão (férias).

Nas negativas, simplesmente posicionamos not logo após o auxiliar will, ou fazemos uma contração
com eles (will+not= won’t).

Para interrogar, faz-se a colocação do auxiliar will antes do sujeito das frases (Will I...?, Will you...?).

PRESENTE SIMPLES: este tempo verbal nos fala de situações que acontecem rotineiramente.
Estas situações não acontecem no exato momento da fala, mas usualmente durante o dia a dia. Por
exemplo, você pode dizer em português “eu trabalho”. Essas suas palavras indicam algo rotineiro para
você, não querem dizer que você esteja trabalhando agora, neste exato momento. É essa noção de que
algo acontece no presente mas como uma rotina é o que o presente simples indica. Vamos ver a
conjugação de alguns verbos no presente simples com frases afirmativas primeiro:

I work in the evening.


Eu trabalho a noite (no período da noite).

You like to dance.


Você gosta de dançar.

He sleeps a lot.
Ele dorme muito.

She cooks well.


Ela cozinha bem.

It barks too much.


Ele/ela* late muito. (Lembrando que o pronome it é utilizado como ele/ela quando se refere a
animais ou objetos, neste caso um cachorro ou cadela).

We speak English fluently.


Nós falamos Inglês fluentemente.

You drive fast.


Você dirige rapidamente.

They drink beer.


Eles bebem cerveja.

Perceba que basta seguir a ordem “sujeito + verbo no infinitivo sem to (+complemento)” para formar
algumas sentenças. É a ordem natural das palavras em Português também. Assim, se você souber uma
boa gama de verbos, poderá montar muitas frases para praticar.

31
1103362 E-book gerado especialmente para MARCUS ICARO CRUZ DA SILVA
Neste caso de sentenças afirmativas somente necessitamos tomar cuidado com os detalhes em
negrito e em sublinhado. Todas as vezes em que o sujeito da frase for a terceira pessoa do singular
(he/she/it), devemos acrescentar um -s no final do verbo. Em algumas situações será um -es, e no caso
do verbo ter (to have) a forma será has. Repito: só nas afirmativas com 3ª pessoa singular.

As negativas precisam fazer o uso dos verbos auxiliares do e does, acrescidos de not (do+not=don’t /
does+not=doesn’t). Doesn’t será usado somente com 3ª pessoa singular. Exemplos:

I don’t work in the evening.


Eu não trabalho a noite (no período da noite).

You don’t like to dance.


Você não gosta de dançar.

He doesn’t sleep a lot.


Ele não dorme muito.

She doesn’t cook well.


Ela não cozinha bem.

It doesn’t bark too much.


Ele/ela* não late muito. (Lembrando que o pronome it é utilizado como ele/ela quando se refere a
animais ou objetos, neste caso um cachorro ou cadela).

We don’t speak English fluently.


Nós não falamos Inglês fluentemente.

You don’t drive fast.


Você não dirige rapidamente.

They don’t drink beer.


Eles não bebem cerveja.

Para fazermos perguntas, posicionaremos do e does antes do sujeito da frase e acrescentaremos o


ponto de interrogação.

Do I work in the evening?


Você trabalha a noite (no período da noite)?

Do you like to dance?


Você gosta de dançar?

Does he sleep a lot?


Ele dorme muito?

Does she cook well?


Ela cozinha bem?

Does it bark too much?


Ele/ela* late muito? (Lembrando que o pronome it é utilizado como ele/ela quando se refere a
animais ou objetos, neste caso um cachorro ou cadela).

Do we speak English fluently?


Nós falamos Inglês fluentemente?

Do you drive fast?


Você dirige rapidamente?

32
1103362 E-book gerado especialmente para MARCUS ICARO CRUZ DA SILVA
Do they drink beer?
Eles bebem cerveja?

Ótimo. Agora, para finalizarmos o presente simples, passemos ao principal verbo inglês: o to be. A
conjugação do presente do to be possui três formas: am, is e are. Este verbo significa duas coisas ao
mesmo tempo: ser e estar. Mas como identificar se numa frase ele quer se referir ao verbo ser ou se ao
verbo estar? Resposta: depende da frase, depende do contexto. Veja:

I am a teacher.
Eu sou um(a) professor(a).

You are a student.


Você é um(a) aluno(a).

He is late.
Ele está atrasado.

She is early.
Ela está adiantada.

It is tall.
Ele/Ela é alto(a).

We are Brazilians.
Nós somos brasileiros.

You are busy.


Você(s) está(ão) ocupado(s).

They are happy.


Eles/Elas estão/são felizes.

Note que am é usado na primeira pessoa do singular, is na terceira do singular e are nas outras.

Para negarmos, usamos not logo após o to be ou fazemos contração entre eles.

I am not a teacher.
Eu não sou um(a) professor(a).
You aren’t a student.
Você não é um(a) aluno(a).

He isn’t late.
Ele não está atrasado.

She isn’t early.


Ela não está adiantada.

It isn’t tall.
Ele/ela não é alto(a).

We aren’t Brazilians.
Nós não somos Brasileiros(as)

You aren’t busy.


Você não é(são)/não está(estão) ocupado(a)(s).

They aren’t happy.


Eles não estão/são feliz(es).

33
1103362 E-book gerado especialmente para MARCUS ICARO CRUZ DA SILVA
Finalizando, para transformarmos estas frases em interrogações, temos que por o to be antes dos
sujeitos. Lembrete: ponto de interrogação! Assim:

Am I a teacher?
Eu sou um(a) professor(a)?

Are you a student?


Você é um(a) aluno(a)?

Is he late?
Ele está atrasado?

Is she early?
Ela está adiantada?
Is it tall?
Ele/ela* é alto(a)?

Are we Brazilians?
Nós somos brasileiros?

Are you busy?


Você está ocupado?

Are they happy?


Eles estão felizes?

PASSADO SIMPLES: indica alguma ação completa no passado, ou seja, algo já finalizado. O
passado simples caracteriza-se pela adição da terminação -ed ao verbos REGULARES nas afirmativas.
Nas interrogativas, usamos Did antes dos sujeitos das frases e, nas negativas, did not ou didn’t.
Vejamos:

I worked yesterday.
Eu trabalhei ontem.

You answered my e-mail.


Você respondeu ao meu e-mail.

He traveled a lot.
Ele viajou muito.

She watched the movie.


Ela assitiu o filme.

It barked all night.


Ele/Ela* latiu a noite toda.

We stayed here.
Nós ficamos aqui.

You played very well.


Vocês jogaram muito bem.

They parked far.


Eles estacionaram longe.

I didn’t work yesterday.


Eu não trabalhei ontem.

34
1103362 E-book gerado especialmente para MARCUS ICARO CRUZ DA SILVA
You didn’t answer my e-mail.
Você não respondeu ao meu e-mail.

He didn’t travel a lot.


Ele não viajou muito.

She didn’t watch the movie.


Ela não assistiu o filme.

It didn’t bark all night.


Ele/Ela* não latiu a noite toda.

We didn’t stay here.


Nós não ficamos aqui.
You didn’t play very well.
Vocês não jogaram muito bem.

They didn’t park far.


Eles não estacionaram longe.

Did I work yesterday?


Eu trabalhei ontem?

Did you answer my e-mail?


Você respondeu ao meu e-mail?

Did he travel a lot?


Ele viajou muito?

Did she watch the movie?


Ela assistiu o filme?

Did it bark all night?


Ele/Ela* latiu a noite toda?

Did we stay here?


Nós ficamos aqui?

Did you play very well?


Vocês jogaram muito bem?

Did they park far?


Eles estacionaram longe?

Quanto aos verbos irregulares, procederemos da mesma forma. A única diferença é nas afirmações,
pois eles não recebem terminação -ed. É essencial memorizar as formas irregulares. Vejamos:

I went to the beach. (to go: ir)


Eu fui para a praia.

You left early. (to leave: sair, deixar)


Você saiu cedo.

He drank too much. (to drink: beber)


Ele bebeu demais.

She had a sister. (to have: ter)


Ela tinha uma irmã.

35
1103362 E-book gerado especialmente para MARCUS ICARO CRUZ DA SILVA
It slept under the bed. (to sleep: dormir)
Ele/Ela* dormiu embaixo da cama.

We ate pizza last night. (to eat: comer)


Nós comemos pizza ontem a noite.

You won together. (to win: vencer, ganhar)


Vocês venceram juntos.

They cut the meat. (to cut: cortar)


Eles cortaram a carne.

FUTURO SIMPLES: Usamos o futuro simples para dizer que algo vai acontecer ou deverá
acontecer, para expressar ações que iremos fazer mas que não tínhamos planejado anteriormente, para
fazer previsões sobre o futuro, uma vez que não temos certeza se essa previsão irá mesmo se
concretizar ou não. Usamos também o futuro simples para promessas, ofertas e propostas. A estrutura
é formado pela utilização do auxiliar will após o sujeito seguido de algum verbo. A negativa é obtida
com will not ou com a contração won’t. Para perguntar no futuro simples, é só colocar will antes do
sujeito. Exemplos:

I will buy a car.


Eu vou comprar um carro.

You will have a baby.


Você vai ter um bebê.

He will study abroad.


Ele irá estudar no exterior.

She will go to the park.


Ela irá para o parque.

It will stay at the veterinarian.


Ele/ela* irá permanecer no veterinário.

We will make a barbecue.


Nós iremos fazer um churrasco.

You will help me later.


Você irá me ajudar depois.

They will be partners.


Eles serão parceiros.

FUTURO IMEDIATO: Utilizamos o futuro imediato para expressar algo que já foi planejado e por isso
existe a certeza de que irá acontecer. Por ser algo que temos certeza que iremos fazer o futuro imediato
acaba sendo usado frequentemente para expressar ações que acontecerão num futuro bem próximo,
por isso chamado de imediato. A estrutura do futuro imediato é o sujeito + o verbo to be no presente
(am, is, are) + going to + verbo principal + complemento.

I’m going to visit my mother tonight.


Eu irei visitar minha mãe hoje a noite.

Jack is going to swim tomorrow.


Jack irá nadar amanhã.

It is going to rain in a few minutes.


Irá chover em alguns minutos.

36
1103362 E-book gerado especialmente para MARCUS ICARO CRUZ DA SILVA
Como o futuro imediato é composto do to be, para fazermos frases interrogativas e negativas, basta
utilizar as mesmas regras acrescentando not após o to be, ou colocando o mesmo antes do sujeito
para a interrogativa.

Steve is not going to dance samba.


Steve não irá dançar samba.

They aren’t going to play soccer.


Eles não irão jogar futebol.

Is he going to buy a new car?


Ele vai comprar um carro novo?

Are you going to call Ann?


Você irá ligar pra Ann?

Apenas em conversas e diálogos informais o going to pode ser substituído pela


expressão/abreviação gonna:

I’m gonna study tonight.


Eu irei estudar hoje a noite.

Are you gonna help me?


Você irá me ajudar?

PRESENTE PERFEITO: formado pela utilização do auxiliar have ou has (has para he, she, it) mais
a forma do particípio de outro verbo (conhecida como “a terceira forma do verbo”). Indica quando
descrevemos situações que já ocorreram, mas que não sabemos quando. O tempo é indefinido, não
interessa, ou simplesmente não importa, pois o que importa é o fato acontecido.

Mike has seen the ocean many times.


Mike viu o oceano muitas vezes.

Sheila and Susan have already been to New York.


Sheila e Susan já estiveram em Nova Iorque.

I have already made my bed.


Eu já arrumei minha cama.

As formas negativas serão:

I haven’t made my bed.


Eu não arrumei minha cama.

Mike hasn’t seen the ocean.


Mike não viu o oceano.

Sheila and Susan haven’t been to New York.


Sheila e Susan não estiveram em Nova Iorque.

Se quisermos, podemos acrescentar no final da frase a palavra yet, que significa tal ação “ainda” não
aconteceu. (apenas nas negativas)

I haven’t made my bed yet.


Eu ainda não arrumei minha cama.

Mike hasn’t seen the ocean yet.


Mike ainda não viu o oceano.

37
1103362 E-book gerado especialmente para MARCUS ICARO CRUZ DA SILVA
Sheila and Susan haven’t been to New York yet.
Sheila e Susan ainda não estiveram em Nova Iorque.

Para fazermos perguntas no present perfect, basta colocar have ou has antes do sujeito da frase. Às
vezes, fazemos uso da palavra ever, que significa “alguma vez”, em perguntas: (o uso da palavra ever
é opcional)

Have you bought Milk for the baby?


Você comprou leite para o bebê?

Has he talked to the police officer?


Ele falou com o policial?

Has Tina ever traveled to Salvador?


A Tina viajou para Salvador alguma vez?

Have you ever seen a famous person?


Você alguma vez viu uma pessoa famosa?

PRESENTE PERFEITO CONTÍNUO: formado pela utilização do auxiliar have ou has (has para he,
she, it) mais o presente perfeito do verbo be e o gerúndio do verbo principal. Esta forma verbal enfatiza
uma ação que começou no passado e que contina se repetindo até hoje.

I have been playing tennis for one hour.


Eu estou jogando tennis há uma hora.

Daniel has been waiting for two hours.


Daniel está esperando a duas horas.

Anna has been teaching in the university since April.


Anna tem lecionado na universidade desde Abril.

As formas negativas:

She has not been working at that company for three years.
Ela não tem trabalhado naquela companhia a três anos.

I haven’t been watching much television lately.


Eu não tenho assistido muita televisão ultimamente.

Roberto hasn’t been feeling well in the past few days.


Roberto não tem se sentido bem nos últimos dias.

Para fazermos perguntas no present perfect continuos, basta colocar have ou has antes do sujeito
da frase.

Has David been doing his homework everyday?


David está fazendo sua tarefa todos os dias?

Have Donald and Mike been training for the race?


Donald e Mike estão treinando para aquela corrida?

Have you been playing video games all day?


Você está jogando video games o dia inteiro?

PASSADO PERFEITO: usado para dizer que alguma coisa ocorreu antes de outra no passado.
Formado por had mais o particípio de algum verbo. Veja no próximo exemplo que há duas situações
acontecendo, mas, aquela que aconteceu primeiro está usando o past perfect. E aquela que aconteceu
em seguida está no passado simples. Ambas as orações estão unidas por when.

38
1103362 E-book gerado especialmente para MARCUS ICARO CRUZ DA SILVA
I had already left when my father called home.
Eu já tinha saído quando meu pai ligou para casa.

She had called a taxi when I told her I would pick her up.
Ela já tinha chamado um taxi quando eu disse a ela que a pegaria.

O passado perfeito não precisa acontecer obrigatoriamente com as duas situações em uma mesma
oração.

David had bought meat for the barbecue this morning.


David tinha comprado carne para o churrasco hoje de manhã.

A negativa é formada com had not ou hadn’t. Para perguntar, devemos posicionar o had antes do
sujeito.

He hadn’t gone to the bar.


Ele não tinha ido ao bar.

Had you brought me those documents?


Você tinha me trazido aqueles documentos?

VERBOS AUXILIARES

Em perguntas você pode mudar o tempo verbal de uma frase simplesmente alterando o verbo
auxiliar. Por exemplo:

Do you work?
Você trabalha?

Does He work?
Ele trabalha?

Did you work?


Você trabalhou?

Will you work?


Você vai trabalhar?

Os verbos auxiliares não possuem tradução nas frases:

Do you play volleyball?


Voce joga volei?

A presença de um verbo auxiliar numa frase nos indica em que tempo verbal ela está (no presente,
no passado ou no futuro), dependendo do auxiliar que foi usado. Do e does indicam tempo presente,
did indica tempo passado, e will indica tempo futuro.

Suas formas negativas são don’t (do not), didn’t (did not) e won’t (will not).

Para montarmos interrogações, basta posicionar o auxiliary desejado antes do sujeito da frase.

O auxiliar também pode facilitar as coisas nas respostas. Ele pode substituir o verbo e todos os seus
complementos. Assim, se alguém faz um pergunta muito longa, você pode responder rapidamente:

Do you always go to work by car on weekdays?


Você sempre vai para o trabalho de carro nos dias da semana?

Sua resposta pode ser, simplesmente, “Yes, I do”.

39
1103362 E-book gerado especialmente para MARCUS ICARO CRUZ DA SILVA
Estas respostas curtas são conhecidas como short answers.

Os verbos auxiliares seguidos de um verbo principal são usados praticamente só em perguntas ou


frases negativas:

Do you like pizza?


Você gosta de pizza?

I don't like pizza


Eu não gosto de pizza.

Numa frase afirmativa diríamos:

I like pizza.
Eu gosto de pizza.

As formas does e doesn’t são usadas quando o sujeito da frase no presente for terceira pessoa do
singular (he, she, it).

I don’t eat pizza.


Eu não como pizza.

You don’t eat pizza.


Você não come pizza.

She doesn’t eat pizza.


Ela não come pizza.

He doesn’t eat pizza.


Ele não come pizza.

It doesn’t eat pizza.


Ela/Ele* não come pizza.

We don’t eat pizza.


Nós não comemos pizza.

You don’t eat pizza.


Vocês não comem pizza.

They don’t eat pizza.


Eles não comem pizza.

Do I eat pizza?
Eu como pizza?

Do you eat pizza?


Você come pizza?

Does she eat pizza?


Ela come pizza?

Does he eat pizza?


Ele come pizza?

Does it eat pizza?


Ela/Ele* come pizza?

Do we eat pizza?

40
1103362 E-book gerado especialmente para MARCUS ICARO CRUZ DA SILVA
Nós comemos pizza?

Do you eat pizza?


Vocês comem pizza?

Do they eat pizza?


Eles comem pizza?

VERBOS MODAIS

Os verbos modais são distintos dos regulares e irregulares pois possuem características próprias:

1. Não precisam de auxiliares na formação de negativas e interrogativas;


2. Sempre após os modais, usamos um verbo regular ou irregular no infinitivo, mas sem o “to”;
3. Não sofrem alteração na terceira pessoa do singular do presente. Logo, nunca recebem “s”, “es”
ou “ies” para he/she/it.

São verbos modais: can, could, may, might, should, must, ought to.

May - pode ser usado para pedir permissão:

Exemplos:
May I open the window?
Posso abrir a janela?

May I use your bathroom?


Posso usar seu banheiro?

May e Might - podem indicar possibilidade mais certa ou probabilidade mais remota:

Exemplos:
It may rain.
Pode chover.

May indica algo com mais certeza do que might.

Exemplos:
It might rain.
Pode chover. (a probabilidade de chover é pequena.)

He might come to the party, but I don't think he will.


Ele pode vir à festa, mas não creio que virá.

May e might podem ser usados para exprimir um propósito, uma aspiração ou uma esperança:

Exemplos:
May he rest in peace.
Que ele repouse em paz.

I hope that he might like this cake.


Espero que ele possa gostar deste bolo.

May all your dreams come true.


Que todos os seus sonhos se realizem.

Para dizermos algo no passado e no futuro, ao invés de may e might, normalmente usamos os
verbos “to be allowed to” ou “to be permitted to”, que significam “ser permitido”.

Exemplos:

41
1103362 E-book gerado especialmente para MARCUS ICARO CRUZ DA SILVA
He will be allowed to leave prison.
Ser-lhe-á permitido sair da prisão.

I wasn’t allowed to enter without a uniform.


Não me deixaram entrar sem um uniforme.

Must - precisar, dever, ter que.

Must é usado no presente e no futuro. Must pode exprimir ordem, necessidade, obrigação, dever. É
equivalente a have to (ter que):

Exemplos:

I must go now.
Preciso ir agora.

You must obey your parents.


Você deve obedecer a seus pais.

You must follow your doctor's advice.


Você tem que seguir os conselhos do seu médico.

He has worked a lot; he must be tired.


Ele trabalhou muito; ele deve estar cansado.

A forma negativa mustn't (must not) exprime uma proibição ou faz uma advertência.

Exemplos:
Visitors must not feed the animals.
Visitantes estão proibidos de alimentar os animais.

You mustn't miss the 9:00 train.


Você não pode perder o trem das 9:00.

Can – poder, conseguir.

Can pode ser usado para expressar talentos e habilidades no presente.

Exempos:
They can sing really well.
Eles consigo cantar realmente muito bem.

I can speak English.


Eu consigo falar Inglês.

Há duas formas negativas, can’t e cannot.

Exemplos:
He can’t dance at all.
Ele não consegue dançar nada.

Tim cannot control his feelings.


Tim não consegue controlar seus sentimentos.

Could - conseguia, podia, poderia.

Usamos could para expressar ideias como sendo o passado de can.

Exemplos:

42
1103362 E-book gerado especialmente para MARCUS ICARO CRUZ DA SILVA
When I was a teenager I could swim better.
Quando eu era adolescente eu podia nadar melhor.

I could run, now I can’t anymore.


Eu podia correr, mas agora não consigo mais.

Usamos could para pedir permissão. Could é mais educado e formal que can:

Exemplos:
Could you help me?
Você poderia me ajudar?

Could I borrow your cell phone?


Eu poderia pegar emprestado seu celular?

Should e Ought to – deveria.

Usamos should e ought to para expressar nossa opinião, para dar sugestão ou conselho:

Exemplos:
He should travel more.
Ele deveria viajar mais.

I ought to go right now.


Eu deveria ir imediatamente.

As formas negativas são shouldn’t e ought not to.

Exemplos:
You shouldn’t talk like that.
Você não deveria falar desse jeito.

I ought not to see her.


Eu não deveria vê-la.

ORDEM DAS PALAVRAS NA ORAÇÃO

Para começar, vejamos a ordem das palavras em sentenças afirmativas:

sujeito verbo objeto


I study English.
I play tennis.

Se você já é um estudante um pouco mais avançado, lembre-se da seguinte regra:

Sujeito verbo objeto indireto objeto direto lugar tempo


I will tell you the story at school tomorrow.

A ordem das palavras em sentenças negativas é a mesma que nas afirmativas. Note, entretanto, que
nas negativas nós normalmente precisamos de um verbo auxiliar:

Sujeito verbo objeto indireto objeto direto lugar tempo


I will not tell you the story at school tomorrow.

Posição de Expressões de Tempo (recently, now, then, yesterday, etc.)

43
1103362 E-book gerado especialmente para MARCUS ICARO CRUZ DA SILVA
Advérbios de tempo são normalmente postos no final da sentença.

Sujeito verbo objeto indireto objeto direto tempo


I will tell you the story tomorrow.

Posição de Advérbio de modo (slowly, carefully, awfully, etc.)

Estes são postos atrás do objeto direto, ou atrás do verbo se não houver objeto direto.

Sujeito verbo objeto direto advérbio


He drove the car carefully.
He drove carefully.

Advérbios de lugar (here, there, behind, above, etc.)

Assim como os advérbios de modo, estes são colocados atrás do objeto direto ou do verbo.

Sujeito verbo objeto direto advérbio


I didn’t see him there.
He stayed behind.

Advérbios de tempo (recently, now, then, yesterday, etc.)

Advérbios de tempo são normalmente colocados no final da sentença.

Sujeito verbo objeto indireto objeto direto tempo


I will tell you the story tomorrow.

Se você não quiser impor ênfase no tempo da ação, você pode por o advérbio de tempo no início da
sentença.

Tempo sujeito verbo objeto indireto objeto direto


Tomorrow I will tell you the story.

Advérbios de Frequência (always, never, seldom, usually, etc.)

São posicionados diretamente antes do verbo principal. Se o “to be” for o verbo principal e não
houver nenhum verbo auxiliar, os advérbios de frequência devem ser postos atrás do “to be”. Se houver
um verbo auxiliar, entretanto, advérbios de frequência são posicionados antes do “to be”.

Sujeito auxiliar/to be advérbio verbo principal objeto, lugar ou tempo


We are usually here in the summer.

Ordem de palavras em sentenças interrogativas:

Em perguntas, a ordem sujeito-verbo-objeto é a mesma que nas sentenças afirmativas. A única coisa
que pode se alterar é que você normalmente tem que interpor o verbo auxiliar antes do sujeito.
Pronomes Interrogativos são colocados no início das sentenças:

Interrog. verbo aux. Suj. outro verbo obj. indireto obj. direto lugar tempo
What would you like to tell me?
Did you have a party at home yesterday?
When were you there?

44
1103362 E-book gerado especialmente para MARCUS ICARO CRUZ DA SILVA
Não podemos usar um verbo auxiliar quando procuramos ou perguntamos pelo sujeito da oração.
Neste caso, o pronome interrogativo simplesmente toma o lugar do sujeito.

Interrogativo verbo(s) objeto


Who invited you?

VOCABULÁRIO DE INFORMÁTICA E TECNOLOGIA

Iremos listar aqui as palavras mais utilizadas nas área de informática, tecnologia e gestão.
A função desta lista é ajudar o estudante a entender os textos propostos nos concursos da
área. Quanto mais palavras em uma determinada frase o candidato conseguir identificar, mais
facilidade ele terá para entender tal frase e consequentemente ligar ela ao contexto geral do
texto.

Computing professionals - profissionais da área da computação.


Information Technology - Tecnologia da Informação. (sigla IT)
IT Department - Departamento de TI. (sigla acima)
IT Professional - Profissional em TI. (sigla acima)
Software Development - Desenvolvimento de Softwares.
Systems Integration - Integração de Sistemas.
Data Management - Gerenciamento de Dados.
Computer Networks - Redes de Computadores.
Wireless Networking - Redes de Tecnologia Sem Fio.
Information Security - Segurança da Informação.
Technical Support – SuporteTécnico.
Computer Engineering - Engenharia da Computação.
Computer Science - Ciência da Computação.
Software Engineering - Engenharia de Softwares.
Digital Signature - Assinatura Digital.

VOCABULÁRIO CORPORATIVO

B2B – Business to Business - É o comércio entre empresas sem a participação do


consumidor.
B2C – Business to Consumer – É a venda direta da empresa para o consumidor.
Benchmark - Parâmetros de excelência, exemplos de coisas boas.
Board - Conselho diretor.
Brainstorm - literalmente, significa "tempestade cerebral". É uma reunião para se fazer
exatamente isso: trocar ideias, geralmente para se criar algo novo.
Breakthrough - trata-se de um avanço em determinada área, quando um desafio é
vencido.
Break even point - o momento a partir do qual custos e receitas de um negócio se
equilibram.
Broad band - banda larga.
Budget - orçamento.
Cash - dinheiro vivo.
CEO - chief executive officer - É o cargo mais alto da empresa. É chamado também de
presidente, principal executivo, diretor geral, entre outros.
CFO - chief financial officer - Um nome mais sofisticado para diretor de finanças.
Chairman - presidente do conselho que dirige a empresa.
CHRO - chief human resources officer - É o cargo de diretor de recursos humanos.
CIO - chief information officer - Responsável pelo planejamento e estratégia por trás da
tecnologia.

45
1103362 E-book gerado especialmente para MARCUS ICARO CRUZ DA SILVA
Commodity - produto primário, geralmente com grande participação no comércio
internacional.
COO - chief operating officer - executivo chefe de operações. Geralmente o braço direito
dos CEO´s
Core business - negócio principal da empresa.
Corporate purpose - objetivo da empresa.
Downsizing - redução no número de funcionários da empresa.
Factoring - prática de algumas empresas que consiste em comprar cheques pré- datados
de lojistas cobrando comissão.
Follow-up - dar prosseguimento a uma discussão ou debate.
Forecast - previsão.
Headhunter - caça-talentos do mundo corporativo.
Income - renda.
Intrapreneur (não confundir com entrepreneur) - empreendedor interno, pessoa que dirige
uma unidade do negócio como se ela fosse uma empresa independente.
Market share - fatia de mercado.
Markup - é um sobre-preço que se acrescentado ao preço final do produto.
MBA: master of business administration - pós-graduação em administração de empresas.
Sales manager - gerente de vendas.
Target - alvo.
Trend - tendência.

Questões

Leia o texto abaixo e responda as perguntas.

The Hunger Games: Book Review

Brendan Dunne
Fonte: http://goo.gl/UN3XpB

Introduction

The Hunger Games is a book by the American author Suzanne Collins. It was published in 2008 and
has sold millions of copies. It is the first book for young readers to sell a million electronic books and you
can buy it in 26 different languages. The Hunger Games is now a very successful film. It made 152.5
million dollars in its first weekend in North America!

A Fight to the Death

The story is set in the future, after the destruction of North America. The country is called Panem.
There are twelve poor districts governed by the rich Capitol. There was a thirteenth district in the past but
the Capitol destroyed them because they rebelled.
The Capitol organises the Hunger Games every year to punish the districts. One boy and one girl
aged 12 to 18 fight in a battle. Only one person will live. The chosen teenagers are called “tributes”. The
whole country must watch the games on television.
The story is about Katniss Everdeen, a sixteen-year-old girl. Her father died and now she has to kill
animals for her family to eat. Her younger sister, Prim, is chosen to be a “tribute”, but Katniss volunteers
to go instead. The other “tribute” from District 12 is a boy named Peeta.

Where did the idea for The Hunger Games come from?

One night in 2003, Suzanne Collins was watching TV. It was at the time of the US invasion of Iraq.
The only programmes she could find on TV were ‘reality’ programmes of young people competing to win
a million dollars and news programmes about the war. Suzanne says that the two things started to mix
together in her head and she had the idea for The Hunger Games.

46
1103362 E-book gerado especialmente para MARCUS ICARO CRUZ DA SILVA
She has always found news programmes of wars upsetting. When she was a child, her father was a
pilot in the US airforce and he fought in Vietnam. It was a very frightening experience for her.

Too much violence?

Some parents in the US have complained about the violence in the book. But Suzanne says she was
very worried about how much violence we see on TV nowadays. Suzanne is also worried about the
amount of reality TV we watch. “We put too much of our lives on TV,” she says. “And we care less for
people because of this.” She said that writing about death and violence in the story was the hardest thing
for her to do and she hopes it will make people think about what they watch in future.

1. Quantas milhões de cópias o livro eletrônico vendeu?


A) 2008 milhões.
B) 1 milhão
C) 26 milhões.
D) 152.5 milhões.

2. Qual a relação entre o filme “The Hunger Games” e o número 152.5 milhões de acordo com o
texto?
A) O custo do filme.
B) O lucro do filme.
C) O investimento do filme.
D) O preço na América do Norte.

3. De acordo com o texto o que é “Panem”?


A) O nome do personagem principal.
B) O nome do vilão do filme.
C) O apelido de Katniss Everdeen.
D) O nome do pais onde o filme acontece.

4. Por que a capital organiza “The Hunger Games”?


A) Para premiar os distritos.
B) Para criar aliança entre os distritos.
C) Para punir os distritos.
D) Para conhecer melhor os distritos.

5. Quantos anos tem Katniss?


A) Quatroze anos.
B) Quinze anos.
C) Dezesseis anos.
D) Dezessete anos.

6. Quem é “Prim”?
A) O Pai de Katniss.
B) A irmã de Katniss.
C) Um voluntário do distrito.
D) O tribudo do distrito 12.

7. Quem é Suzanne Collins?


A) Autora do livro.
B) Diretora do filme.
C) Apresentadora de TV.
D) Autora do texto.

8. De onde veio a idéia para o livro?


A) De um programa de TV.
B) De uma invasão ao Iraque.
C) De um programa de notícias.
D) Da mistura de um reality show e um documentário sobre a guerra.

47
1103362 E-book gerado especialmente para MARCUS ICARO CRUZ DA SILVA
9. Quem lutou na guerra do Vietnam?
A) Sua mãe.
B) Seu pai.
C) Seu irmão.
D) Ela mesma.

10. Qual a preocupação dos pais americanos?


A) Com a violência do filme.
B) Com a violência na TV.
C) Com a violência no livro.
D) Com a violência nos reality shows.

11- A Tall Order

The sky isn’t the limit for an architect building the world’s first invisible skyscraper.

Charles Wee, one of the world's leading high-rise architects, has a confession to make: he's bored with
skyscrapers. After designing more than 30, most of which punctuate the skylines of rapidly expanding Asian cities,
he has struck upon a novel concept: the first invisible skyscraper.
As the tallest structure in South Korea, his Infinity Tower will loom over Seoul until somebody pushes a button
and it completely disappears.
When he entered a 2004 competition to design a landmark tower, the Korean-American architect rejected the
notion of competing with Dubai, Toronto, and Shanghai to reach the summit of man-made summits. “I thought, let's
not jump into this stupid race to build another ‘tallest’ tower,” he says in a phone conversation. “Let's take an
opposite approach — let's make an anti-tower.”
The result will be a 150-story building that fades from view at the flick of a switch. The tower will effectively
function as an enormous television screen, being able to project an exact replica of whatever is happening behind it
onto its façade. To the human eye, the building will appear to have melted away.
It will be the most extraordinary achievement of Wee's stellar architectural career. After graduating from UCLA,
he worked under Anthony Lumsden, a prolific Californian architect who helped devise the modern technique of
wrapping buildings inside smooth glass skins.

HINES, N. Disponível em: http://mag.newsweek.com. Acesso em: 13 out. 2013 (adaptado).

No título e no subtítulo desse texto, as expressões A Tall Order e The sky isn’t the limit são usadas para
apresentar uma matéria cujo tema é:
A) Inovações tecnológicas usadas para a construção de um novo arranha-céu em Seul.
B) Confissões de um arquiteto que busca se destacar na construção de arranha-céus.
C) Técnicas a serem estabelecidas para a construção de edifícios altos na Califórnia.
D) Competição entre arquitetos para a construção do edifício mais alto do mundo
E) Construção de altas torres de apartamentos nas grandes metrópoles da Ásia.

12- Masters of War

Come you masters of war


You that build all the guns
You that build the death planes
You that build all the bombs
You that hide behind walls
You that hide behind desks
I just want you to know
I can see through your masks.

You that never done nothin’


But build to destroy
You play with my world
Like it's your little toy
You put a gun in my hand
And you hide from my eyes
And you turn and run farther
When the fast bullets fly.

48
1103362 E-book gerado especialmente para MARCUS ICARO CRUZ DA SILVA
Like Judas of old
You lie and deceive
A world war can be won
You want me to believe
But I see through your eyes
And I see through your brain
Like I see through the water
That runs down my drain.

BOB DYLAN. The Freewheelin’ Bob Dylan. Nova York: Columbia Records, 1963 (fragmento)

Na letra da canção Masters of War, há questionamentos e reflexões que aparecem na forma de protesto contra
A) o envio de jovens à guerra para promover a expansão territorial dos Estados Unidos.
B) o comportamento dos soldados norte-americanos nas guerras de que participaram.
C) o sistema que recruta soldados para guerras motivadas por interesses econômicos.
D) o desinteresse do governo pelas famílias dos soldados mortos em campos de batalha.
E) as Forças Armadas norte-americanas, que enviavam homens despreparados para as guerras.

13- The Road Not Taken (by Robert Frost)

Two roads diverged in a wood, and I — I took the one less traveled by, And that has made all the difference.

Disponível em: www.poetryfoundation.org. Acesso em: 29 nov. 2011 (fragmento).

Estes são os versos finais do famoso poema The Road Not Taken, do poeta americano Robert Frost. Levando-
se em consideração que a vida é comumente metaforizada como uma viagem, esses versos indicam que o autor
A) festeja o fato de ter sido ousado na escolha que fez em sua vida.
B) lamenta por ter sido um viajante que encontrou muitas bifurcações.
C) viaja muito pouco e que essa escolha fez toda a diferença em sua vida.
D) reconhece que as dificuldades em sua vida foram todas superadas.
E) percorre várias estradas durante as diferentes fases de sua vida.

14-

49
1103362 E-book gerado especialmente para MARCUS ICARO CRUZ DA SILVA
A internet tem servido a diferentes interesses, ampliando, muitas vezes, o contato entre pessoas e instituições.
Um exemplo disso é o site WeFeedback, no qual a internauta Kate Watts
A) comprou comida em promoção.
B) inscreveu-se em concurso.
C) fez doação para caridade.
D) participou de pesquisa de opinião.
E) voluntariou-se para trabalho social.

15- If You Can’t Master English, Try Globish

PARIS — It happens all the time: during an airport delay the man to the left, a Korean perhaps, starts talking to
the man opposite, who might be Colombian, and soon they are chatting away in what seems to be English. But the
native English speaker sitting between them cannot understand a word.
They don't know it, but the Korean and the Colombian are speaking Globish, the latest addition to the 6,800
languages that are said to be spoken across the world. Not that its inventor, Jean-Paul Nerrière, considers it a
proper language.
“It is not a language, it is a tool,” he says. “A language is the vehicle of a culture. Globish doesn't want to be that
at all. It is a means of communication.”
Nerrière doesn't see Globish in the same light as utopian efforts such as Kosmos, Volapuk, Novial or staunch
Esperanto. Nor should it be confused with barbaric Algol (for Algorithmic language). It is a sort of English lite: a
means of simplifying the language and giving it rules so it can be understood by all.

BLUME, M. Disponível em: www.nytimes.com. Acesso em: 28 out. 2013 (fragmento).

Considerando as ideias apresentadas no texto, o Globish (Global English) é uma variedade da língua inglesa
que
A) tem status de língua por refletir uma cultura global.
B) facilita o entendimento entre o falante nativo e o não nativo.
C) tem as mesmas características de projetos utópicos como o esperanto.
D) altera a estrutura do idioma para possibilitar a comunicação internacional.
E) apresenta padrões de fala idênticos aos da variedade usada pelos falantes nativos.

Respostas:

1 B
2 B
3 D
4 C
5 C
6 B
7 A
8 D
9 B
10 C

Questão 11:
A pergunda da questão se baseia no título do texto que é uma expressão. “Tall Order” tem significa um pedido
que é difícil de ser atendido, uma tarefa de que é difícil de ser cumprida. Na expressão “tall” tem o sentido
figurado, mas neste caso ela também tem o sentido literal por estarmos no contexto de arranha céus. O subtítulo
afirma que o céu não é o limite para um arquiteto construbindo o primeiro arranha céu invisível do mundo. Por aqui
já conseguimos chegar na resposta da questão. Para garantir a resposta correta, no segundo parágrafo o texto
afirma que a mais alta estrutura na Coreia do Sul, a sua torre infinita se destacará sobre Seoul.

Resposta correta: A

Questão 12:
Esta questão é um exemplo do que falamos no começo do material, ela requer mais um conhecimento prévio
sobre o assunto do que uma interpretação do que a música apresenta. Com os conhecimentos prévios nós
podemos elimitar duas alternativas neste contexto das forças armadas americanas e suas guerras. A Alternativa
“A” está incorreta pois nenhuma nação está fazendo expansão territorial. A alternativa “E” está incorreta pois todos
sabendos que as Forças Armadas norte-americanas são as mais preparadas e bem equipadas do mundo.

50
1103362 E-book gerado especialmente para MARCUS ICARO CRUZ DA SILVA
A música na primeira parte faz a referência a você que constroe todas as armas, todos os aviões da morte, que
constroe todas as bombas. Está é uma referencia a indústria da guerra, e que depois essa pessoa se esconde
atrás das paredes, se esconde atrás das mesas.
A segunda parte fala da destruição, que você brinca com o mundo como se fosse um pequeno brinquedo. Não
existem referências ao comportamento dos soldados ou sobre as famílias dos soldados.

Alternativa correta: C

Questão 13:
Esta é novamente mais uma questão que refer mais um conhecimento prévio de vida e filosofia do que de
interpretação literal do texto. A pergunta afirma que a frase em destaque vem de um famoso poema e que
devemos determinar qual é a linha de pensamento do autor em relação a ela.
“Two roads diverged in a wood, and I – I took the one less traveled by, And that has made all the difference.”

Duas estradas se divergem em uma floresta, e eu – eu peguei aquela que foi menos viajada, e isto fez toda a
diferença.

Como afirmado na questão, a vida é comumente metaforizada como uma viagem e nesta viagem nós tenhos
diferentes caminhos, diferentes escolhas. Neste caso o autor escolhe a menos popular e implica que no final o
resultado foi melhor do que se ele tivesse escolhido a popular.

A única alternativa que reflete isto é a alternativa “A”. Em relação as outras alternativas, a frase não afirma
nada sobre o viajante ter encontrado muitas bifuracações. Existem apenas duas. (Alternativa B). A frase não trás
informações sobre a quanto tempo o autor está viajando, se é muito ou se é pouco (Alternativa C). A alternativa D
poderia estar correta mas não trás informação alguma sobre a escolha que o autor realizou que é o foco da frase.
A Alternativa E está incorreta pois a frase não menciona a quanto tempo, fases ou estradas o autor já percorreu.

Resposta correta: A

Questão 14:
Esta questão é bem simples de ser respondida se o canditator for familiar com o verbo “feed” que quer dizer
alimentar e o verbo “share” que quer dizer compartilhar. Assim a expressão, que é o nome do site, “wefeedback”
se torna nós alimentando de volta. A primeira parte do texto explica qual o propósito do site, p orém a parte
importante é a última linha “we can help change the lives of hungry school children around the world”. Nós
podemos ajudar a mudar as vidas de crianças escolares famintas ao redor do mundo.

Na caixa abaixo a Kate Watts quer compartilhar um prato de sushi que geralmente ela paga 20 dólares. Ela
escolhe três pratos de sushi que somam o total de 60 dólares. Com os 60 dólares a caneta aponta que a
instituição será capaz de alimentar 240 crianças com este dinheiro. Apenas uma alternativa reflete a d oação de
dinheiro para os mais carentes.

Alternativa correta: C

Questão 15:
Esta questão requer um pouco de atenção para que o candidato possa eliminar as alternativas incorretas até
chegamos ao resultado desejado. O título do texto “Se você não consegue dominar o Inglês, tente Globish”. A
pergunta da questão explica o que é Globish, Global English, ou Inglês global.
Na primeira parte o texto trás uma situação. A esqueda um Coreano e a direita um Colombiano. No meio um
Inglês que não entende uma palavra entre os dois, pois eles estão falando Globish. A primeira frase afirma “Não é
uma língua, é uma ferramenta”. Aqui já imaginamos o que é o Globish, o nosso “Portunhol” quando tentamos nos
comunicar em Espanhol, mas para a versão Inglesa.
Vamos as alternativas. A alternativa A está incorreta. Ela não tem a função de refletir a cultura, ela é apenas
uma ferramenta (pobre diga-se de passagem) para a comunicação entre pessoas que não dominam a língua
inglesa. A alternativa B está incorreta pois já no começo o texto afirma que o nativo não é capaz de entender o
que os não nativos estão falando. A alternativa C está incorreta. Na última parte o texto afirma que “Nerrière
doesn’t see Globish in the same light as utopian efforts such ....”, ou seja, Nerrière não vê o Globish da mesma
forma que as demais línguas. A alternativa E está incorreta exatamente pelo mesmo motivo que a letra B está
incorreta. Somente os não nativos entendem a bagunça e improviso que eles mesmos criam.

Alternativa correta: D

51
1103362 E-book gerado especialmente para MARCUS ICARO CRUZ DA SILVA
Espanhol
1. Estrutura dos sintagmas nominal, adjetival e adverbial.
2. O verbo e o sintagma verbal: tempo, aspecto e modo; voz; modais.
3. A estrutura de frases simples e complexa:
coordenação e subordinação; orações reduzidas.
4. Conectivos.

Serão, ainda, apresentadas oportunidades de interação comunicativa.

Antes de começarmos a compreensão de textos, vamos revisar “Los Falsos Amigos” (Falsos
Cognatos) que são palavras semelhantes a do português e que se usadas de forma inadequada pode
dar a maior confusão.

Los Falsos Amigos

Hay algunas palabras que pueden inducir al estudiante brasileño a una comprensión equivocada de
su significación. Son los vocablos heterosemánticos, los cuales tienen grafía igual, o casi igual, a
palabras del portugués, pero con significados diferentes. Ejemplos:
La secretaria y el gerente de ventas trabajan en la misma oficina.
Aquella gata se pone rabiosa cuando uno se acerca a sus cachorros.
Mi nombre es Jair y mi apellido es Souza.

Los falsos amigos o falsos cognatos en español:

Almohada - Travesseiro Embarazada - Grávida Presunta - Suposta


Apellido - Sobrenome Escoba - Vassoura Regalo - Presente
Berro - Agrião Escritorio - Rojo - Vermelho
Escrivaninha
Bolsa - Sacola Estante - Prateleira Saco - Paletó
Bolsillo - Bolso Estofado - Ensopado Salada - Salgada
Borracha - Bêbada Exquisita - Gostosa Salsa - Molho
Borrar - Apagar Funda - Fronha Se Acercó - Se Aproximou
Brincar - Pular, Saltar Largo - Longo Se Acordó - Se Lembrou
Carpa - Barraca Latir - Bater Sitio - Lugar
Cachorros - Filhotes Muela - Dente Molar Sótano - Porão
Cena - Jantar Oficina - Escritório Rato - Momento
Cita - Encontro Oso - Urso Rubio - Loiro
Cola - Fila Pelado - Careca Tirar - Jogar
Copos - Flocos Pelo - Cabelo Vaso - Copo
Cuello - Pescoço Pimpollos - Botão Zorro - Raposa
Desquitar-se - Vingar-se Polvo - Poeira Zurdo - Canhoto

Cuestiones

1- Marca el heterosemántico:
(A) mujer.
(B) viaje.
(C) doler.
(D) bolso.

52
1103362 E-book gerado especialmente para MARCUS ICARO CRUZ DA SILVA
2- Señala la alternativa correcta para las traduciones de las palabras la gente, se enojó y taller:
(A) a gente – enojou-se – atelier.
(B) as pessoas – embraveceu – oficina.
(C) as pessoas – se embraveceu – escritório.
(D) a gente – enjou-se – oficina.

3- Respectivamente, los significados de las palabras “embarazada, rico, empezar, desarrollo, llamar,
exquisito” son:
(A) grávida, gostoso, começar, desenvolvimento, chamar, delicioso.
(B) embaraçada, milionário, pesar, desenvolvimento, lhama, esquisito.
(C) grávida, rico, começar, desenvolvimento, chamar, agradável.
(D) embaraçada, rico, pesar, desenvolvimento, lhama, esquisito.

Respuestas: 1-D / 2-B / 3-A.

Compreensão de Textos

Para que obtenha um bom desempenho na prova: primeiramente deve-se fazer uma leitura para
conhecer o contexto, prestando atenção às palavras-chave e grifando as ideias principais. E ao
responder às questões faça outra leitura, pois com isso você identificará com mais facilidade a resposta
correta. Observar os falsos amigos que podem estar presente no texto.

Lea el texto atentamente y a continuación escoja la alternativa adecuada para cada una de las
siguientes cuestiones.

China eliminará la ley que prohíbe tener más de un hijo

José Reinoso - Pekín

China eliminará gradualmente la política de hijo único (que fue puesta en marcha a finales de la
década de 1970 para controlar la explosión demográfica), según ha asegurado Zhao Baige, viceministra
de la Comisión Nacional de Población y Planificación Familiar. "Queremos cambiarla poco a poco. No
puedo decir cuándo o de qué manera, pero éste se ha convertido en un tema clave entre quienes toman
las decisiones en el Gobierno", ha declarado Zhao a la agencia Reuters.
El país asiático sufre un serio problema de envejecimiento de la población y una creciente disparidad
de género, que ha provocado la alarma entre los expertos. Se estima que, de seguir la tendencia actual,
en 15 años puede haber 30 millones de hombres más que de mujeres en edad de formar una familia, lo
que podría provocar migraciones, tráfico de mujeres e inestabilidad social.
Las autoridades aseguran que los estrictos controles de natalidad han evitado más de 300 millones
de nacimientos y han favorecido una elevación más rápida del nivel de vida del país.
Sin embargo, sus detractores afirman que, además de violar las libertades del individuo, han
desembocado en numerosos abortos y esterilizaciones forzadas, y el abandono de muchas niñas, dada
la preferencia de las familias por los varones.
En China, nacen 118 niños por cada 100 niñas, cuando el ratio normal en todo el mundo es entre 103
y 107 varones por cada 100 hembras. El 51,5% de los 1.314 millones de habitantes que tenía China en
2006 era varón.
La mayoría de las parejas en las ciudades sólo puede tener por ley un descendiente, mientras que en
las zonas rurales se les permite dos si el primero es niña. Las minorías étnicas pueden tener dos o más.
Zhao aseguró que el Gobierno está estudiando la cuestión con mucho cuidado para que cualquier
decisión que se tome no provoque un alza repentina de la población.

El País, Edición Impresa.

01- El texto afirma que, desde finales de 1970, en China:


(A) se lucha contra el exceso de nacimientos femeninos.
(B) quienes toman decisiones en el gobierno, son claves para elaboración de leyes contra la
expansión demográfica.
(C) la comisión nacional de planificación familiar quiere cambiar la situación gradualmente.
(D) se inició un control de expansión demográfica.

53
1103362 E-book gerado especialmente para MARCUS ICARO CRUZ DA SILVA
02- La frase de la viceministra de la Comisión Nacional de Población y Planificación Familiar (línea
3), afirma que “Queremos cambiarla poco a poco”. En la palabra CAMBIARLA, la partícula LA, se refiere
a:
(A) la ley.
(B) china.
(C) la década de 1970.
(D) la explosión demográfica.

03- El texto afirma que los problemas importantes señalados por los especialistas son:
(A) aumento y envejecimiento del número de mujeres.
(B) incremento de envejecimiento de la población y desproporción de géneros.
(C) población en disparidad demográfica y envejecimiento en algunas regiones.
(D) desproporción demográfica y tráfico de mujeres como consecuencia del envejecimiento de la
población.

04- Se estima que los problemas demográficos de China:


(A) pueden provocar éxodos humanos, desequilibrio social y comercio de mujeres.
(B) van a seguir la tendencia actual.
(C) deben provocar el control de natalidad de 300 millones de nacimientos.
(D) van a dificultar la elevación acelerada del nivel de vida.

05- En el tercer párrafo, línea 3, la expresión SIN EMBARGO, puede sustituirse, sin cambiar el
sentido de la frase por:
(A) sino.
(B) no obstante.
(C) sin duda.
(D) en consecuencia.

06- Las críticas a la política china de control de natalidad se basan en los siguientes principios:
(A) a la ausencia de derechos individuales, a la permisión de abortos, a la esterilización opcional y a
la discriminación de los nacimientos femeninos.
(B) a la obligación del aborto, a la falta de decisión personal para la esterilización forzada y al
abandono de las niñas.
(C) al abandono de las niñas, a incontables abortos, a la incumplimiento del derecho humano de la
libertad individual y la castración impuesta.
(D) a la desconsideración de la libertad de decidir personalmente el número de hijos, al abandono de
las recién nacidas, a la permisión de abortos y a la castración libre.

07- La legislación China establece que una pareja rural:


(A) solamente puede tener un descendiente, sea hombre o mujer.
(B) puede tener más de dos hijos, siempre que el primer descendiente sea mujer.
(C) se rige por la misma ley de las parejas de minorías étnicas.
(D) pueden tener como máximo dos hijos, cuando el primer descendiente sea mujer.

Hacienda acelera los embargos preventivos para cobrar las deudas

La recesión somete a las arcas del Estado a un doble perjuicio: los ingresos caen por la atonía de la
actividad y, a la vez, los incumplimientos tributarios se disparan. El Ministerio de Economía y Hacienda de
España admite que esa deuda está creciendo y se propone extremar las medidas para intentar cobrarla. La
necesidad es acuciante: en un año en que el Gobierno prevé ingresar como mínimo 40.000 millones de
euros menos de lo que proyectó inicialmente, cada euro recaudado cuenta.
Entre enero y agosto de este año se han adoptado medidas cautelares por valor de 753 millones,
cantidad 24% superior a la de todo el ejercicio 2008, según datos de la Agencia Tributaria (AT), que tiene
37.000 millones de deuda pendiente, con datos a 31 de julio. De esa cantidad, una deuda de algo más de
12.000 millones cuenta con algún tipo de garantía para ser cobrada. La más incierta es la que está en vía
ejecutiva (Hacienda intenta ya adoptar medidas como el embargo para cobrarla) y la que figura en período
de notificación (la fase previa). Ambas partidas suman casi 16.000 millones hasta final de julio. Sólo con
recuperar esas cantidades, el fisco ingresaría más dinero que con la subida de impuestos aprobada para
2010, que se acerca a 11.000 millones.
(El pais.es - Texto adaptado)

54
1103362 E-book gerado especialmente para MARCUS ICARO CRUZ DA SILVA
08- En el texto se dice que los cofres públicos españoles:
(A) sufren pérdidas.
(B) son independientes del sistema tributario.
(C) se destinan al pago de deudas del estado.
(D) están inmunes a la depresión.
(E) recibirán recursos de 40 mil millones de euros.

09- De acuerdo con el texto, el monto de los incumplimientos tributarios:


(A) es similar al del ejercicio anterior.
(B) está en vía ejecutiva.
(C) tiene garantías de cobrarse.
(D) es mayor que el ingreso esperado con el alza de impuestos.
(E) ha menguado en lo que va de año.

10- Según el texto, la baja de los ingresos:


(A) se deriva de las medidas cautelares.
(B) es fruto de una fiscalización ineficiente.
(C) alcanza los 37 mil millones de euros.
(D) es semejante a la del año fiscal 2008.
(E) se debe a la debilidad de la actividad.

Ecuador busca repatriar USD 500 millones

Ecuador repatriaría la próxima semana 500 millones de dólares de sus reservas internacionales de
corto plazo, como parte de las medidas que anunció para enfrentar el embate de la crisis global, informó
hoy la ministra de Finanzas, Elsa Viteri.
Ecuador, el socio más pequeño de la OPEP, anunció en agosto pasado que repatriará 1.600 millones
de dólares de la reserva internacional para aumentar el crédito productivo e irrigar dinero a la inversión.
Esa decisión en relación con el dinero depositado en financieras del exterior, que a mediados de agosto
se ubicó en 4.000 millones de dólares, fue justificada por el Gobierno con el argumento de que con su
economía dolarizada desde el 2.000 no se requiere de reservas y se constituyen en ahorro nacional.
La economía del país, productor de petróleo, banana y flores y con 14 millones de habitantes, se
contrajo un 1,06 por ciento en el segundo trimestre del 2009, frente a una expansión de un 8,28 por
ciento en el mismo lapso del 2.008.
(El Universal)

11- En el texto se dice que el gobierno ecuatoriano ha decido:


(A) invertir recursos en sus reservas internacionales.
(B) inyectar dinero en la economía.
(C) repatriar sus reservas externas.
(D) expandir sus recursos financieros en un 8,28%.
(E) destinar fondos foráneos al sector productivo.

12- De acuerdo con el texto, el regreso de fondos depositados en el extranjero:


(A) justifica el uso oficial de la moneda estadounidense.
(B) busca fortalecer al país en la opep.
(C) desincentiva el producto interno bruto.
(D) reduce las reservas y el ahorro nacional.
(E) se explica por la expansión de la economía local.

13- Según el texto, la economía ecuatoriana:


(A) recibirá recursos extranjeros.
(B) se dolarizará.
C) se ha recuperado este año.
D) intenta capear la crisis global.
E) encogió un 8,28% el año pasado.

55
1103362 E-book gerado especialmente para MARCUS ICARO CRUZ DA SILVA
Gobierno no se muestra seguro

Ni el propio Gobierno está seguro de que una rebaja en las tasas de interés – actualmente en 4% –
pueda contribuir a frenar la acentuada revaluación (más del 18,8% en los últimos meses) que afecta a
varios sectores de la economía.
Los empresarios achacan al actual nivel de las tasas de interés parte de la galopante apreciación del
peso que le resta competitividad a las exportaciones colombianas. Consideran que esta diferencia de
intereses está propiciando el ingreso de los llamados capitales golondrinas (de poca duración en la
economía) que están debilitando el dólar.
El ministro de Hacienda, Óscar Iván Zuluaga no se mostro muy seguro de que el pretendido recorte
sea una buena decisión en los actuales momentos en que la inflación sigue cediendo. Admitió, sin
embargo, que con las medidas adoptadas “hemos evitado que se revalúe” el dólar.

(El espectador.com)

14- Según el texto, en materia de tasas de interés, los empresarios colombianos:


(A) abogan por la apreciación.
(B) muestran inseguridad.
(C) discrepan del gobierno.
(D) relacionan los tipos con la inflación.
(E) están conformes.

15- En el texto se dice que las tasas de interés que se practican actualmente:
(A) elevan la cotización del dólar.
(B) perjudican las exportaciones.
(C) deben mantenerse.
(D) impiden la apreciación del peso.
(E) acentúan la inflación.

Comercio Chile-China

De acuerdo a un informe de Aduanas, China se consolidó como el principal destino de las


exportaciones chilenas con embarques por US$ 8.257 millones – en un intercambio bilateral de US$
11.881 millones – y el único de los socios comerciales hacia donde nuestros envíos crecieron (6,3%)
durante los primeros nueve meses de este año (los envíos totales menguaron un 35%). El incremento
registrado en los embarques al gigante asiático fue muy superior a las caídas entre 20% y 68%
anotadas en el resto de los países con los que Chile mantiene intercambio comercial, incluso socios tan
importantes como la Unión Europea, Aladi y Nafta, destinos a los que Chile exportó productos por un
monto menor a US$ 6.700 millones.
Las exportaciones chilenas a Estados Unidos cayeron un 32% al pasar de US$ 6.625 millones a US$
4.495 millones; mientras los envíos a Japón bajaron 45% de US$ 3.493 millones a US$ 1.501 millones.
En relación a Europa, El informe de Aduanas mostró que la cantidad embarcada también disminuyó en
los primeros nueve meses del año (49%) respecto del mismo lapso de 2.008, debido a La disminución
de los envíos a Italia y Holanda.
(Diario Financiero - texto adaptado)

16- En el texto se dice que las exportaciones chilenas:


(A) se han reducido a US$ 6.700 millones.
(B) rebasan las expectativas para el período.
(C) tuvieron un alza de 35%.
(D) se han diversificado.
(E) sólo crecieron hacia un destino.

17- De acuerdo con el texto, en el informe de Aduanas se dice que:


(A) el intercambio con China creció en ambas direcciones.
(B) los envíos a Estados Unidos se redujeron al 32%.
(C) Europa es la que compró menos productos chilenos.
(D) las exportaciones totales de Chile se contrajeron.
(E) el comercio exterior chileno da señales de recuperación.

56
1103362 E-book gerado especialmente para MARCUS ICARO CRUZ DA SILVA
La calle está tranquila

Zapatero ha declarado a Newsweek que para comprobar que España no se hunde «sólo hay que
salir a la calle». Subraya así la paradoja de que no exista conflictividad social con cuatro millones de
parados. Tiene razón en parte, aunque quizá debería preguntarse por qué la calle está tranquila y
entonces su análisis no podría ser tan complaciente. Según publicamos hoy, la econom ia sumergida ha
aumentado el 30% en el verano que ahora finaliza. Es el cálculo de los inspectores de Trabajo, que han
estimado en 320 millones de euros la recaudación en multas para 2009, una cifra récord. Así pues, en el
auge de la economía sumergida – de la que un país no puede sentirse orgulloso, sino todo lo contrario –
está la explicación de la calma de la calle, puesto que los parados optan por las «chapuzas» para ir
tirando. Aquí estaría la explicación de otro dato que Zapatero comentó, asombrado, ante el Comité
Federal: sólo 28.000 parados sin ingresos han solicitado los 420 euros, cuando los posibles
beneficiarios de esta medida son en torno a un millón. Como el subsidio lleva aparejada la obligación de
asistir a cursos de formación, tal vez la mayoría de estos desempleados sin ingresos en realidad sí
cobran por algún trabajo, aunque no declaran, ni cotizan. Ello quiere decir que la política social de
Zapatero se vuelve contra él y contra los que él dice proteger. El empleo que se crea es clandestin o por
las rigideces del mercado laboral y al subir los impuestos aumentará la economía del dinero negro.
(El mundo)

18- En el texto se dice que en España no hay conflictividad social:


A) por la solidez institucional.
B) gracias a las prestaciones públicas asistenciales.
C) a pesar del desempleo.
D) debido a la eficacia de la política laboral del gobierno.
E) pese al aumento de la recaudación por multas.

19- De acuerdo con el texto, las actividades económicas marginales en España:


A) disminuirán si suben los impuestos.
B) han aumentado.
C) elevarán las cotizaciones al tesoro.
D) garantizan un aumento de los ingresos públicos.
E) ocupan a cerca de un millón de parados.

20- Dentro del texto, la palabra «chapuzas» significa:


A) pequeñas contravenciones.
B) obras sin arte ni esmero.
C) contratos temporales.
D) cursos de reciclaje profesional.
E) subsidios fraudulentos.

Los 17 mil chilenos con mayores ingresos entregan el 35% del impuesto a la renta

En Chile 1.329.297 personas pagan impuestos a la renta, ya sea bajo la modalidad de segunda
categoría o de global complementario. A estas se suman otras 6.346.693 que si bien son formalmente
contribuyentes para los registros del Servicio de Impuestos Internos (SII), en la práctica están en el
primer tramo impositivo, que es cero. De este 1,32 millón de chilenos, el 1,27% está en el tramo más
alto de impuestos – con una tasa marginal del 40% – que es para quienes tienen ingresos de cerca de
$5,5 millones de pesos mensuales o más. En conjunto, pagan más tributos al año que los bancos o todo
el sector comercio. De acuerdo con los datos del SII, este grupo es el responsable del 35,1% de la
recaudación por impuesto a la renta; es decir, US$ 908 millones del total de US$ 2.589 millones que los
contribuyentes chilenos aportan por este concepto.
Para el socio principal de Ernst & Young, Cristián Levefre, la cifra demuestra que el grueso del aporte
de impuestos lo entregan quienes más dinero reciben. Según él, existe una discriminación entre los
contribuyentes de los tramos altos. Los independientes – muchos de ellos empresarios – tienen
opciones de desarrollar una planificación tributaria para reducir el pago y los dependientes no las tienen.
A su juicio, Chile debería impulsar beneficios para este segmento, como los que hay en economías más
desarrolladas, como EE.UU., Inglaterra y España, donde se deduce por pago de colegios e intereses de
créditos hipotecarios.
(El Mercurio)

57
1103362 E-book gerado especialmente para MARCUS ICARO CRUZ DA SILVA
21- Según el texto, el cobro del impuesto sobre la renta en Chile:
(A) se concentra en los mayores ingresos.
(B) incluye a la mayoría de la población.
(C) alcanza al 35% de los trabadores.
(D) castiga a los autónomos.
(E) es menor entre los empleados bancarios.

22- En el texto se aconseja:


(A) aumentar los tributos al sector bancario.
(B) disminuir los intervalos impositivos.
(C) introducir deducciones por determinados pagos.
(D) reducir la tasa marginal de los tramos de impuestos.
(E) gravar más a los independientes.

23- En el contexto del texto, “a su juicio” conserva su sentido al sustituirse por:


(A) a su aire.
(B) en su prejuicio.
(C) a lo suyo.
(D) en su opinión.
(E) a regañadientes.

24- De acuerdo con el texto, los contribuyentes dependientes chilenos:


(A) se benefician con desgravaciones.
(B) están concentrados en los tramos impositivos intermedios.
(C) cotizan más que los bancos.
(D) planifican el pago de sus tributos.
(E) sufren discriminación.

Europa ha fracasado en la política de empleo: OIT

Los gobiernos europeos han fracasado en la política de empleo, según reporta la Organización
Internacional Del Trabajo (OIT) en su nuevo informe "Mundo del trabajo", donde señala que con sus
programas de ahorro, los países europeos, sobre todo los del sur, no se han centrado en crear puestos
de trabajo, sino en recortar el déficit. El autor principal del informe, Raymond Torres, apela en su
análisis a los países a que emprendan "un cambio dramático" en el rumbo político. Agrega que la
estrategia de los países europeos de reducir el déficit apuntaba a allanar el camino a una mayor
inversión y crecimiento; "pero esas expectativas no se han cumplido", por su incapacidad de estimular la
inversión privada.
El director de la investigación señala que "la intensa concentración de muchos países de la eurozona
en La política de ahorro ha profundizado la crisis de empleo y podría conducir a más recesión en
Europa". Por el contrario, aquellos países que apuestan por una política de fomentar el empleo arrojan
mejores datos económicos. "Tenemos que observar bien esa experiencia y sacar conclusión de ello",
añadió. Desde 2011 el desempleo vuelve a crecer.
Y especialmente problemático considera esta agencia de la ONU el desempleo juvenil y de larga
duración. Para los países industrializados, la OIT no espera una recuperación en las cifras del
desempleo hasta 2016, cuando habrá cifras como las de antes de la crisis de 2008.
La Jornada en línea http://www.jornada.unam.mx/

25- Según el texto, la política europea de empleo ha fracasado porque:


(A) se han incumplido los programas de austeridad.
(B) ha aumentado el desempleo juvenil.
(C) ha habido un cambio de modelo productivo.
(D) ha continuado alto el déficit fiscal.
(E) ha faltado incentivo a la inversión.

26- En las líneas 5 y 6 del texto (…no se han centrado en crear puestos de trabajo, sino en recortar
el déficit), La palabra “sino” denota:
(A) idea de excepción.
(B) contraposición de ideas.

58
1103362 E-book gerado especialmente para MARCUS ICARO CRUZ DA SILVA
(C) adición de otro elemento.
(D) solamente.
(E) afrenta.

CGR imputó Responsabilidad Fiscal al Ex Gobernador Luis Alfredo Ramos y dos de sus ex
secretarios

La Contraloría General de la República (CGR) imputó cargos de responsabilidad fiscal, por un


presunto detrimento patrimonial superior a $18 mil millones de pesos, contra el ex Gobernador de
Antioquia, Luis Alfredo Ramos Botero, los ex secretarios de Infraestructura Física, Mauricio Restrepo
Gutiérrez y María Cristina Mesa Zapata, 10 consorcios y uniones temporales que abarcan 20 empresas
de obras civiles y la Fundación para el Buen Gobierno (hoy Fundación Cubo). La decisión se origina al
verificar, en más de 22 indagaciones preliminares y procesos fiscales, que hubo deficiencias en la
planeación contractual, demoras injustificadas en la ejecución de las obras contratadas y adiciones no
justificadas, entre otras irregularidades.
De otra parte, es importante señalar que en desarrollo de las indagaciones preliminares y procesos
de responsabilidad fiscal que se adelantaron en su momento sobre estos hechos, en las vigencias 2008
a 2010, se presentaron situaciones irregulares en la actuación de funcionarios de la CGR de la gerencia
departamental de Antioquia de esa época, que fueron puestos en conocimiento de la Fiscalía General
de la Nación, Regional Medellín.
(Adaptado de Controlaría General de la República de Colombia)

27- En el texto se dice que la Contraloría General de Colombia:


(A) calculó pérdidas patrimoniales de $18 mil millones de pesos, imputables al ex Gobernador de
Antioquia.
(B) informó a la Fiscalía sobre la participación de funcionarios de la CGR en actos de cohecho.
(C) abrió 22 procesos contra el ex Gobernador de Antioquia y funcionarios de su gobierno.
(D) reconoció la implicación de funcionarios suyos en El desvío de recursos públicos.
(E) acusó al ex Gobernador y descubrió irregularidades en la actuación de funcionarios de la CGR.

Wikileaks y la transparencia energética

Las filtraciones de Wikileaks que están sacudiendo a la opinión pública mundial también hablan de
energía y no, en este caso, para contar cotilleos sino para mostrar la inseguridad que rodea la
geopolítica del gas, del petróleo y de la energía nuclear. Los negocios del gas que tanto perjudican a la
seguridad europea, el descontrol en torno a la energía nuclear y la búsqueda de uranio, las cifras
infladas del petróleo, los riesgos en torno a oleoductos y rutas de abastecimiento, el desastre de la
cumbre de Copenhague sobre cambio climático, son algunas de las revelaciones de estos documentos
del Departamento de Estado norteamericano que confirman y ratifican los riesgos de mantener un
sistema energético dependiente al 100% de fuentes energéticas que no podemos controlar en absoluto,
ni en sus costes, ni en sus precios ni en sus riesgos.
En Wikileaks también se puede leer cómo el Ministro del Petróleo de Arabia Saudí apoyó
decididamente la energía solar para evitar la mala imagen de los países petroleros que se oponían a la
reducción de emisiones en la cumbre de Copenhague. El mismo doble lenguaje que se utiliza con las
energías renovables se aplica también a la lucha contra el cambio climático. El precio del CO2 se va a
multiplicar hasta 2020 y su impacto en la economía va a ser mayor a partir de 2012, cuando los
derechos de emisión no se repartan gratuitamente; mientras, tan sólo una décima parte de las
empresas lo tienen en cuenta.
Cada vez es más evidente que la pasividad va a resultar más cara. La contaminación atmosférica ya
provoca más muertes que la carretera y se extiende como una nueva epidemia invisible con costes
cada vez mayores. En nuestras costas es frecuente ver cómo todos los años se repone la arena de las
playas y los daños de sus paseos marítimos por los fenómenos cada vez más extremos del clima y de
los mares. La Agencia Española de Meteorología ha anunciado un incremento de 6 grados de la
temperatura en los próximos 60 años cuando el riesgo admitido tan solo es de 2 grados. ¿Alguien ha
calculado el coste económico y de bienestar de un cambio tan espectacular? Ningún gobierno piensa en
periodos de décadas y a pesar de que las evidencias crecen, se prefiere un discurso más complaciente
o simplemente ignorar el riesgo del cambio climático. Los resultados de
la cumbre de Cancún esconden una nueva falta de transparencia y la misma codicia que está en el
origen de la crisis financiera de 2008 y de la que en estos momentos ya se está gestando.

59
1103362 E-book gerado especialmente para MARCUS ICARO CRUZ DA SILVA
La crisis de 2008 tuvo su origen en la subida de los precios del crudo en el verano de 2004 que
acabó provocando la subida de los tipos de interés y la ruina de las hipotecas basura. Seis años
después el petróleo vuelve a encarecerse; ahora deberíamos estar advertidos. Todo el mundo parece
saber el coste de las renovables pero nadie conoce el coste real de los combustibles fósiles y de la
energía nuclear a medio y largo plazo. (...) Y aún a costa de no difundir que los pequeños avances
registrados, como la reducción de emisiones en el sector energético, se han debido a la mayor
producción de renovables.
Urge resolver estas contradicciones: no se puede defender la creación de empleo a través de la
economía verde con una regulación que los destruye y deslocaliza la industria nacional de renovables;
no se puede defender las renovables en los discursos e imponer, a la vez, un mayor consumo de
carbón y de gas. Es preciso pasar de una cultura energética que promueve el mayor consumo de
combustibles fósiles a otra que se base en el ahorro de energía y de emisiones de CO2. No se trata de
crear nuevos impuestos, sino de incentivar fiscalmente los hábitos de ahorro sobre el despilfarro y
promover de esta manera el uso racional de la energía. Hace años lo conseguimos con la cultura del
agua y hoy nadie lo cuestiona. Hagamos lo mismo con la energía. Esa es la propuesta de
corresponsabilidad de la Fundación Renovables, porque esa nueva cultura energética no corresponde
en exclusiva al consumidor eléctrico sino a todos los consumidores de energía, en las empresas, el
transporte, los hogares y las ciudades. Es una cuestión de equidad y de transparencia, la misma que
nos hace creer firmemente que la sociedad civil no ha muerto.
Javier García Breva es presidente de la Fundación Renovables.
Fuente: El País
Disponible en: http://calentamientoglobalclima.org/(Adaptado)

28- El fragmento del Texto I que permite identificar una crítica explícita del autor es:
(A) “El precio del CO2 se va a multiplicar hasta 2020 (…).” (línea 25-26)
(B) “Cada vez es más evidente que la pasividad va a resultar más cara.” (línea 30-31)
(C) “La contaminación atmosférica ya provoca más muertes que la carretera (…).” (línea 31-32)
(D) “La Agencia Española de Meteorología ha anunciado un incremento de 6 grados de la
temperatura en los próximos 60 años.” (línea 38-40)
(E) “La crisis de 2008 tuvo su origen en la subida de los precios del crudo en el verano de 2004 (…).”
(línea 51-52)

29- La lectura del primer párrafo posibilita inferir que Wikileaks a veces tiene como objetivo
reproducir:
(A) descontrol
(B) riesgos
(C) chismes
(D) inseguridad
(E) filtraciones

30- Acerca de las palabras costas (línea 34) y costa (línea 59) se afirma que:
(A) la primera tiene como idea fatiga o dispendio causado por algo.
(B) la segunda se refiere a orilla, sea del mar, de un río o de un lago.
(C) la segunda forma parte de una locución equivalente a: A expensas de, por cuenta de.
(D) la última significa parte posterior del tronco de un animal y solo se usa en plural.
(E) poseen significados equivalentes con una simple diferencia de número.

31- De acuerdo con el autor:


(A) la contaminación atmosférica, además del gran número de muertes que ya ha provocado, sigue
aumentando y presupone costes todavía mayores.
(B) la ruina de las hipotecas basura y la subida de los tipos de interés son los hechos que deflagraron
las crisis de 2004 y 2008.
(C) los resultados de la cumbre realizada en Cancún han escondido las consecuencias negativas de
la crisis financiera de 2008.
(D) el impacto del precio del CO2 en la economía empezará en el año de 2012 y seguirá hasta 2020,
a partir de la repartición de sus derechos gratuitos de emisión.
(E) es imprescindible la defensa de las renovables en los discursos y, a la vez, la imposición de un
gran consumo de carbón y de gas.

60
1103362 E-book gerado especialmente para MARCUS ICARO CRUZ DA SILVA
32- El último párrafo posibilita inferir que él término ahorro (línea 71) en el texto significa el resultado
de:
(A) diferenciar, variar, desunir o desviar.
(B) evitar un gasto o consumo mayor.
(C) exceder de lo debido.
(D) contravenir a lo razonable.
(E) quebrantar un precepto.

33- El conectivo sino en “No se trata de crear nuevos impuestos, sino de incentivar fiscalmente los
hábitos de ahorro sobre el despilfarro y promover de esta manera el uso racional de la energía” (línea
72-75) denota idea de:
(A) concomitancia
(B) coincidencia
(C) espacio
(D) explicación
(E) adversidad

Texto I

Lo que nos enseñan a los economistas

Muhammad Yunus

Discurso de aceptación del premio “Ayuda a la Auto-ayuda” de la Fundación Stromme. 26 de


septiembre de 1997, Oslo, Noruega.

No me enseñaron a entender la iniciativa personal. Me enseñaron, como a todos los estudiantes de


ciencias económicas, a creer que toda la gente, a medida que va creciendo, debe prepararse para
conseguir empleo en el mercado laboral. Si Ud. No logra conseguir un puesto, se inscribe para recibir
ayuda del gobierno. Pero no podía sustentar estas creencias cuando me enfrenté a la vida real de los
pobres en Bangladesh. Para la mayoría de ellos, el mercado de trabajo no significaba mucho. Para
sobrevivir, se concentraban en sus propias actividades económicas. Pero las instituciones políticas y
económicas no se daban cuenta de su lucha. Eran rechazados por las instituciones formales, sin haber
hecho nada para merecerlo.
Me asombraba ver como sufrían los pobres porque no podían conseguir una pequeña suma de
capital de trabajo - la cantidad que necesitaban era inferior a un dólar por persona. Algunos de ellos sólo
podían conseguir el dinero en condiciones muy injustas. Tenían que vender los bienes al prestamista al
precio arbitrario que él decidía.
Creamos instituciones y políticas basadas en la manera en que hacemos suposiciones sobre
nosotros y otros. Aceptamos el hecho que siempre habrá pobres entre nosotros. Por eso hemos tenido
gente pobre entre nosotros. Si hubiéramos creído que la pobreza es inaceptable para nosotros, y que
no debe pertenecer a un mundo civilizado, habríamos creado instituciones y políticas apropiadas para
crear um mundo sin pobreza. Queríamos ir a la Luna - y fuimos a ella. Queríamos comunicarnos unos
con otros muy rápidamente - por lo que hicimos los cambios necesarios en la tecnología de las
comunicaciones.
Logramos lo que queremos lograr. Si no estamos logrando algo, mi primera sospecha recae sobre la
intensidad de nuestro deseo de lograrlo.
Creo firmemente que podemos crear un mundo sin pobreza, si queremos. En ese mundo, el único
lugar para ver la pobreza es en un museo. Cuando los escolares visiten el museo de pobreza, se
horrorizarán al ver la miseria e indignidad de los seres humanos. Culparán a sus antepasados por
tolerar esta condición inhumana de una manera masiva.
Grameen me ha enseñado dos cosas: primero, nuestra base de conocimientos sobre las personas y
como actúan todavía es inadecuada; segundo, cada persona es muy importante. Cada persona tiene
gran potencial. Ella sola puede influir en las vidas de otros en comunidades, y naciones - dentro y más
allá de su propio tiempo. Cada uno de nosotros tenemos en nuestro interior mucho más de lo que
hemos tenido oportunidad de explorar hasta ahora. A menos que creemos un ambiente favorable para
descubrir los límites de nuestro potencial, nunca sabremos lo que tenemos dentro. Grameen me ha
dado fe, una fe inquebrantable en la creatividad de los seres humanos. Esto me lleva a creer que los

61
1103362 E-book gerado especialmente para MARCUS ICARO CRUZ DA SILVA
seres humanos no nacen para sufrir la desdicha del hambre y la pobreza. Sufren ahora, y sufrieron en el
pasado porque ignoramos al tema.
Disponible en: <http://isis.faces.ula.ve/computacion/emvi/textos/
yunus-economia.htm>

34- “No me enseñaron a entender la iniciativa personal. Me enseñaron, como a todos los estudiantes
de ciencias económicas, a creer que toda la gente, a medida que va creciendo, debe prepararse para
conseguir empleo en el mercado laboral.” (líneas 1-5) La locución conjuntiva destacada en el fragmento
aporta, en ese contexto, un sentido:
(A) causal
(B) condicional
(C) consecutivo
(D) final
(E) temporal

35- En el primer párrafo del Texto I el pronombre usted (Ud.) tiene valor de:
(A) acercamiento
(B) funcionalidad
(C) discontinuidad
(D) generalización
(E) informalidad

36- En el título del Texto I es posible comprender que el pronombre nos se refiere a los:
(A) concentrados en las actividades de ciencias económicas
(B) economistas como aprendices
(C) miembros del mercado laboral
(D) electores del autor como ganador del premio
(E) profesores de ciencias económicas

37- Una idea presente en el tercer párrafo es:


(A) el mundo civilizado ha sido responsable por la situación actual de los pobres.
(B) el mundo solo puede mejorar desde nuevas políticas públicas.
(C) la injusta realidad existente en el mundo se debe a las instituciones políticas.
(D) las clases políticas son las únicas capaces de extinguir las diferencias sociales.
(E) los antepasados no tienen la responsabilidad de las condiciones inhumanas de hoy.

38- Muhammad Yunus hace algunos análisis relativos a su entorno y la que encuentra
correspondencia de sentido en el texto es:
(A) La vida en comunidades es la solución capaz de eliminar el hambre, la pobreza y las diferencias
sociales en general.
(B) Las iniciativas de los estudiantes de económicas que se preparan para el mercado de trabajo son
incomprensibles.
(C) Las condiciones inhumanas en las que vive gran parte de la población se tienen que transformar
en cosa del pasado.
(D) Los conocimientos que tenemos acerca de las personas aún hoy día actúan de forma
inadecuada.
(E) No se puede culpabilizar a los hombres en general por las diferencias sociales, sino a las clases
políticas.

39- “Para sobrevivir, se concentraban en sus propias actividades económicas.” (líneas 10-12) En el
Texto I, en el pronombre sus se refiere a:
(A) creencias
(B) economistas
(C) estudiantes de económicas
(D) instituciones políticas
(E) pobres de Bangladesh

62
1103362 E-book gerado especialmente para MARCUS ICARO CRUZ DA SILVA
Texto II
Los nadie

Eduardo Galeano

Sueñan las pulgas con comprarse un perro y sueñan los nadie con salir de pobres,
que algún mágico día llueva de pronto la buena suerte,
que llueva a cántaros la buena suerte;
pero la buena suerte no llueve ayer, ni hoy, ni mañana, ni nunca.
Ni en lloviznita cae del cielo la buena suerte,
por mucho que los nadie la llamen,
aunque les pique la mano izquierda,
o se levanten con el pie derecho,
o empiecen el año cambiando de escoba.
Los nadie: los hijos de nadie, los dueños de nada.
Los nadie: los ningunos, los ninguneados, corriendo la liebre,
muriendo la vida, jodidos, rejodidos.
[…]
Que no hablan idiomas, sino dialectos.
Que no profesan religiones, sino supersticiones.
Que no hacen arte, sino artesanía.
Que no practican cultura, sino folklore.
Que no son seres humanos, sino recursos humanos.
Que no tienen cara, sino brazos.
Que no tienen nombre, sino número.
Que no figuran en la historia universal,
sino en la crónica roja de la prensa local.
Los nadie, que cuestan menos que la bala que los mata.
Disponible en: <http://info.nodo50.org/Los-nadies.html>

40- Por medio del uso del modo subjuntivo en el Texto II, Galeano:
(A) aporta sus dudas con relación a la condición de los nadie.
(B) enseña sus propias ganas y asimismo opiniones acerca del tema.
(C) expresa las condiciones reales contra las cuales combate.
(D) indica lo que serían los deseos de los nadie.
(E) niega los hechos producidos por la realidad.

41- A lo largo de un texto, los autores, en general, usan varias designaciones que se refieren a una
misma expresión con la finalidad de evitar repeticiones innecesarias. En el Texto II, la única expresión
por la cual NO se puede sustituir los nadies es:
(A) hijos de nadie (línea 10)
(B) dueños de nada (línea 10)
(C) ningunos (línea 11)
(D) ninguneados (línea 11)
(E) corriendo la liebre (línea 11)

42- Entre las líneas 13 y 21 del Texto II se identifican diversas oposiciones que indican:
(A) el menosprecio de los excluidos en lo que atañe a los ricos.
(B) el poco valor que se le otorga a lo que viene de los excluidos sociales.
(C) la contradicción entre lo que desean las clases dominantes y dominadas.
(D) la legitimación de la lucha de los que desean cambiar algo.
(E) los objetos que representan los resultados de las clases menos privilegiadas.

43- El Texto II presenta la idea de que:


(A) la historia universal se cuenta para ilusionar y engañar a los excluidos.
(B) la prensa local y la prensa roja ignoran a los desfavorecidos.
(C) las supersticiones son insuficientes para cambiar la vida de la gente pobre.
(D) los desfavorecidos cuentan con la lluvia para mejorar sus cosechas.
(E) los nadie aceptaron su destino y ya no esperan por mejores días.

63
1103362 E-book gerado especialmente para MARCUS ICARO CRUZ DA SILVA
Texto I
Sindicatos europeos presentan su receta contra la crisis

Crisis en Europa

La Confederación Europea de Sindicatos (CES) presentó hoy su receta contra la crisis en Breslavia
(Polonia), donde se celebra una reunión de ministros de Finanzas de la Unión Europea.
La CES ha convocado una manifestación para el sábado en la ciudad polaca, en la que calcula que
participarán unos 40.000 trabajadores de toda Europa.
La “euro manifestación” recorrerá las calles de Breslavia para mostrar el desencanto con las medidas
de austeridad y pedir a los titulares de Economía comunitarios medidas urgentes que permitan la
creación de empleo en el continente.
“Después de dos años de una crisis devastadora, los líderes europeos deben apostar por la
solidaridad y no dejar que los mercados financieros y a las agencias de calificación tomen el mando”,
explicó la secretaria general de la CES, Bernadette Ségol.
El presidente de la CES y secretario general de CCOO (España), Ignacio Fernández Toxo, lamentó
las malas políticas desarrolladas por la Unión Europea (UE), especialmente “a partir de mayo de 2010”,
ya que “han puesto el acento en la reducción del déficit público, lo que a su vez está provocando una
ralentización en la salida de la crisis y el aumento del desempleo”, dijo.
Toxo presidirá la manifestación de mañana, que transcurrirá por la capital de la Silesia polaca
mientras los ministros de Economía de la UE mantienen a escasos metros un encuentro informal para
debatir el futuro del rescate a Grecia y las medidas para superar la crisis del euro.
“Es preciso resolver de forma urgente el segundo rescate griego y es preciso enviar un mensaje
urgente al mundo financiero que deje claro que Europa está dispuesta a sostener la moneda única”,
explicó el sindicalista español, quien defendió que así se evitarán movimientos especulativos como los
que provocaron “una situación dramática como la vivida en el pasado mes de agosto”.
“Nos encontramos en un tiempo muy difícil y en ocasiones sentimos que estamos al borde del
colapso”, reconoció Ségol, que lamentó el aumento del paro, la precariedad laboral y la presión sobre
los salarios y el sistema de relaciones laborales.
La CES recordó que sus recetas pasan por la solidaridad en Europa, incluyendo la activación de los
eurobonos, el uso de los fondos estructurales disponibles a nivel europeo, la introducción de una tasa a
las transacciones financieras, la creación de una base imponible justa y la lucha contra el fraude fiscal.
(EFE)
Disponible en: <http://laverdad.es/agencias/20110916/economia/
sindicatos-europeos-presentan-receta-contra-201109161528>.

44- Además de la voz del autor, la noticia reúne voces de otras personas, cuya función sería
aportarle al texto:
(A) autoridad
(B) cohesión
(C) creatividad
(D) oposición
(E) realidad

45- En el quinto párrafo, se puede sustituir el término ralentización (línea 26), sin perjudicar el sentido
del texto, por:
(A) especulación
(B) financiación
(C) lentificación
(D) restructuración
(E) sindicalización

46- El Texto I aborda actitudes y comportamientos de personas y entidades. Con relación a eso,
¿cuál de las siguientes lecturas encuentra correspondencia de sentido en el texto?
(A) El presidente de los sindicatos, Ignacio Fernández Toxo, asevera que no se puede permitir de
ninguna forma que los mercados financieros y las agencias de calificación sean los que manejen la
situación.
(B) El secretario general de la CCOO afirma que los Fondos estructurales disponibles a nivel europeo
y el Fraude Fiscal son los principales responsables de la crisis que se ha instaurado en Europa.

64
1103362 E-book gerado especialmente para MARCUS ICARO CRUZ DA SILVA
(C) La Confederación Europea de Sindicatos (CES) propuso medidas de austeridad que impiden a
los titulares de la Economía seguir con la creación de empleo en el continente.
(D) La secretaria general de la CES se compadece del aumento de los paros, de la calidad precaria
de las condiciones de trabajo y también de la presión sobre los salarios y sobre el sistema de relaciones
laborales.
(E) Los poloneses han hecho una manifestación en Breslavia en contra las medidas de austeridad y
las medidas urgentes que permiten la creación de empleo en el continente.

47- En el tercer y cuarto párrafo del Texto I, se encuentran varias comillas. La justificación para su
uso es:
(A) 3er - destacan una palabra que forma parte del castellano.
(B) 3er - determinan enumeración de elementos.
(C) 4o - marcan la introducción de voz distinta a la del autor.
(D) 4o - traducen un equívoco del autor.
(E) En los dos párrafos porque señalan el discurso directo libre.

48- En el texto I, el conectivo mientras (línea 29) tiene sentido de:


(A) adversidad
(B) analogía
(C) causa
(D) comprobación
(E) concomitancia

49- En el texto I, a escasos metros (líneas 30-31) es una locución:


(A) causal
(B) consecutiva
(C) temporal
(D) de lugar
(E) de conformidad

La periodista Lydia Cacho abandona


México tras las últimas amenazas

La periodista Lydia Cacho ha decidido abandonar México después de haber recibido varias
amenazas de muerte en las últimas semanas. Cacho ha aclarado que su marcha es temporal. “Sólo Salí
mientras hacemos estrategia de seguridad. Nadie me sacará de mi hogar”.
El último suceso que empujó a la escritora a tomar esta decisión fue la amenaza recibida mientras se
encontraba en su domicilio de Cancún. La página web de Amnistía Internacional
México cuenta que Cacho atendió una llamada al creer que se trataba de un compañero de trabajo
que intentaba localizarla.
Ricardo González, portavoz de Artículo 19, ha confirmado que la periodista estará
“momentáneamente fuera de México hasta que se den las condiciones para que pueda desarrollar su
vida dentro con normalidad”. La asociación que defiende los derechos de los
profesionales de la comunicación considera que el Estado ha respondido de manera “muy lenta y a
cuentagotas” en este caso.
Artículo 19 asegura que la periodista cuenta con una amplia red de apoyo y que su estado anímico
es bueno. “Ella es una persona muy fuerte, muy consciente de los riesgos que sufre, pero a la vez muy
apegada a sus convicciones. Por eso sabe que esta lucha no es sólo por ella, sino también por muchos
otros periodistas”.
La activista empezó a recibir amenazas y agresiones después de publicar en el año 2005 el libro Los
Demonios del Edén, en el que denunció una red de pornografía infantil que habría actuado con el
conocimiento y la protección de políticos y empresarios de los estados de Quintana Roo y Puebla.
Internet: <http://internacional.elpais.com> (adaptado).

50- Según el texto, Lydia Cacho:


A) es una periodista que fue amenazada cuando estaba en su oficina.
B) afirmó que jamás regresará a México.

65
1103362 E-book gerado especialmente para MARCUS ICARO CRUZ DA SILVA
C) recibió una llamada amenazadora por equívoco; en realidad, la amenaza iba dirigida a un
compañero.
D) está escondida en algún lugar de México.
E) ejerce una profesión que está relacionada con la información y la investigación.

51- El vocablo “suceso” (R.5) podría sustituirse correctamente por:


A) investigación.
B) presunto.
C) secuestro.
D) hecho.
E) éxito.

52- De acuerdo con el texto:


A) Lydia Cacho muestra señales de arrepentimiento del trabajo que realiza.
B) las amenazas que Lydia Cacho está recibiendo son recientes.
C) el Estado ha respondido rápida y eficazmente a la situación vivida por Lydia Cacho.
D) en 2005 Lydia Cacho publicó una obra en la que denuncia la ocurrencia de abuso infantil con el
beneplacito de algunos políticos y empresarios.
E) Lydia Cacho se siente bastante abandonada por sus compañeros en su lucha.

Respuestas: 01-D / 02-A / 03-B / 04-A / 05-B / 06-C / 07-D / 08-A / 09-D / 10-E / 11-B / 12-A / 13-D /
14-C / 15-B / 16-E / 17-D / 18-C / 19-B / 20-B / 21-A / 22-C / 23-D / 24-E / 25-A / 26-C / 27-A / 28-B / 29-
C / 30-C / 31-A / 32-B / 33-E / 34-E / 35-D / 36-B / 37-A / 38-C / 39-E / 40-D / 41-E / 42-B / 43-C / 44-A /
45-C / 46-D / 47-C / 48-E / 49-D / 50-E / 51-D / 52-D

Podemos, tranquilamente, ser bem-sucedidos numa interpretação de


texto. Para isso, devemos observar o seguinte:

- Ler todo o texto, procurando ter uma visão geral do assunto;


- Se encontrar palavras desconhecidas, não interrompa a leitura, vá até o fim, ininterruptamente;
- Ler, ler bem, ler profundamente, ou seja, ler o texto pelo menos umas três vezes;
- Ler com perspicácia, sutileza, malícia nas entrelinhas;
- Voltar ao texto tantas quantas vezes precisar;
- Não permitir que prevaleçam suas ideias sobre as do autor;
- Partir o texto em pedaços (parágrafos, partes) para melhor compreensão;
- Centralizar cada questão ao pedaço (parágrafo, parte) do texto correspondente;
- Verificar, com atenção e cuidado, o enunciado de cada questão;
- Quando duas alternativas lhe parecem corretas, procurar a mais exata ou a mais completa;
- Quando o autor apenas sugerir ideia, procurar um fundamento de lógica objetiva;
- Cuidado com as questões voltadas para dados superficiais;
- Não se deve procurar a verdade exata dentro daquela resposta, mas a opção que melhor se
enquadre no sentido do texto;
- Às vezes a etimologia ou a semelhança das palavras denuncia a resposta;
- Procure estabelecer quais foram as opiniões expostas pelo autor, definindo o tema e a mensagem;
- O autor defende ideias e você deve percebê-las;
- Esclarecer o vocabulário;
- Entender o vocabulário;
- Viver a história;
- Ative sua leitura;
- Ver, perceber, sentir, apalpar o que se pergunta e o que se pede;
- Não se deve preocupar com a arrumação das letras nas alternativas;
- As perguntas são fáceis, dependendo de quem lê o texto ou como o leu;
- Cuidado com as opiniões pessoais, elas não existem;
- Sentir, perceber a mensagem do autor;

66
1103362 E-book gerado especialmente para MARCUS ICARO CRUZ DA SILVA
- Cuidado com a exatidão das questões em relação ao texto;
- Descobrir o assunto e procurar pensar sobre ele.

Vejamos dois textos e suas traduções para ampliação do vocabulário:

Texto I

El Impacto de Cambios en la Educación: las Nuevas Demandas

María Inés Abrile de Vollmer

La mayoría de los sistemas educativos ha iniciado procesos de reformas y transformaciones, como


consecuencia de la aguda conciencia del agotamiento de un modelo tradicional que no ha sido capaz de
conciliar el crecimiento cuantitativo con niveles satisfactorios de calidad y de equidad.
Hoy hablamos de un nuevo orden mundial competitivo basado en el conocimiento, en el cual la
educación y la capacitación son el punto de apoyo de largo plazo más importante que tienen los
gobiernos para mejorar la competitividad y para asegurar una ventaja nacional.
El funcionamiento óptimo de los sistemas educativos pasa a ser una prioridad esencial de los países.
Enunciamos a continuación las principales demandas que los cambios plantean a los sistemas
educativos y que se incorporan a las agendas de especialistas y de gobernantes:
- Preparar ciudadanas y ciudadanos capaces de convivir en sociedades marcadas por la diversidad,
capacitándolos para incorporar las diferencias de manera que contribuyan a la integración y a la
solidaridad, así como para enfrentar la fragmentación y la segmentación que amenazan a muchas
sociedades en la actualidad.
- Formar recursos humanos que respondan a los nuevos requerimientos del proceso productivo y a
las formas de organización del trabajo resultantes de la revolución tecnológica.
- Capacitar al conjunto de la sociedad para convivir con la racionalidad de las nuevas tecnologías,
transformándolas en instrumentos que mejoren la calidad de vida.

Tradução:

O Impacto da Mudança na Educação: as novas demandas

Maria Inés Abrile de Vollmer

A maioria dos sistemas educativos tem iniciado processos de reformas e transformações, como
consequência da aguda consciência do esgotamento de um modelo tradicional que não tem sido capaz
de conciliar o crescimento quantitativo com os níveis satisfatórios de qualidade e de equidade.
Hoje falamos de uma nova ordem mundial competitiva baseada no conhecimento, no qual a
educação e a capacitação são os pontos de apoio de largo prazo mais importante que têm os governos
para melhorar a competitividade e para assegurar uma vantagem nacional.
O funcionamento ótimo dos sistemas educativos passa a ser uma prioridade essencial dos países.
Enunciamos a seguir as principais demandas que as mudanças colocam aos sistemas educativos e que
se incorporam para as agendas de especialistas e de governantes:
- Preparar cidadãs e cidadãos capazes de conviver em sociedades marcadas pela diversidade,
capacitando-os para incorporar as diferenças de maneira que contribuam para a integração e para a
solidariedade, assim como para enfrentar a fragmentação e a segmentação que ameaçam muitas
sociedades na atualidade.
- Formar recursos humanos que respondam aos novos requerimentos do processo produtivo e às
formas de organização do trabalho resultante da revolução tecnológica.
- Capacitar ao conjunto da sociedade para conviver com a racionalidade das novas tecnologias,
transformando-as em instrumentos que melhorem a qualidade de vida.

67
1103362 E-book gerado especialmente para MARCUS ICARO CRUZ DA SILVA
Texto II

Dimensiones en la Formación del Profesorado

Marcos Sarasola / Cecilia Von Sanden

Nuestra idea de una visión integral de la formación docente toma en cuenta diferentes aspectos.
Existe una dimensión cultural (inter-subjetiva), relacionada con la construcción colectiva de significados
comunes que se van haciendo implícitos para una determinada comunidad hasta transformarse en
supuestos básicos: cultura organizacional.
Existe una dimensión subjetiva que es la de la percepción de los educadores, cómo vive cada cual,
con su propia historia su formación y su práctica. Existe una dimensión objetiva de la formación docente
en la que ubicamos los diferentes currículos, metodologías, herramientas, medios auxiliares,
tecnologías. Y, estas tres dimensiones, también están condicionadas por una cuarta: el contexto en el
que se desarrollan (geográfico, social, económico, cultural, político), el sistema educativo y el centro de
formación.
Una formación integral e integrada supone incidir en todas las dimensiones, ya que todas ellas están
profundamente imbricadas. Todas las dimensiones están relacionadas, vinculadas, son parte de un todo
y no se puede interpretar una sin las otras.
En este sentido, la formación debería ser integral, no por agregar materias nuevas y más contenidos,
sino por el enfoque. Integral porque concibe al ser humano como una totalidad mente-cuerpo-emoción,
y al aprendizaje como un proceso de construcción colectiva. Creemos, finalmente, que una comunidad
educativa cimentada en personas desarrolladas plenamente, con capacidad de amar, es un
apasionante desafío para la formación del profesorado.

Tradução:

Dimensões da Formação Docente

Marcos Sarasola / Cecilia Von Sanden

Nessa ideia de uma visão integral da formação docente toma em conta diferentes aspectos. Existe
uma dimensão cultural (inter-subjetiva), relacionada com a construção coletiva de significados comuns
que se vão fazendo implícitos para uma determinada comunidade até se transformar em supostos
básicos: cultura organizacional.
Existe uma dimensão subjetiva que é a da percepção dos educadores, como vive cada qual, com
sua própria história, sua formação e sua prática. Existe uma dimensão objetiva da formação docente na
que situamos os diferentes currículos, metodologias, ferramentas, recursos auxiliares e tecnologias. E
estas três dimensões, também estão condicionadas por uma quarta: o contexto no qual se desenrolam
(geográfico, social, econômico, cultural e político), do sistema educativo e do centro de formação.
Uma formação integral e integrada supõe incidir em todas as dimensões, já que todas elas estão
profundamente ligadas. Todas as dimensões estão relacionadas, vinculadas, são parte de um todo e
não se pode interpretar uma sem as outras.
Neste sentido, a formação deveria ser integral, não por agregar matérias novas e más conteúdos, e
sim pelo enfoque. Integral porque concebe ao ser humano como uma totalidade mente-corpo-emoção, e
ao aprendizado como um processo de construção coletiva. Cremos, finalmente, que uma comunidade
educativa construída em pessoas desenvolvidas plenamente, com capacidade de amar, é um
apaixonante desafio para a formação docente.

Traduzir para o Português o trecho a seguir:

Las Industrias de este País

Desearía visitar al Estado de San Pablo.


Me han dicho que es el mayor centro industrial sudamericano.
Sin duda. Y el Estado de Rio es el segundo,
Tenemos en Rio unas siete mil fábricas, siendo veinte grandes fabricas de tejidos.
El Estado de Rio posee también muchos establecimientos industriales. ¿Pero dónde están las
fabricas cariocas, que no se ven?

68
1103362 E-book gerado especialmente para MARCUS ICARO CRUZ DA SILVA
Para verlas hay que recorrer todo el Estado de Rio, que es casi del tamaño de un país cómo
Portugal, por ejemplo.
Sabe, amigo, cuanto más paseo en esta capital, más me siento maravillado.
Siempre encuentro nuevos barrios atrás de las montañas.
Cómo hay esas montañas, imaginamos que allí se acaba la ciudad.
No creía que Rio fuera tan grande.
Si, existen centenas de montañas y montes entre los barrios,
¿Cuántos son nichos barrios?
Llegan casi a doscientos, y algunos tienen doscientos a cuatrocientos mil habitantes.
Podria decirme cuales son los principales?
Como nó! Meyer, etc ,,,

As Indústrias deste País

Desejaria visitar o Estado de São Paulo.


Disseram-me que é o maior centro industrial sul-americano.
Sem dúvida, é o Estado do Rio o segundo.
Temos no Rio umas sete mil fábricas, sendo vinte grandes fábricas de tecidos.
O Estado do Rio possui também muitos estabelecimentos industriais.
Mas onde estão essas fábricas cariocas, que não se veem?
Para vê-las faz-se preciso percorrer todo o Estado do Rio, que é quase do tamanho de um país como
Portugal, por exemplo.
Sabe, amigo, quanto mais passeio nesta capital, mais me sinto maravilhado. Sempre encontro novos
bairros atrás das montanhas.
Não acreditava fosse tão grande o Rio.
Sim, existem centenas de montanhas e montes entre os bairros.
Quantos são ditos bairros?
Chegam quase a duzentos, e alguns têm de 200.000 a 400.000 habitantes.
Poderia dizer-me quais são os principais?
Pois, não! - Meyer, Penha , cascadura, Tijuca, Botafogo, Madureira, Copacabana, São Cristóvão,
Jacarepaguá.
Traduzir para o Português:

Se cree usted que vendrá la Tercera Guerra Mundial?


Si no fuere posible amonizar a los intereses comerciales de las grandes potencias, habrá guerra.
El motivo de las luchas entre naciones es siempre el mismo.
¿Por qué atacó a Alemania el mundo (2ª Guerra)?
Las razones son simples: El pueblo alemán trabajaba activamente. Sus mercaderías eran mejores y
más baratas.
Por eso inventaron que Alemania quería conquistar al mundo, ¿No estaban engañados los países,
pensando que las guerras resuelven a sus problemas?

Tradução

Acredita você que virá a Terceira Guerra Mundial?


Se não for possível harmonizar,os interesses comerciais das grandes potências, haverá guerra,
O motivo das lutas entre nações é sempre o mesmo,
Por que o mundo atacou a Alemanha (2ª Guerra)?
As razões são simples: O povo alemão trabalhava ativamente. As suas mercadorias eram melhores
e mais baratas.
Por isso inventaram que a Alemanha queria conquistar o mundo.
Não estarão enganados os países, pensando que as guerras resolvem seus problemas?

Textos para Leitura

Trabaja

Trabaja, joven; sin cesar trabaja:


La frente honrada que en sudor se moja

69
1103362 E-book gerado especialmente para MARCUS ICARO CRUZ DA SILVA
Jamás ante otra frente se sonroja,
Ni se rindo servil a quien la ultraja,

Tarde la nieve de los años cuaja


Sobre quien lejos la indolencia arroja;
Su cuerpo al roble, por lo fuerte enoja;
Su alma del mundo al lodazal no haya,

El pan que da el trabajo es ‘mas sabroso


Que la escondida miel que con empeño
Liba la abeja en el rosal frondoso.

Si comes use pan serás tú dueño,


Mas si del ocio ruedas al abismo,
Todo serlo podrás, menos tú mismo.
La Palabra

Naturaleza: gracias por este don supremo


del verso, que me diste:
yo soy la mujer triste
a quien Caronte ya mostró su remo.

Qué fuera de mi vida sin la dulce palabra?


Como el óxido labra
sus arabescos ocres,
yo me grabé en los hombres, sublimes o mediocres.

Mientras vaciaba el pomo, caliente, de mi pecho.


no sentía el acecho.
Torvo y feroz, de la sirena negra.

Me salí de mi carne, goce el goce más alto:


oponer una frase de basalto
al genio oscuro que nos desintegra

María

Virgen sin mancha, como el sol hermosa,


Virgen más pura que la luz del alba,
Flor de las flores, del amor estrella;
Virgen María ,

Madre de Dios y de los hombres madre,


Cielos y tierra en tu esplendor se fozan;
Hija de Adán, los serafines te alzan
Trono viviente,

Mística rosa del amor divino;


Cuya hermosura al contemplar el ángel
Besa tu sombra y remontando el vuelo,
Canta arrobado,

Así la alondra, con el sol de oriente,


Canta agitando, sin volar, sus alas;
Y sobre el nido, en extásis materno,
Ciérnese inmóvil ,

“Toda eres bella" el serafín te canta,


"Toda eres pura" te saluda el ángel,

70
1103362 E-book gerado especialmente para MARCUS ICARO CRUZ DA SILVA
"Llena de gracia y del Señor bendita".
Todas las gentes.

Tuyu es el nombre que en la cuna el niño,


Oye el arrullo del amor materno;
Tuyo es el nombre que tu la lucha invoca
Todo el que triunfa,
R. del Valle Ruiz

Imigracão

Cuando el Emperador Don Pedro lI libertó a los esclavos, la alta clase se revoltó, expulsándolo del
Brasil.
Ia que debían pagar a el trabajador, prefirieron importar italianos y otros.
En los siglos pasados, portugueses y españoles descubrieron innumerables tierras.
Ellos fueron los señores del Nuevo Mundo.
Con el tiempo fueron perdiendo los territorios descubiertos y conquistados. Los países se han
tornado independientes; las islas les fueron tomadas, unas fueron vendidas.
Si en Estados Unidos tantos lugares aun conservan nombres españoles es porque aquello era
español.
Europa es un continente tan pobre, que no puede nutrir a sus hijos, ni darles bastante trabajo.
Por eso son forzados a emigrar en millones.

Sueño Y Despertar

He soñado que estabas a mi verá


y que tenía tus manos en las mías;
ya no recuerdo lo que me decías.
pero era dulce oírte, compañera.

Me mirabas de amor, con la sincera,


clara mirada de los bellos días
y se iban enredando mis poesías
en el perfume de tu cabellera,

Era tan dulce oírte, y era tanta


la maravilla de tu voz serena
que, al sentir mi sellar desvanecido.

Me desperté con llanto en la garganta,


y las carnes doliéndome de pena,
y el corazón doliéndome de olvido.

José Maria Souvirón

Tobías

Un día estaba Tobías en la plaza, esperando el autobús, cuando se le acercó un muchacho muy
joven que le dijo:
- Por favor, ¿puede usted darme fuego?
- Enciende del mío - le dijo Tobías, sin quitarse el cigarrillo de la boca.
EI muchacho, a pesar de sus esfuerzos, no pude llegar con su cigarrillo al de Tobías porque este era
bastante alto. Al fin, tras inútiles esfuerzos, dijo:
- Lo siento, señor; pero no alcanzo su cigarrillo. ¿Puede bajarlo un poco, por favor?
- Más lo siento yo - respondió Tobías; cuando crezcas lo suficiente y consigas alcanzar, entonces
podrás fumar.

Juan D. Luque Durán


Cuestiones

71
1103362 E-book gerado especialmente para MARCUS ICARO CRUZ DA SILVA
01- Tobías estaba en
(A) la playa;
(B) una plaza;
(C) una finca;
(D) la carretera;
(E) la urbanización.

02- Tobías estaba esperando


(A) el tren;
(B) el taxi;
(C) el carro;
(D) el tranvía;
(E) el ómnibus.

03- Tobías le dijo al muchacho que


(A) no tenia fuego;
(B) comprara fósforos;
(C) no fumara cigarros;
(D) el tabaco hacia daño;
(E) prendera de su cigarrillo.

04- Al muchacho le resultó:


(A) facíl alcanzar el cigarrillo de Tobías;
(B) oportuno llegar con su cigarrillo al de Tobías;
(C) posible obtener fuego porque Tobías se nclinó;
(D) agradable alzarse a la altura de la boca de Tobías;
(E) imposible prender el cigarrillo debido a su estatura.

05- Según Tobías, el muchacho podría fumar cuando


(A) se graduara;
(B) se jubilara;
(C) fuera más alto;
(D) tuviera suficiente plata;
(E) tuviera matudurez y fuera más alto.

06- Con base en el texto, la intención de Tobías fue:


(A) loable;
(B) indigna;
(C) egoísta;
(D) excéntrica;
(E) reprochable.

07- La opción cuja secuencia no se completa es:


(A) mar, so, playa;
(B) iglesia, cura, misa;
(C) cocina, casa, paraguas;
(D) luna, noche, estrellas;
(E) farmacia, medicinas, enfermo.

08- La opción donde no aparece un par de antónimos es:


(A) alto-bajo;
(B) venir-salir;
(C) llena-vacía;
(D) bella-hermosa;
(E) moderna-antigua.

09- En una escuela, solo no se ve:


(A) maestro, tiza, libras;
(B) alumnos, mapas, dibujos;

72
1103362 E-book gerado especialmente para MARCUS ICARO CRUZ DA SILVA
(C) reglas, cerdos, estantes;
(D) mesas, sillas, antejos
(E) pupitres, lápices, cuadernos.

10- Coche es el mismo que


(A) barco;
(B) ómnibus;
(C) tranvía;
(D) aeroplano;
(E) diligencia.

11- Numere la columna de la derecha para la de la izquierda, según la contestación:


1) ¿Qué día es hoy?
2) ¿Quién ha escrito?
3) ¿Cuál es su dirección?
4) ¿A cuánto estamos hoy?
5) ¿Cuándo cae el día del trabajo?

( ) Se conmemora esta fiesta el primero de Mayo


( ) Hoy es lunes
( ) Calle Cuba, 25
( ) Mi hermana
( ) Estamos as do de Enero

La numeración correcta está en la opción:


(A) 3 – 1 – 5 – 4 – 2
(B) 1 – 3 – 4 – 2 – 5
(C) 5 – 1 – 3 – 2 – 4
(D) 1 – 3 – 5 – 4 – 2
(E) 4 – 2 – 3 – 1 – 5

12- ¡Qué señor distinto! La opción que mejor expresa el sentido de la frase es:
(A) El señor es muy distinguido;
(B) El señor se iguala à los demás;
(C) El señor tiene buenos modales;
(D) El señor se porta como un caballero;
(E) El señor no es igual a los demás caballeros.

13- En el comedor no se usan:


(A) mesa y silla;
(B) sábanas y cobijas;
(C) cuchillo y tenedor;
(D) mantel y servilleta;
(E) cuchara y cucharita.

14- En la cocina no se usa


(A) olla;
(B) fogón;
(C) panal;
(D) sartén;
(E) tenedor.

15- El calor que engalana a la naturaleza es el


(A) gris;
(B) rojo;
(C) verde;
(D) blanco;
(E) amarillo.

73
1103362 E-book gerado especialmente para MARCUS ICARO CRUZ DA SILVA
16- Cuando uno se va a bañar usa
(A) peine;
(B) jabón
(C) funda;
(D) navaja;
(E) mantel.

17- La fórmula de cortesía para despedirse es:


(A) ¡Permiso!
(B) Le presento mis saludos.
(C) ¡Mucho gusto, señorita!
(D) ¿Se conocen hace mucho?
(E) ¡Hasta la vista, señorita!

18- Solo no es animal salvaje


(A) oso;
(B) león;
(C) perro;
(D) tigre;
(E) zorro.
19- Cuando uno va a viajar al exterior, él prevíamente:
(A) zapatos;
(B) guantes;
(C) lápices;
(D) sombrero;
(E) pasaporte.

Millonarios

Tómame de la mano. Vámonos a la lluvia,


descalzos y ligeros de ropa, sin paraguas,
con el cabello al viento y el cuerpo a la caricia
oblicua, refrescante y menuda del agua.

Querían los vecinos! Puesto que somos jóvenes


y los dos amamos y nos gusta la lluvia,
vamos a ser felices con el gozo sencillo
de un casal de gorriones que en la vía se arrulla

Más al lá están los campos y el camino de acacias


y la quinta suntuosa de aquel pobre señor
millonario y obeso que con todos sus oros,

no podría comprarnos ni un grama del tesoro


inefable y supremo que nos ha dado Dios:
Ser flexibles, ser jóvenes, estar llenos de

Juana de lbarbourou – Melo


(Uruguay), 1895
20- Tómame de la mano...
La idea central contenida en esta frase e
(A) lejanía;
(B) altruismo;
(C) sumisión;
(D) imposición;
(E) compañerismo.

21- ... descalzos y ligeros de ropa, sin paraguas....


Este verso contiene la idea de:

74
1103362 E-book gerado especialmente para MARCUS ICARO CRUZ DA SILVA
A) vanidad;
B) seriedad;
C) ansiedad;
D) sencillez;
E) ostentación.

22- Querían los vecinos!


La palabra subrayada indica que los vecinos
A) se burlan de ellos;
B) reprochan sus actos;
C) miran a hurtadillas;
D) envídian a la pareja;
E) miran con curiosidad.

23- Los gorriones son


A) peces;
B) équidos;
C) pájaros;
D) insectos;
E) reptiles.

24- ... pobre señor millonario.


El sentimiento que el señor inspira a la autora es:
A) asco;
B) envidia;
C) repulsa;
D) ternura;
E) compasión.

25- Ser flexibles, ser jóvenes, estar llenos de amor.


Estas bienes son considerados
A) regalo insuperable de Dios;
B) atributos de quien es pobre;
C) tesoro que cualquiera compra;
D) característicos de los millonarios;
E) riquezas materiales que nadie compra.

26- Todavía no he visto la quinta.


La opción que sustituye el término subrayado es:
A) ya;
B) aún;
C) ahora;
D) además;
E) por supuesto.

27- Solo no están en el diminutivo las palabras de la opción


A) amarillo - ardilla;
B) pechuelo - jovenzuelo;
C) avocilla - hermanita;
D) ramillo - corazoncito;
E) pimpollos - muchachito.

28- Solo no es un cuadrúpede:


(A) vaca;
(B) cebú;
(C) búfalo;
(D) bisonte;
(E) culebra.

75
1103362 E-book gerado especialmente para MARCUS ICARO CRUZ DA SILVA
29- Usted irá al club hoy?
Absolutamente!
Si se oye el dialogo arriba, es correcto afirmar que la contestación es de una persona que:
(A) jamás fue al club;
(B) no irá al club hoy;
(C) prensa en quizás ir al club;
(D) irá al club con toda seguridad;
(E) invita al amigo para ir al club.

30- EI niño está:

(A) mirando por la ventana;


(B) brincando y tirando piedras;
(C) acostado en el piso, leyendo un libro;
(D) tendido en un sofá leyendo una revista;
(E) acostado en el piso, consultando un mapa.

31- Solo no se ve en los dibujos um

(A) libro;
(B) reloj;
(C) anillo;
(D) pescado;
(E) cuaderno.

32- Se puede decir que el hombre va apurado porque:

(A) usa sobrero;


(B) come un cambur
(C) carga un maletín
(D) corre por la calle;
(E) mira hacia el reloj.

33- En el dibujo, se ve un hombre:

76
1103362 E-book gerado especialmente para MARCUS ICARO CRUZ DA SILVA
(A) tomando té;
(B) desayunando;
(C) bailando en una fiesta;
(D) cenando en un restaurante;
(E) charlando con sus familiares.

34- Las personas van a viajar

(A) a las doce;


(B) por la noche;
(C) por la mañana;
(D) temprano por la tarde;
(E) muy temprano por la madrugada.

35- La muchacha está

(A) cocinando;
(B) lavando los vasos;
(C) planchando la ropa;
(D) limpiando los muebles;
(E) cepillando la alfombra.

Las cuestiones siguientes están basadas en el texto

La Chispa

A quien no le gusta el vino


si es argentino de tradición!
... Déle a la Juana un besito...
solo un traguito para la ración!
Que beban agua... las plantas!
Ellas aguantan la mojazón!

Marque A si lo que se afirma confirma el texto.


Marque B si lo que se afirma contraria el texto.

36- Solamente a los hombres argentinos les gusta el.

37- Las mujeres están prohibidas de beber.

38- A los hombres y mujeres en Argentina, les gusta el vino.

39- El autor prefiere el agua al vino.

40- La lechuza es un animal que

77
1103362 E-book gerado especialmente para MARCUS ICARO CRUZ DA SILVA
(A) vive en el acuario;
(B) canta por la mariana;
(C) simboliza la pereza;
(D) habita en los pantanos;
(E) simboliza la filosofía y el saber.

Respuestas: 01-B / 02-E / 03-E / 04-E / 05-E / 06-A / 07-C / 08-D / 09-C / 10-E / 11-C / 12-E / 13-B /
14-C / 15-C / 16-B / 17-E / 18-C / 19-E / 20-E / 21-D / 22-D / 23-C / 24-E / 25-A / 26-B / 27-A / 28-E / 29-
D / 30-C / 31-E / 32-D / 33-D / 34-C / 35-A / 36-B / 37-B / 38-A / 39-B / 40-E

Marque a opción correcta en las cuestiones.

La Adoración De Los Reyes

Desde la puesta del sol se alzaba el cántico de los pastores en torno de las hogueras, y desde la
puesta del sol, guiados por aquella otra luz que apareció inmóvil sobre una colina, caminaban los tres
Santos Reyes.
Jinetes en camellos blancos, iban los tres en la frescura apacible de la noche atravesando el
destierro. Las estrellas fulguran en el cielo, y la pedrería de las coronas reales fulguraba en sus frentes.
Una brisa suave hacía flamear los recamados mantos: El de Gaspar era de púrpura de Corinto: ElI de
Melchor era de púrpura de Tiro: El de Baltasar era de púrpura de Menfis. Esclavos negros, que
caminaban a pie enterrando sus sandalias en la arena, guiaban los camellos con una mano puesta en el
cabezal de cuero escarlata.
Ondulaban sueltos los corvos rendajes, y entre sus flecos de seda temblaban cascabeles de oro. Los
Reyes Magos cabalgaban en fila: Baltasar el Egipcio iba adelante, y su barba luenga, que descendía
por el pecho, era a veces esparcida sobre los hombros... Cuando estuvieron a las puertas de la ciudad,
se arrodillaron los camellos, y los tres Reyes se apearon, y, despojándose las coronas, hicieron oración
sobre las arenas.

Y Baltasar dijo:
- Es llegado el término de nuestra jornada...

Y Melchor dijo:
- Adoremos al que nació Rey de Israel...
Y Gaspar dijo:
- Los ojos le verán, y todo será purificado en nosotros!...

Entonces volvieron a montar en sus camellos y entraron en la ciudad por la Puerta Romana, y,
guiados por la estrella, llegaron al establo donde había nacido el Niño. Allí los esclavos negros, como
eran idólatras, y nada comprendían, llamaron con rudas voces:
Abrid! ... Abrid la puerta a nuestros señores!
Entonces los tres Reyes se inclinaron sobre los arzones y hablaron a sus esclavos. Y su cedió que
los tres Reyes les decían en voz baja:

- ¡Cuidad de despertar al Niño!


Y aquellos esclavos, llenos de temeroso respeto, quedaron mudos, y los camellos, que permanecían
inmóviles ante la puerta, llamaron blandamente con la pezuña, y casi al mismo tiempo aquella puerta de
viejo y oloroso cedro se abrió sin ruido. Un anciano de calva sien y nevada barba asomó en el umbral:
sobre el armiño de cabellera luenga y nazarena temblaba el arco de una aureola... Su túnica era azul y
bordada con estrellas con el cielo de Arabia en las noches serenas, y el manto era rojo, como el mar de
Egipto, y el báculo en que se apoyaba era de oro, florecido en lo alto con tres lirios blancos de plata: Al
verse en su presencia, los tres Reyes inclinaron. El anciano sonrió con el candor de un niño, y,
franqueándoles la entrada, dijo con santa alegría:
- Pasad!

Ramón del Valle


01- " Desde la puesta del sol (...) "
El término subrayado puede ser reemplazado por

78
1103362 E-book gerado especialmente para MARCUS ICARO CRUZ DA SILVA
(A) el ocaso;
(B) la mañana;
(C) el levante;
(D) el mediodía;
(E) la medianoche.

02- "Jinetes en camellos blancos (...)"


Jinete es quien
(A) mercadea con camellos;
(B) desconoce la equitación;
(C) conduce una cabalgadura;
(D) conoce bien los camellos;
(E) cuida de las cabalgaduras.

03- Escarlata es el color


(A) del sol;
(B) del mar;
(C) de la arena;
(D) de la sangre;
(E) de los árboles.

04- (...) alzaba los cánticos (...)


Alzar es lo mismo que
(A) bajar;
(B) cantar;
(C) levantar;
(D) edificar;
(E) construir.

05- (...) hacía flamear (...)


Flamear es lo mismo que
(A) ondear;
(B) quemar;
(C) alargar;
(D) extender;
(E) inflamar.

6- (...) temblaban cascabeles de oro (...)


Cascabel es
(A) un látigo;
(B) un cencerro;
(C) una culebra;
(D) una cascada;
(E) una cubierta.
Para las cuestiones 07 a 11 use es cuadro:
a = báculo;
b = pezuña;
c = establo;
d = hoguera;
e = pedrera.

07- Los pastores, al conducir los animales, usan el ______

08- Las joyas son enriquecidas con _________

09- Para caletarse es común hacerse una ___________

10- Los bueyes y los caballos duermen en el __________

79
1103362 E-book gerado especialmente para MARCUS ICARO CRUZ DA SILVA
11- Los pies de los bueyes, chivos y camellos son terminados por una _________.

12- (...) arrodillándose los camellos (...) Quedó dicho que los camellos
(A) se ponían de pie;
(B) salieron en disparada;
(C) se quedaron inmóviles;
(D) se bajaron hasta el suelo;
(E) se erguieron lo más que pudieron.

13- Los tres Santos Reyes viajaban


(A) por la mañana;
(B) durante el día
(C) sólo al anochecer;
(D) durante la noche;
(E) cuando el sol nacía.

14- Los esclavos negros conducían los camellos poniendo las manos en
(A) la cola;
(B) la cabeza;
(C) el flanco;
(D) las patas:
(E) la corcovada.
15- "Un anciano de calva sien y nevada barba (...)"
"(...) sobre el armiño de su cabellera (...)"
El color sugerido arriba es el
(A) rojo;
(B) azul;
(C) negro;
(D) blanco;
(E) amarillo,

16- Sólo no es correcto afirmar que


(A) Melchor conducía los tres.
(B) Baltasar iba adelante, en el cortejo
(C) El manto de Melchor era de púrpura de Tiro.
(D) Los Reyes llegaron al establo guiados por una estrella.
(E) Gaspar, al llegar a la ciudad, también dijo una oración.

La Fracción Del Pan

Camino de un pueblo distante de Jerusalén e sesenta estadías, avanzaban dos discípulos, en vuelto
por la melancolía de una tarde primaveral:
Tardo el paso, la cabeza caída, turbios de lágrimas los ojos, iban hablando los dos caminantes de su
tristeza y orfandad. Grandes cosas eran acontecidas en Israel. Un varón justo, poderoso en la obra y en
la palabra había vivido entre ellos, embriagándoles con una visión maravillosa. Pero había sido
crucificado y muerto. Y esté era el tercer día después de la muerte En verdad, el Sepulcro estaba vacío.
Pero al Maestro nadie le había visto aún. Y con la vaciedad del sepulcro los discípulos se sentían
todavía más sus discípulos.
Ahora todo parecía un sueño.
Y he aquí que Jesús se encuentra de Pronto entre los dos, y anda el camino en su compañía. Pare
embargados, los ajos no le conocen.
Y Jesús habla a los discípulos, y ellos le contestan, pero siguen sin conocerle.
Les recuerda decir de Moisés y las profecías. Les llama insensatos y tardas de corazón. Y ellos no le
conocen aún.
Mas, arribados al lugar los discípulos, Jesús dió muestras de querer seguir más adelante. Y ellos le
retenían por la fuerza, diciéndole: "Quédate con nosotros porque se ha hecho tarde y ha declinado el
día".
"Y se entró con ellos, Y, estando sentado con ellos a la mesa tomo el pan y lo bendijo, y, después de
partir les daba de él."

80
1103362 E-book gerado especialmente para MARCUS ICARO CRUZ DA SILVA
"Entonces fueran abiertos los ojos de ellos, y le reconocieran..."
Más tarde "contaban cómo le habían reconocido en el partir del pan",
Cómo podía ser esta manera de partir el pan de Jesús, en que alcanzaba a conocerle quien no le
conocía; ni en la presencia, ni en la voz, ni en la palabra, ni en el reproche?
Debía ser como una bendición. ¡Las manos del Maestro dejando una porción en cada mano!

Eugénio DÓrs
17- "(...) y los tres Reyes se apearon, y despojándose las caronas (...)"
Se entiende que los Reyes sólo no demostraron
(A) bondad;
(B) vanidad;
(C) humildad;
(D) respecto;
(E) simplicidad.

18- Los esclavos negros no respectaron al niño de pronto, porque


(A) eran muy malos;
(B) se rebelaron contra los reyes;
(C) no comprendían lo que se pasaba;
(D) cometieron un acto de desobediencia;
(E) eran portadores de un mensaje de guerra.

19- La comitiva de los Reyes fue recibida por:


(A) un niño;
(B) un viejo;
(C) un joven;
(D) una mujer;
(E) un esclavo.

20- " Tardo el paso (...)" "(...) tardos de corazón


Los términos subrayados significan, respectivamente
(A) lento, torpes;
(B) corto, torpes;
(C) pesado, lentos;
(D) perezoso, lentos;
(E) pesado, pequeños.

21- "(...) arribados al lugar (...)"


Es decir que
(A) habían llegado al destino;
(B) estaban lejos del destino;
(C) estaban cerca del destino;
(D) bestaban muy arriba de donde deseaban llegar;
(E) no tenían confianza en la estrada que seguían..

22- "(...) de su tristeza y orfandad'


Orfandad, en el texto significa
(A) una gran pérdida;
(B) una gran soledad;
(C) la pérdida del padre;
(D) la pérdida de la madre;
(E) la pérdida de los progenitores.

23- En el texto queda dicho que la vaciedad del sepulcro


(A) derramó la fe por los demás;
(B) ofuscó la fe de los discípulos;
(C) alargó la fe de los discípulos;
(D) no alteró la fe de ellos en nada;
(E) confirmó de la fe era en sueño apenas.

81
1103362 E-book gerado especialmente para MARCUS ICARO CRUZ DA SILVA
24- No se refiere a los discípulos la opinión:
(A) varón justo;
(B) lardo el paso;
(C) la cabeza caída;
(D) turbios los ojos;
(E) tardos de corazón,

25- "Ahora todo parecía un sueño"


Lo que está dicho quiere decir que:
(A) todo en la vida as un sueño;
(B) el pasado no decía nada a ellos;
(C) los discípulos sólo sabían soñar;
(D) no era fácil acreditar en lo que ocurría;
(E) todo el pasado cayó en el olvido de los discípulos.

26- Los discípulos conocieron al Maestro por:


(A) la voz;
(B) el paso;
(C) el ropaje;
(D) el aspecto;
(E) la actitud.

27- Numere las acciones de los discípulos según el orden en que aparecen en el texto:
a) Oyeron las palabras del Maestro.
b) Vieron a Jesús.
c) Fueron llevados a recordar el pasado.
d) Recibieron un reproche.

(A) 2, 1, 3, 4
(B) 2, 3, 1, 4
(C) 3, 2, 1, 4
(D) 4, 3, 1, 2
(E) 4, 2, 3, 1

28- El Maestro de escuela sólo no usa en el aula:


(A) tiza;
(B) lápiz;
(C) pizarra;
(D) borrador;
(E) periódico.

29- El alumno solo no lleva


(A) regla;
(B) libro;
(C) pupitre;
(D) estuche;
(E) cuaderno,

30- La cocinera solo no usa


(A) fogón;
(B) sartén;
(C) cuchara;
(D) plancha;
(E) cuchillo.

31- El sastre sólo no usa


(A) tela;
(B) aguja;
(C) línea;

82
1103362 E-book gerado especialmente para MARCUS ICARO CRUZ DA SILVA
(D) tijera;
(E) cuchillo.

32- solo no es parte de una casa


(A) techo;
(B) pared;
(C) tejado;
(D) cadena;
(E) ventana.

33- solo no es parte del cuerpo humano


(A) cola;
(B) cuello;
(C) rodilla;
(D) mejilla;
(E) tobillo.

Complete las cuestiones 34 a 37 / según el cuadro.

A - lunes
B - martes
C – jueves
D - viernes
34- El día que precede al sábado es___________

35- El día de trabajo en la semana es___________

36- El ___________es el segundo día de trabajo semanal.

37- A _________ sigue viernes.

Use el cuadro para las cuestiones 38 a 40.

A - sombrero
C - falda
B - guantes

38- En la cabeza se lleva __________.

39- Son las mujeres que usan___________

40- Cuando hace frío se cubren las manos con_________

Respuestas: 01-A / 02-C / 03-D / 04-C / 05-A / 06-B / 07-A / 08-E / 09-D / 10-C / 11-B / 12-D / 13-D /
14-B / 15-D / 16-A / 17-B / 18-C / 19-B / 20-A / 21-A / 22-A / 23-C / 24-A / 25-D / 26-E / 27-A / 20-C / 29-
C / 30-D / 31-E / 32-D / 33-A / 34-D / 35-A / 36- B / 37-G / 38-A / 39-C / 40-B.

Hacienda acelera los embargos preventivos para cobrar las deudas

La recesión somete a las arcas del Estado a un doble perjuicio: los ingresos caen por la atonía de la
actividad y, a la vez, los incumplimientos tributarios se disparan. El Ministerio de Economía y Hacienda
de España admite que esa deuda está creciendo y se propone extremar las medidas para intentar
cobrarla. La necesidad es acuciante: en un año en que el Gobierno prevé ingresar como mínimo 40.000
millones de euros menos de lo que proyectó inicialmente, cada euro recaudado cuenta.
Entre enero y agosto de este año se han adoptado medidas cautelares por valor de 753 millones,
cantidad 24% superior a la de todo el ejercicio 2008, según datos de la Agencia Tributaria (AT), que
tiene 37.000 millones de deuda pendiente, con datos a 31 de julio. De esa cantidad, una deuda de algo
más de 12.000 millones cuenta com algún tipo de garantía para ser cobrada. La más incierta es la que
está en vía ejecutiva (Hacienda intenta ya adoptar medidas como el embargo para cobrarla) y la que fi

83
1103362 E-book gerado especialmente para MARCUS ICARO CRUZ DA SILVA
gura en periodo de notifi cación (la fase previa). Ambas partidas suman casi 16.000 millones hasta fi nal
de julio. Sólo com recuperar esas cantidades, el fi sco ingresaría más dinero que con la subida de
impuestos aprobada para 2010, que se acerca a 11.000 millones.

(Elpais.es - Texto adaptado)

01- En el texto se dice que los cofres públicos españoles:


(A) sufren pérdidas.
(B) son independientes del sistema tributario.
(C) se destinan al pago de deudas del estado.
(D) están inmunes a la depresión.
(E) recibirán recursos de 40 mil millones de euros.

02- De acuerdo con el texto, el monto de los incumplimientos tributarios:


(A) es similar al del ejercicio anterior.
(B) está en vía ejecutiva.
(C) tiene garantías de cobrarse.
(D) es mayor que el ingreso esperado con el alza de impuestos.
(E) ha menguado en lo que va de año.

03- Según el texto, la baja de los ingresos:


(A) se deriva de las medidas cautelares.
(B) es fruto de una fi scalización inefi ciente.
(C) alcanza los 37 mil millones de euros.
(D) es semejante a la del año fi scal 2008.
(E) se debe a la debilidad de la actividad.

Gobierno no se muestra seguro

Ni el propio Gobierno está seguro de que una rebaja em las tasas de interés –actualmente en 4% –
pueda contribuir a frenar la acentuada revaluación (más del 18,8% em los últimos meses) que afecta a
varios sectores de la economía. Los empresarios achacan al actual nivel de las tasas de interés parte
de la galopante apreciación del peso que le resta competitividad a las exportaciones colombianas.
Consideran que esta diferencia de intereses está propiciando el ingreso de los llamados capitales
golondrinas (de poca duración en la economía) que están debilitando el dólar. El ministro de Hacienda,
Óscar Iván Zuluaga no se mostro muy seguro de que el pretendido recorte sea una buena decisión en
los actuales momentos en que la infl ación sigue cediendo. Admitió, sin embargo, que con las medidas
adoptadas “hemos evitado que se revalúe “ el dólar.

(Elespectador.com)

04- Según el texto, en materia de tasas de interés, los empresarios colombianos:


(A) abogan por la apreciación.
(B) muestran inseguridad.
(C) discrepan del gobierno.
(D) relacionan los tipos con la infl ación.
(E) están conformes.

05- En el texto se dice que las tasas de interés que se practican actualmente:
(A) elevan la cotización del dólar.
(B) perjudican las exportaciones.
(C) deben mantenerse.
(D) impiden la apreciación del peso.
(E) acentúan la infl ación.

84
1103362 E-book gerado especialmente para MARCUS ICARO CRUZ DA SILVA
Comercio Chile-China

De acuerdo a un informe de Aduanas, China se consolidó como el principal destino de las


exportaciones chilenas com embarques por US$ 8.257 millones –en un intercambio bilateral de US$
11.881 millones– y el único de los sócios comerciales hacia donde nuestros envíos crecieron (6,3%)
durante los primeros nueve meses de este año (los envios totales menguaron un 35%). El incremento
registrado em los embarques al gigante asiático fue muy superior a las caídas entre 20% y 68%
anotadas en el resto de los países con los que Chile mantiene intercambio comercial, incluso socios tan
importantes como la Unión Europea, Aladi y Nafta, destinos a los que Chile exportó productos por um
monto menor a US$ 6.700 millones.
Las exportaciones chilenas a Estados Unidos cayeron um 32% al pasar de US$ 6.625 millones a
US$ 4.495 millones; mientras los envíos a Japón bajaron 45% de US$ 3.493 millones a US$ 1.501
millones. En relación a Europa, el informe de Aduanas mostró que la cantidad embarcada también
disminuyó en los primeros nueve meses del año (49%) respecto del mismo lapso de 2008, debido a la
disminución de los envíos a Italia y Holanda.

(Diario Financiero - texto adaptado)

06- En el texto se dice que las exportaciones chilenas:


(A) se han reducido a US$ 6.700 millones.
(B) rebasan las expectativas para el período.
(C) tuvieron un alza de 35%.
(D) se han diversifi cado.
(E) sólo crecieron hacia un destino.

07- De acuerdo con el texto, en el informe de Aduanas se dice que:


(A) el intercambio con China creció en ambas direcciones.
(B) los envíos a Estados Unidos se redujeron al 32%.
(C) Europa es la que compró menos productos chilenos.
(D) las exportaciones totales de Chile se contrajeron.
(E) el comercio exterior chileno da señales de recuperación.

‘La calle’ está tranquila

Zapatero ha declarado a Newsweek que para comprovar que España no se hunde “sólo hay que salir
a la calle”. Subraya así la paradoja de que no exista confl ictividad social con cuatro millones de
parados. Tiene razón em parte, aunque quizá debería preguntarse por qué la calle está tranquila y
entonces su análisis no podría ser tan complaciente. Según publicamos hoy, la economia sumergida ha
aumentado el 30% en el verano que ahora finaliza. Es el cálculo de los inspectores de Trabajo, que han
estimado en 320 millones de euros la recaudación en multas para 2009, una cifra récord. Así pues, en el
auge de la economía sumergida – de la que un país no puede sentirse orgulloso, sino todo lo contrario –
está la explicación de la calma de la calle, puesto que los parados optan por las «chapuzas» para ir
tirando. Aquí estaría la explicación de otro dato que Zapatero comentó, asombrado, ante el Comité
Federal: sólo 28.000 parados sin ingresos han solicitado los 420 euros, cuando los posibles benefi
ciarios de esta medida son en torno a um millón. Como el subsidio lleva aparejada la obligación de
asistir a cursos de formación, tal vez la mayoría de estos desempleados sin ingresos en realidad sí
cobran por algún trabajo, aunque no declaran, ni cotizan. Ello quiere decir que la política social d e
Zapatero se vuelve contra él y contra los que él dice proteger. El empleo que se crea es clandestino por
las rigideces del mercado laboral y al subir los impuestos aumentará la economía del dinero negro.

(El mundo)

08- En el texto se dice que en España no hay confl ictividad social:


(A) por la solidez institucional.
(B) gracias a las prestaciones públicas asistenciales.
(C) a pesar del desempleo.
(D) debido a la efi cacia de la política laboral del gobierno.
(E) pese al aumento de la recaudación por multas.

85
1103362 E-book gerado especialmente para MARCUS ICARO CRUZ DA SILVA
09- De acuerdo con el texto, las actividades económicas marginales en España:
(A) disminuirán si suben los impuestos.
(B) han aumentado.
(C) elevarán las cotizaciones al tesoro.
(D) garantizan un aumento de los ingresos públicos.
(E) ocupan a cerca de un millón de parados.

10- Dentro del texto, la palabra «chapuzas» signifi ca:


(A) pequeñas contravenciones.
(B) obras sin arte ni esmero.
(C) contratos temporales.
(D) cursos de reciclaje profesional.
(E) subsidios fraudulentos.

Los 17 mil chilenos con mayores ingresos entregan el 35% del impuesto a la renta

En Chile, 1.329.297 personas pagan impuestos a la renta, ya sea bajo la modalidad de segunda
categoría o de global complementario. A estas se suman otras 6.346.693 que si bien son formalmente
contribuyentes para los registros del Servicio de Impuestos Internos (SII), en la práctica están en el
primer tramo impositivo, que es cero. De este 1,32 millón de chilenos, el 1,27% está en el tramo más
alto de impuestos – con una tasa marginal del 40% –, que es para quienes tienen ingresos de cerca de
$5,5 millones de pesos mensuales o más. En conjunto, pagan más tributos al año que los bancos o todo
el sector comercio. De acuerdo com los datos del SII, este grupo es el responsable del 35,1% de la
recaudación por impuesto a la renta; es decir, US$ 908 millones del total de US$ 2.589 millones que los
contribuyentes chilenos aportan por este concepto.
Para el socio principal de Ernst & Young, Cristián Levefre, la cifra demuestra que el grueso del aporte
de impuestos lo entregan quienes más dinero reciben. Según él, existe uma discriminación entre los
contribuyentes de los tramos altos. Los independientes – muchos de ellos empresarios – tienen
opciones de desarrollar una planifi cación tributaria para reducir el pago y los dependientes no las
tienen. A su juicio, Chile debería impulsar benefi cios para este segmento, como los que hay en
economías más desarrolladas, como EE.UU., Inglaterra y España, donde se deduce por pago de
colegios e intereses de créditos hipotecarios.

(El Mercurio)

11- Según el texto, el cobro del impuesto sobre la renta em Chile:


(A) se concentra en los mayores ingresos.
(B) incluye a la mayoría de la población.
(C) alcanza al 35% de los trabadores.
(D) castiga a los autónomos.
(E) es menor entre los empleados bancarios.

12- En el texto se aconseja:


(A) aumentar los tributos al sector bancario.
(B) disminuir los intervalos impositivos.
(C) introducir deducciones por determinados pagos.
(D) reducir la tasa marginal de los tramos de impuestos.
(E) gravar más a los independientes.

13- En el contexto del texto, “a su juicio” conserva su sentido al sustituirse por:


(A) a su aire.
(B) en su prejuicio.
(C) a lo suyo.
(D) en su opinión.
(E) a regañadientes.

14- De acuerdo con el texto, los contribuyentes dependientes chilenos:


(A) se benefi cian con desgravaciones.
(B) están concentrados en los tramos impositivos intermedios.

86
1103362 E-book gerado especialmente para MARCUS ICARO CRUZ DA SILVA
(C) cotizan más que los bancos.
(D) planifi can el pago de sus tributos.
(E) sufren discriminación.

Necesidad de préstamos

Los gobiernos latinoamericanos necesitarán entre 350 y 400 billones de dólares en préstamos en
2010 para reactivar sus economías tras la crisis fi nanciera global, según la vicepresidenta del Banco
Mundial para América Latina y el Caribe, Pamela Fox. La obtención de prestamos no será fácil, incluso
para proyectos de inversión, advirtió la funcionaria, debido a las difi cultades para conseguir fondos que
están restringidos por la enorme demanda internacional de las naciones desarrolladas para sus
paquetes de estímulo.
“A raíz de la crisis el papel del Estado ha crecido a niveles que eran inimaginables años atrás”, dijo
Fox, pero el Estado dispone de menos recursos, de forma tal que “este momento exige más de los
ciudadanos que pagan impuestos, especialmente de aquéllos con más ingresos, que deberían tener
que afrontar una mayor carga tributaria”, indicó. La funcionaria destacó que según la Organización de
Cooperación Económica y Desarrollo menos de un 4% de los ingresos públicos en America Latina
provienen de pagos de impuestos personales, comparado con un 27% en las naciones industrializadas.

(El Nacional)

15- Según el texto, la reactivación económica de América Latina tras la crisis fi nanciera global:
(A) supondrá una mayor regulación de la intervención estatal.
(B) demandará entre 350 y 400 mil millones de dólares.
(C) recabará más de toda la ciudadanía.
(D) exigirá entre 350 y 400 millones de millones de dólares.
(E) incidirá en los paquetes de estímulo de los países desarrollados.

Respuestas: 01-A / 02-D / 03-E / 04-C / 05-B / 06-E / 07-D / 08-C / 09-B / 10-B / 11-A / 12-C / 13-D /
14-E / 15-D

VERBO

Os verbos são palavras que atuam como núcleo da oração. Em espanhol, os verbos são
classificados em três grupos, conforme a terminação:
1) Primeira conjugação: verbos cujo infinitivo termina em -ar, como: cantar, tomar, hablar.
2) Segunda conjugação: verbos cujo infinitivo termina em -er, como: beber, comer, poseer.
3) Terceira conjugação: verbos cujo infinitivo termina em -ir, como: vivir, asistir, permitir.

Cada uma das conjugações (-ar, -er, -ir) pode ser regular ou irregular.
Os modos verbais são três: indicativo, subjuntivo e imperativo.

Indicativo: Conjuga-se o modo indicativo nos seguintes tempos:

Presente
HABLAR COMER VIVIR
Hablo Como Vivo
Hablas Comes Vives
Habla Come Vive
Hablamos Comemos Vivimos
Habláis Coméis Vivís
Hablan Comen Viven
Usa-se para:

- Referir-se a hábitos ou costumes. Ex.: Me levanto todos los días a las seis de la mañana / Levanto-
me todos os dias às seis da manhã.
- Falar de ações futuras. Ex.: Mañana tengo una reunión muy importante / Amanhã tenho uma
reunião muito importante.

87
1103362 E-book gerado especialmente para MARCUS ICARO CRUZ DA SILVA
- Falar de acontecimentos passados. Ex.: El pintor Salvador Dalí nace en Figueres el 11 de mayo de
1904 / O pintor Salvador Dalí nasce em Figueres em 11 de maio de 1904.
- Dar instruções. Ex.: Para hablar por un teléfono público, primero colocas la tarjeta y luego marcas el
número. Para falar em um telefone público, primeiro você coloca o cartão e depois disca os números.

Pretérito Imperfecto
HABLAR COMER VIVIR
Hablaba Comía Vivía
Hablabas Comías Vivías
Hablaba Comía Vivía
Hablábamos Comíamos Vivíamos
Hablabais Comíais Vivíais
Hablaban Comían Vivían
Usa-se para:
- Referir-se a ações passadas, de caráter duradouro ou repetitivo, que não têm um fim determinado
no tempo. Ex.: Cuando era niña iba al colegio por la tarde. / Quando era criança, ia ao colégio à tarde.
- Descrever no passado. Ex.: En la época de mis abuelos la ciudad era tranquila y los días pasaban
lentamente. / Na época de meus avós a cidade era tranquila e os dias passavam lentamente.

Pretérito Perfecto
HABLAR COMER VIVIR
Hablé Comí Viví
Hablaste Comiste Viviste
Habló Comió Vivió
Hablamos Comimos Vivimos
Hablateis Comisteis Viviteis
Hablaron Comieron Vivieron

Chamado também de indefinido, usa-se para:


- Referir-se a ações concluídas em um momento determinado no passado. Ex.: El papa Juan Pablo II
falleció el año 2005 / O papa João Paulo II faleceu em 2005.
- Referir-se a ações únicas no passado. Ex.: Ayer fui al cine / Ontem fui ao cinema.

Futuro Simple
HABLAR COMER VIVIR
Hablaré Comeré Viviré
Hablarás Comerás Vivirás
Hablará Comerá Vivirá
Hablaremos Comeremos Viviremos
Hablaréis Comeréis Viviréis
Hablarán Comerán Vivirán

Usa-se para:
- Falar de ações futuras. Ex.: La próxima semana saldré de vacaciones / Na próxima semana sairei
de férias.
- Falar de planos. Ex.: Cuando termine de estudiar buscaré empleo en una gran empresa / Quando
terminar de estudar procurarei emprego em uma grande empresa.

Condicional Simple
HABLAR COMER VIVIR
Hablaría Comería Viviría
Hablarías Comerías Vivirías
Hablaría Comería Viviría
Hablaríamos Comeríamos Viviríamos
Hablarías Comeríais Viviríais
Hablarían Comerían Vivirían

88
1103362 E-book gerado especialmente para MARCUS ICARO CRUZ DA SILVA
O condicional expressa um fato irreal, mas possível ou provável de realizar no futuro. Corresponde
em português ao futuro do pretérito. Ex.: Me iría contigo al cine, pero no tengo tiempo./ Eu iría contigo
ao cinema, mas não tenho tempo.

Tiempos compuestos de Indicativo:

Os tempos compostos tem uma relação significativa com os tempos simples do indicativo dos quais
derivam.

Compuesto Pretérito Perfecto

Ação iniciada no passado e que perdura até o presente, ou seja, quando o espaço de tempo
expresso na frase ainda não está acabado. Conjuga-se o verbo HABER no Presente do
Indicativo + o Particípio do verbo principal.

Ejemplo: Este año casi no he viajado. (o ano não acabou) (Este ano quase não viajei.)

HABLAR TEMER PARTIR


he hablado he temido he partido
has hablado has temido has partido
ha hablado ha temido ha partido
hemos hablado hemos temido hemos partido
habéis hablado habéis temido habéis partido
han hablado han temido han partido

¡Atención!

Pretérito Indefinido Simple X Pretérito Perfecto Compuesto


Yo amé. (Eu amei.)
Yo he amado. (Eu amei.)

Se ao traduzirmos as frases acima encontramos a mesma tradução, qual seria então a real diferença
entre esses dois pretéritos? O Pretérito Indefinido Simple indica uma ação passada e acabada. O
Pretérito Perfecto Compuesto indica uma ação passada que guarda relação com o tempo atual. Sendo
assim:
Yo amé. (Eu amei e não amo mais.)
Yo he amado. (Eu amei e ainda amo.)

Pretérito Pluscuamperfecto

Indica uma ação passada e terminada, anterior a outra, também passada. Se conjuga o
verbo HABER no Pretérito Imperfeito + o Particípio do verbo principal.

Ejemplo: Había salido con ellos. (Havia/tinha saído com eles.)

HABLAR TEMER PARTIR


Había hablado Había temido Había partido
Habías hablado Habías temido Habías partido
Había hablado Había temido Había partido
Habíamos hablado Habíamos temido Habíamos partido
Habíais hablado Habíais temido Habíais partido
Habían hablado Habían temido Habían partido

89
1103362 E-book gerado especialmente para MARCUS ICARO CRUZ DA SILVA
Pretérito Anterior

Usa-se muito pouco na língua escrita ou falada. Denota uma ação passada anterior, mas imediata no
tempo. Este tempo foi substituído pelo pretérito indefinido ou pelo pretérito pluscuamperfecto em quase
todos os usos.

Ex.: Apenas hubo amanecido, se fue. (Apenas amanheceu, se foi)

HABLAR TEMER PARTIR


Hube hablado Hube temido Hube partido
Hubiste hablado Hubiste temido Hubiste partido
Hubo hablado Hubo temido Hubo partido
Hubimos hablado Hubimos temido Hubimos partido
Hubisteis hablado Hubisteis temido Hubisteis partido
Hubieron hablado Hubieron temido Hubieron partido

Futuro Perfecto Compuesto

Indica um fato futuro, acabado, anterior a outro, também futuro. Conjuga-se o


verbo HABER no Futuro + o Particípio do verbo principal.

Ejemplo: Para cuando nos mudemos ya habrán terminado las obras. (Quando nos mudarmos já
terão terminado as obras.)

HABLAR TEMER PARTIR


Habré hablado Habré temido Habré partido
Habrás hablado Habrás temido Habrás partido
Habrá hablado Habrá temido Habrá partido
Habremos Habremos Habremos
hablado temido partido
Habréis hablado Habréis temido Habréis partido
Habrán hablado Habrán temido Habrán partido

Condicional Compuesto

Indica uma ação futura a respeito de um momento do passado, mas anterior a outro momento que
aparece na oração. Ex.: Me dijo que cuando yo llegara a casa, ya habría enviado el paquete( a ação
habría enviado é futuro com relação a dijo, mas anterior a llegara)
Pode indicar também suposição ou probalidade no pasado. Ex.: En aquel tiempo, él ya habría
cumplido treinta años. (Naquele tempo, ele já haveria completado trinta años)

HABLAR TEMER PARTIR


Habría hablado Habría temido Habría partido
Habrías hablado Habrías temido Habrías partido
Habría hablado Habría temido Habría partido
Habríamos Habríamos Habríamos
hablado temido partido
Habríais Habríais temido Habríais partido
hablado
Habrían hablado Habrían temido Habrían partido

90
1103362 E-book gerado especialmente para MARCUS ICARO CRUZ DA SILVA
Modo Subjuntivo – Presente

HABLAR TEMER PARTIR


hable tema parta
hables temas partas
hable tema parta
hablemos temamos partamos
habléis temáis partáis
hablen teman partan

As irregularidades no Presente do Modo Subjuntivo são as mesmas já apresentadas no Presente do


Modo Indicativo.

Pretérito Imperfecto

Indica uma ação hipotética, que pode ou não ocorrer.


Ejemplo: Si quisiera acompañarte hasta Madrid, viajaría. (Se quisesse te acompanhar até Madrid,
viajaria.)

HABLAR TEMER PARTIR


hablara temiera partiera
hablaras temieras partieras
hablara temiera partiera
habláramos temiéramos partiéramos
hablarais temierais partierais
hablaran Temieran partieran

2ª FORMA HABLAR TEMER PARTIR


hablase temiese partiese
hablases temieses partieses
hablase temiese partiese
hablásemos temiésemos partiésemos
hablaseis temieseis partieseis
hablasen Temiesen partiesen

Pretérito Perfecto Compuesto

Indica um fato duvidoso, hipotético, que pode ter se realizado no passado. Desejo de que algo já
tenha ocorrido. Conjuga-se o verbo HABER no Presente do Subjuntivo + o Particípio do verbo
principal.

Ejemplos:

Que tú hayas esperado bastante. (Que tu tenhas esperado bastante.)


Espero que ellos hayan llegado temprano. (Espero que eles tenham chegado cedo.)

HABLAR TEMER PARTIR


haya hablado haya temido haya partido
hayas hablado hayas temido hayas partido
haya hablado haya temido haya partido
hayamos hablad hayamos temid hayamos partid
o o o
hayáis hablado hayáis temido hayáis partido
hayan hablado hayan temido hayan partido

91
1103362 E-book gerado especialmente para MARCUS ICARO CRUZ DA SILVA
Pretérito Pluscuamperfecto

Refere-se a um passado que não se realizou.

Ejemplo: Si hubiera tenido tiempo habría salido. (Se houvesse tido tempo haveria saído.)

1ª Forma
HABLAR TEMER PARTIR
hubiera hablado hubiera temido hubiera partido
hubieras hablado hubieras temido hubieras partido
hubiera hablado hubiera temido hubiera partido
hubiéramos hablad hubiéramos temid hubiéramos partid
o o o
hubierais hablado hubierais temido hubierais partido
hubieran hablado hubieran temido hubieran partido
2ª Forma

HABLAR TEMER PARTIR


hubiese hablado hubiese temido hubiese partido
hubieses hablado hubieses temido hubieses partido
hubiese hablado hubiese temido hubiese partido
hubiésemos hablado hubiésemos temido hubiésemos partido
hubieseis hablado hubieseis temido hubieseis partido
hubiesen hablado hubiesen temido hubiesen partido
Modo Imperativo

Indica órdenes, mandatos, ruegos y deseos. O modo imperativo só tem duas formas próprias: a
segunda pessoa do singular e a segunda pessoa do plural, em afirmativo. As outras pessoas (em
afirmativo ou em negativo) se conjugam no Presente do Subjuntivo.

Imperativo Afirmativo

HABLAR TEMER PARTIR


Tú habla teme parte
Él hable tema parta
Nosotros hablemos temamos partamos
Vosotros hablad temed partid
Ellos hablen teman Partan
Imperativo Negativo

Es el presente de subjuntivo en forma negativa.

HABLAR TEMER PARTIR


Tú No hables No temas No partas
Él / usted No hable No tema No parta
Nosotros No No No
Vosotros hablemos temamos partamos
Ellos / ustedes No habléis No temáis No partáis
No hablen No teman No partan

92
1103362 E-book gerado especialmente para MARCUS ICARO CRUZ DA SILVA
2ª p.singular - 2ª p. plural -
Tú Vosotros
Decir Di Decid
Hacer Haz Haced
Ir Ve Id
Poner Pon Poned
Salir Sal Salid
Ser Sé Sed
Tener Ten Tened
Venir Ven Venid
Ejemplos:

Imperativo Afirmativo Negativo


Tú ¡Di la verdad! ¡No digas la verdad!
Él / usted ¡Diga la ¡verdad! ¡No diga la verdad!
Nosotros ¡Digamos la verdad! ¡No digamos la verdad!
Vosotros ¡Decid la verdad! ¡No digáis la verdad!
Ellos / ustedes ¡Digan la verdad! ¡No digan la verdad!

Cuestiones

01- Completa las frases usando el presente de indicativo:


(A) Tú____ el café en la cocina. (preparar)
(B) Él___ por Madrid a pie. (pasear)
(C) Vosotros____ la lección. (estudiar)
(D) El profesor___ al alumno. (contestar)
(E) Nosotros____ en el restaurante. (comer)
(F) Los chicos____ diseños en la televisión. (asistir)
(G) Yo___ hoy por la tarde. (partir)

02- Completa las frases usando el pretérito indefinido:


(A) Ella___ un coche nuevo. (comprar)
(B) Tú___ muy bien. (cantar)
(C) Nosotros___ paz en el mundo. (desear)
(D) Él___del dinero de su padre. (depender)
(E) Los conductores___ por la carretera. (correr)
(F) Ellos____ sus regalos. (abrir )
(G) La historia____ mi corazón. (partir)

03- Completa las frases usando el futuro simple:


(A) Mañana__________ en casa de mis suegros. (estar)
(B) Pepe__________ al piso inferior. (bajar)
(C) Vosotros__________ por teléfono al director. (llamar)
(D) Ellos__________ el periódico de la mañana. (leer)
(E) Yo__________ mucho vino blanco. (beber)
(F) Ustedes__________ a casa de sus amigos. (ir)
(G) Tú__________ una casa en la ciudad. (adquirir)

04- Completa las frases usando el presente de subjuntivo:


(A) No creo que Julián me___ esta tarde. (llamar)
(B) Quizá____ la casa para esos señores. (tú- vender)
(C) Ojalá el jefe me____ salir más temprano. (permitir)
(D) Es bueno que____ el piso este mes. (vosotros-alquilar)
(E) No es seguro que___ a Luisa. (nosotros-visitar)
(F) Posiblemente___ mañana por la mañana. (yo- viajar)
(G) Quiero que____ el recado. (tú-anotar)

93
1103362 E-book gerado especialmente para MARCUS ICARO CRUZ DA SILVA
05- Completa las frases usando el imperativo afirmativo:
(A) María, ____ la ventana, por favor. (cerrar)
(B) Niños, ____ los ejercicios lo más pronto posible. (hacer)
(C) Señora, ____ esta ficha con sus datos y enseguida_____ La tasa de inscripción en el tercer piso.
(rellenar- pagar)
(D) Hombre, ___ un poco de refresco. (beber)
(E) Señoras y señores, ___ con atención los detalles de esta catedral. (observar)
(F) Cariño, ____ conmigo. (quedarse)
(G) Amigos, esta fiesta la hice para vosotros. ___(comer), ___ (beber) y __ (bailar), eso es lo único
que tenéis que hacer.

06- Completa las frases usando el imperativo negativo:


(A) Javier, ___a los niños. (despertar)
(B) Hijo, ____ para mañana lo que puedes hacer hoy. (dejar)
(C) Presidente, ___ de los hambrientos. Ellos lo han elegido. (olvidarse)
(D) Ramón, ____ la tele ahora. Estamos estudiando. (poner)
(E) Niños, _____ tarde. (llegar)
(F) Señora, _____ la basura en el césped. (echar)
(G) Señoras y señores, ____ si van a conducir. (beber)

Respuestas:

1- 2- 3-
a- preparas a- compró a- estaré
b- pasea b- cantaste b- bajará
c- estudiáis c- deseamos c- llamasteis
d- contesta d- dependió d- leerán
e-comemos e-corrieron e- beberé
f- asisten f- abrieron f- irán
g- parto g- partió g- adquirirás
4- 5- 6-
a- llame a- cierra a- no despiertes
b- vendas b- haced/hagan b- no dejes
c- permita c- rellene/pague c- no se olvide
d- alquiléis d- bebe d- no pongas
e- visitemos e- observen e- no lleguéis/lleguen
f- viaje f- quédate f- no eche
g-anotes g- comed/bebed/bailad g- no beban

EL USO DE MUY Y MUCHO

1- Se usa la palabra mucho antes o después de los verbos.

Ejemplos:
El niño estudia mucho.
Se usa también la palabra mucho con sustantivos.

Ejemplos:
Yo tengo muchos libros.

2- Se usa la palabra muy antes de:


a) adjetivos: muy alto, muy fácil, muy difícil, muy malo, etc.
b) adverbios: muy bien, muy mal, muy tarde, muy cerca, etc.

94
1103362 E-book gerado especialmente para MARCUS ICARO CRUZ DA SILVA
ATENCIÓN:

Pero se usa mucho antes de:

a) cuatro adjetivos: (mejor, peor, mayor y menor)


mucho mejor, mucho peor, mucho mayor, mucho menor

b) cuatro adverbios: (más, menos, antes y después)


mucho más, mucho menos, mucho antes, mucho después

Cuestiones

I) Completa las frases con muy o mucho:

1- No camines................rápido.
2- Tú hablas...................
3- Hoy papá está .................mejor que ayer.
4- Tu casa está................cerca de aquí.
5- Ahora Teresa nos escribe................más.
6- Tienes los ojos................hermosos.
7- Esto ahora está................peor que ayer.
8- Estudias................poco para los exámenes.
9- Mi escuela queda................lejos.
10- Fernando come................menos que tú.
11- Carlos es................aplicado en el colegio.
12- Necesito una cosa................menor.
13- Tu casa es................mayor que la mía.
14- Ellos llegaron................antes que nosotros.
15- Este edificio es................alto.
16- Nosotros llegamos................después.
17- Hoy me he levantado................temprano.
18- Él se acostó................tarde.
19- Tengo un piso................amplio.
20- Mi familia me ayudó.................
21- Sé que este coche es................caro, pero es............más cómodo que los otros.
22- Juana está............cansada. Ella trabajó................hoy.
23- Como todos los perezosos, Enrique siempre duerme................
24- No creo que seas.........menos dedicado al estudio que los otros alumnos, pero, estoy seguro de
que eres...........poco dedicado a las tareas para el hogar.
25- Aunque Rodrigo no tenga............pelo, es considerado por las mujeres un hombre.....atrayente.
26- Roberto y yo ejercemos funciones iguales. Sin embargo, yo gano...............menos que él.
27- Es................importante que los niños beban................leche natural entera.
28- Hace................años que me dedico con ahínco a una tarea................linda.
29- .........mujeres todavía no están.........seguras de su derecho a la igualdad con los hombres.
30- Andrea no es una chica..........bella, pero tiene............clase.
31- Reconozco que tu redacción está......buena, pero también debo admitir que la mía
está..........mejor.
32- Felizmente los médicos actuaron com......prontitud y evitaron que Rafael perdiera........sangre.
33- La prueba estaba........difícil, sin embargo, logré responder correctamente a ........cuestiones.
34- El inglés es un idioma...........interesante. Hay en él............vocablos de origen latina.
35- Encontrar un gusano en la guayaba es algo..........malo, pero..........peor es encontrar sólo medio
gusano en ella.

95
1103362 E-book gerado especialmente para MARCUS ICARO CRUZ DA SILVA
Respuestas:

01- muy 13- mucho 25- mucho/muy


02- mucho 14- mucho 26- mucho
03- mucho 15- muy 27- muy/mucha
04-muy 16- mucho 28- muchos/muy
05- mucho 17- muy 29- muchas/muy
06- muy 18- muy 30- muy/mucha
07- mucho 19- muy 31- muy/mucho
08- muy 20- mucho 32- mucha/mucha
09- muy 21- muy/mucho 33- muy/muchas
10- mucho 22- muy/mucho 34- muy/muchos
11- muy 23- mucho 35- muy/mucho.
12- mucho 24- mucho/muy

ADVÉRBIOS - ADVERBIOS

O advérbio é uma palavra que pode modificar um verbo, um adjetivo ou a outro advérbio. É sempre
invariável. Alguns, quando se referem ao substantivo, tomam caráter adjetivo. Os advérbios se dividem
em:

Advérbios de Tempo (Adverbios de Tiempo)

ahora (agora) mientras (enquanto) luego (depois)


**
anteayer (anteontem) temprano (cedo) entonces (então)
ayer (ontem) mañana (manhã) entretanto (enquanto isso) **
anoche (ontem à noite) hoy (hoje) aún (ainda) *
pronto (em pouco aun (inclusive) * anteanoche (anteontem à
tempo) noite)
después (depois) todavía (ainda) *
* todavía = aún (sinônimos) e diferente de aun.
** entretanto = mientras tanto (enquanto isso - sinônimos).

Advérbios de Modo (Adverbios de Modo)

apenas (apenas) como (como)


bien (bem) entonces (então)
mejor (melhor) inclusive (inclusive)
mal (pouco, insuficiente) sólo (somente) *
peor (pior) fácilmente (facilmente) **
así (assim)

* sólo: somente / solo (adjetivo): sozinho


** e outros terminados em mente.

Advérbios de Lugar (Adverbios de Lugar)

abajo (abaixo) delante (diante)


alrededor (ao redor) detrás (atrás)
arriba (acima) ahí (aí) *
cerca (cerca, perto) allí (ali) *
lejos (longe) aquí (aqui) *

96
1103362 E-book gerado especialmente para MARCUS ICARO CRUZ DA SILVA
* aquí: indica o lugar onde se encontra a pessoa que fala.
ahí: designa um lugar mais próximo que allí.
allí: distante da pessoa que fala.

Advérbios de Quantidade (Adverbios de Cantidad)

casi (quase) poco (pouco)


mucho (muito) * muy (muito) *
más (mais) bastante (bastante)
menos (menos) además (além disso)

* o advérbio muy é usado diante de adjetivos e advérbios:


muy fácil (muito fácil)
muy lejos (muito longe)

* o advérbio mucho é usado diante de substantivos e antes ou depois de verbos em qualquer forma:
Tengo mucho trabajo. (Tenho muito trabalho)
Él mucho ha viajado. (Ele muito viajou.)

Diante dos adjetivos mejor, peor, mayor e menor, e dos advérbios más,
menos, antes e después usamos o advérbio mucho.

Advérbios de Afirmação (Adverbios de Afirmación)

ciertamente (certamente) sí (sim)


seguramente (com segurança) claro (claro)

Advérbios de Negação (Adverbios de Negación)

jamás (jamais) nunca (nunca)


no (não) tampoco (tampouco) *

* Não existe a forma también no para negar. Para isso, usa-se o tampoco.

Advérbios de Dúvida (Adverbios de Duda)

acaso (caso/se) quizá (s) (talvez) *


probablemente (provavelmente) tal vez (talvez)
posiblemente (posivelmente)

* Quizá(s) se antepõe ao verbo. Quando a palavra siguinte começa por -s, se usa a forma quizá e
não quizás. O verbo se conjuga no subjuntivo: Quizá salga.

Advérbios de Ordem (Adverbios de Orden)

antes (antes) primeramente (primeiramente)


después (depois) sucesivamente (sucessivamente)

97
1103362 E-book gerado especialmente para MARCUS ICARO CRUZ DA SILVA
A formação em mente:

Observe que o advérbio pode ser forrmado pelo acréscimo do sufixo mente ao adjetivo feminino.
lenta - lentamente
Quando o adjetivo possui acento, ele o conserva.
fácil - fácilmente

PREPOSIÇÃO

As preposições são invariáveis e servem para unir termos de uma oração, estabelecendo uma
relação, um nexo entre duas palavras - verbos, advérbios, pronomes, substantivos ou adjetivos.
Uso e significado das preposições (Uso y Significado de las Preposiciones)
A
Expressa direção, lugar, modo, finalidade, movimento e tempo. Precede o complemento indireto e
também o direto (quando este se refere a pessoa, animal ou coisa personificada). Precede também
infinitivos, artigos, substantivos, demonstrativos e possessivos.

Vamos a Madrid.
(Vamos a Madrid.)

Está a la izquierda.
(Está à esquerda.)

Hecho a mano.
(Feito a mão.)

Llamamos a Rocío.
(Chamamos Rocío.)

Vamos a estudiar por la noche.


(Vamos estudar à noite.)

¡Lo compré a cien pesos!


(Comprei-o a cem pesos!)

Vi a la niña en la calle.
(Vi a menina na rua.)

Conozco a ese escritor.


(Conheço esse escritor.)

No encontré a mi papá.
(Não encontrei meu pai.)

ANTE

Denota uma situação definida. Se usa também em sentido figurado.


Apareció ante todos. (Apareció delante de todos.)
(Apereceu diante de todos.)

Ante la evidencia, me callo. (Corresponde a perante, diante de, em português.)


(Perante a evidência, me calo.)

BAJO
Expressa dependência, situação inferior.

El trabajo lo hizo bajo presión.


(Fiz o trabalho sob pressão.)

98
1103362 E-book gerado especialmente para MARCUS ICARO CRUZ DA SILVA
Bajo su orientación.
(Sob sua orientação.)

Todos giran y giran, todos bajo el sol. (Mariposa Tecknicolor - Fito Paez)
(Todos giram e giram, todos sob o sol.)

CON
Expressa companhia, conteúdo, meio, instrumento ou maneira.

Salimos con Juan.


(Saímos com Juan.)

Una mesa con sillas.


(Uma mesa com cadeiras.)

Lo escribió con el bolígrafo.


(O escreveu com a caneta.)

Lo hizo con ganas.


(O fiz com vontade.)

Voy a viajar para Barcelona con Pablo o sin él.


(Vou viajar para Barcelona com Pablo ou sem ele.)

CONTRA
Denota limite, oposição, contrariedade.

Compré los pantalones contra su voluntad.


(Comprei as calças contra sua vontade.)

DE
Expressa qualidade, material, modo, movimento, origem, permanência, propriedade e tempo.

María tiene un corazón de oro.


(Maria tem um coração de ouro.)
Volvieron de Rio de Janeiro.
(Voltaram do Rio de Janeiro.)

Manta de lana.
(Manta de lã.)

Trabaja de lunes a sábado.


(Trabalha de segunda a sábado.)
DESDE
Indica um ponto de partida, procedência, distância, lugar, movimento e tempo.

Vinimos desde la calle A hasta la calle B.


(Viemos desde a rua A até a rua B.)

Cuidado!

Desde não deve ser usado com a preposição a, somente com a


preposição hasta. De se usa com a preposição a.

99
1103362 E-book gerado especialmente para MARCUS ICARO CRUZ DA SILVA
DURANTE
Como preposição tem o significado de um determinado tempo ou época.

¿Viajaron durante sus vacaciones?


(Viajaram durante suas férias?)

EN
Expressa lugar, modo e tempo.

Vivo en Argentina.
(Vivo/moro na Argentina.)

Cuéntamelo en secreto.
(Conte-me em segredo.)

Estamos en invierno.
(Estamos no inverno.)

Antes dos dias da semana, de advérbios de tempo e


de alguns adjetivos se omite a preposição EN:
El lunes.
Voy a ir el próximo domingo.

Como meio de transporte ou movimento, a preposição se usa diferente do português:


Voy en avión; en coche; en moto, en ómnibus; en tren.

ENTRE
Situação no meio de duas coisas ou pessoas, dúvida, imprecisão, intervalo e participação em
conjunto.

Entre Pablo y María.


(Entre Pablo e Maria.)

Estábamos entre ir a la fiesta y no ir.


(Estávamos entre ir na festa e não ir.)

El color era entre rojo y naranja.


(A cor era entre vermelho e laranja.)

Nuestra clase es entre las siete y las ocho.


(Nossa aula é entre as sete e as oito.)

El trabajo lo hicieron entre todos.


(Fizeram o trabalho entre todos.)

EXCEPTO
Denota exclusão.

Todos son estudiantes, excepto tú.


(Todos são estudantes, exceto tu.)

HACIA
Expressa direção aproximada, movimento, proximidade e tempo vago.

Viajaré hacia fines de junio.


(Viajarei em meados do fim de junho.)

Vamos hacia el sur de España.


(Vamos em direção ao/para o sul da Espanha.)

100
1103362 E-book gerado especialmente para MARCUS ICARO CRUZ DA SILVA
Lo pondré mirando hacia arriba.
(Coloquei-o olhando para cima.)

HASTA
Indica término de lugar, ação e limite de tempo.

Comió hasta el mareo.


(Comeu até o enjoo.)

Llegaré hasta ahí muy pronto.


(Chegarei até aí muito rápido.)

Saldrá hasta las siete.


(Sairei até as sete.)

Em alguns casos indica inclusão.

Vino, hasta llegó temprano.


(Veio, até chegou cedo.)

INCLUSO
Como preposição, significa hasta.

Todos van a la clase, incluso yo.


(Todos vão à aula, inclusive eu.)

MEDIANTE
Equivale a con e por medio de.

Lo compraron mediante tarjeta de crédito.


(Compraram-no mediante cartão de crédito.)

PARA
Expressa movimento, destino, finalidade e situação.

Voy para São Paulo.


(Vou para São Paulo.)

Esto es para mi.


(Isto é para mim.)

POR
Indica lugar, tempo vago, meio, modo e objetivo. É agente da voz passiva.

La foto está por ahí.


(A foto está por aí.)

Martín llega por ahora.


(Martín chega por agora.)

Lo avisaré por teléfono.


(Avisar-lhe-ei por telefone.)

Lo hará por las buenas o por las malas.


(Fará por bem ou por mal.)

El trabajo lo hice por placer.


(Fiz o trabalho por prazer.)

101
1103362 E-book gerado especialmente para MARCUS ICARO CRUZ DA SILVA
Fue comprado por ella.
(Foi comprado por ela.)

SALVO
Indica exceção.

Todos tus compañeros fueron, salvo Pablo y José.


(Todos os teus companheiros foram, salvo Pablo e José.)

SEGÚN
Indica conformidade.

Hazlo según te parezca mejor.


(Faça-o segundo te pareça melhor.)

SIN
Indica falta, negação.

¿Está sin dinero?


(Está sem dinheiro?)

Estamos sin ganas de trabajar.


(Estamos sem vontade de trabalhar.)

SOBRE
Indica apoio, altura, proximidade e assunto.
El libro está sobre la mesa.
(O livro está sobre a mesa.)

El helicóptero voló sobre mi casa.


(O helicóptero voou sobre minha casa.)

Hablamos sobre las chicas inteligentes.


(Falamos sobre as meninas inteligentes.)

TRAS
Indica posterioridad, situação definida.

Tras una fuerte tormenta salió muy bello el sol.


(Depois de uma forte tempestade, saiu muito bonito o sol.)

CONJUNÇÕES

As conjunções são palavras que unem dois termos de uma mesma oração ou duas orações. Estas
orações podem estabelecer uma relação de coordenação, ou seja, uma está relacionada à outra mas
não há dependência entre elas, ou estabelecem relação de subordinação, ou seja, uma depende da
outra para ter sentido completo.

Conjunções Coordenadas (Conjunciones Coordinadas / Coordinantes)


Copulativas

Unem termos ou orações que expressam ideias similares, estabelecendo uma relação de adição:

Ni rojo, ni morado; prefiero verde.


(Nem vermelho, nem roxo; prefiro verde.)

Tengo para desayunar pan y leche.


(Tenho para o café da manhã pão e leite.)

102
1103362 E-book gerado especialmente para MARCUS ICARO CRUZ DA SILVA
Quiero mi gaseosa con limón y hielo.
(Quero meu refrigerante com limão e gelo.)

Cuidado!

A conjunção y muda para e quando a palavra que segue começa


por i, hi, seguida de consoante.

Este libro es facil e interesante.


(Este livro é fácil e interessante.)

Son padre e hijo.


(São pai e filho.)

Disyuntivas
Unem termos ou orações que expressam ideias opostas, estabelecendo relação de exclusão:

Hay que tener dos o tres alumnos.


(Tem que ter dois ou três alunos.)

Cuidado!

A conjunção o muda para u quando a palavra que segue começa por o, ho.

¿Son siete u ocho?


(São sete ou oito?)

¿Tu perro es mujer u hombre?


(Teu cachorro é mulher ou homem?)

Quando a conjunção o aparece entre números, deve ser acentuada para não ser confundida com o
número zero:
12 ó 15.

Distributivas
Unem termos ou orações que expressam diferenças lógicas, temporais, espaciais ou de qualquer
outro tipo:

Bien para mí, bien para tu hermano, tendrás que contarlo todo. (bien... bien)
(Bem para mim, bem para teu irmão, terás que contar tudo.)

Ora por una cosa, ora por otra, nunca consigo estudiar. (ora... ora)
(Ora por uma coisa, ora por outra, nunca consigo estudar.)

Ya en tren, ya en autobús, iremos igual. (ya... ya)


(Seja de trem, seja de ônibus, iremos igual.)

Uno para mí, otro para tí. (uno... otro)


(Um para mim, outro para ti.)

Adversativas
Unem termos ou orações que se contrapõem entre si:

Me gustaría ir, pero no tengo dinero. (= mas)


(Gostaria de ir, mas não tenho dinheiro.)

103
1103362 E-book gerado especialmente para MARCUS ICARO CRUZ DA SILVA
No quiero té sino café solo. (mas sim)
(Não quero chá, mas sim café preto.)

No les gustan comer frutas sino manzanas. (exceto)


(Não gostam de comer frutas, exceto maçãs.)

Esta chica no hace otra cosa sino llorar. (a não ser)


(Esta menina não faz outra coisa a não ser chorar.)

Saldré esta mañana aunque llueva.


(Sairé esta manhã mesmo que chova.)

Tenía muchos motivos para hacerlo hablar, sin embargo no lo hizo.


(Tinha muitos motivos para fazê-lo falar, no entanto não o fiz.)

Outras conjunções que designam ideias contrárias: excepto, no obstante, antes, antes bien, a pesar
de, con todo, más bien, fuera de.

Conjunções Subordinadas (Conjunciones Subordinadas / Subordinantes)

Causales
Expressam casua, motivo da ação expressa pelo verbo da oração principal:

La fiesta será buena, ya que he invitado todos mis amigos.


(A festa será boa, já que convidei todos os meus amigos.)

Vamos sacar buenas notas en las pruebas porque estudiamos mucho.


(Vamos tirar boas notas nas provas porque estudamos muito.)

Outras conjunções que designam causa: como, que, pues, puesto que, debido a que, etc.
Finales
Expressam objetivo ou finalidade da ação expressa pelo verbo da oração principal:

Lo haré a fin de que entiendas.


(Farei isso a fim de que entendas.)

Outras conjunções que designam finalidade: porque, para que, de modo que, etc.

Temporales
Expressam diferentes matrizes do tempo em que ocorre a ação expressa pelo verbo da oração
principal:

Mientras me baño, tu haces las tareas. (enquanto - simultaneidade)


(Enquanto tomo banho, tu fazes as tarefas.)

En cuanto lleguen los invitados, avísame. (tão logo, assim que)


(Assim que chegarem os convidados, avisa-me.)

Te llamaré apenas llegue a Madrid. (tão logo, assim que)


(Te ligarei tão logo chegue em Madrid.)

Cuando era niña, ¿te gustaba ir al cine?


(Quando era menina, gostava de ir ao cinema?)

Consecutivas
Expressam o efeito ou a consequência da ação expressa pela oração principal:

Tengo mucha hambre, conque comeré unas galletas. (portanto)


(Tenho muita fome, portanto comerei umas bolachas.)

104
1103362 E-book gerado especialmente para MARCUS ICARO CRUZ DA SILVA
No estudiaste lo suficiente, luego no tendrás buenas notas.
(Não estudaste o suficiente, logo não terás boas notas.)

Tú eres la única persona que leyó el texto, así que eres quien lo puede explicar. (de modo que)
(Tu és a única pessoa que leu o texto, de modo que és quem pode explicá-lo.)

Concesivas
Expressam concessão ou ainda uma oposição à ideia expressa pelo verbo da oração principal:

Aunque no lo merezcas, te ayudaré. (embora)


(Embora não mereças, te ajudarei.)

Outras conjunções que designam concessão: a pesar de que, y eso que, si bien, etc.

Condicionales
Expressam condição necessária ou hipótese para que se realize a ação expressa pelo verbo da
oração principal:

Como me extrañes mucho, te escribo.


(Como sentes muito minha falta, te escrevo.)

Si buscas la paz, la encontrarás.


(Se buscas a paz, a encontrarás.)

Outras conjunções que designam condição: ya que, siempre que, con tal que.

SUBSTANTIVOS

Plural de Los Substantivos

Añadiendo una S:
1) caso el singular termine en vocal no acentuada (o en E acentuada, algunas veces).
el perro - los perros
el hombre - los hombres
el café - los cafés

Añadiendo la sílaba ES:


2) Caso el singular termine en vocal tónica o consonante
el jabalí - los jabalíes
el rubí - los rubíes
el reloj - los relojes
el corazón - los corazones

Se exceptúan:
papá - papás
mamá - mamás
sofá - sofás

3) El plural es igual al singular cuando éste termina en S, Y si la palabra es grave o esdrújula:


la tesis - las tesis
la dosis - las dosis

4) Los sustantivos terminados en X conservan la misma forma en el plural:


el fénix - los fénix
la ónix - las ónix

105
1103362 E-book gerado especialmente para MARCUS ICARO CRUZ DA SILVA
5) Los sustantivos terminados en Z cambian esa letra en C y se agrega ES:
el pez - los peces
la raíz - las raíces
la luz - las luces
la paz - las paces
la vez - las veces

6) Para los sustantivos terminados en Y, se agrega ES:


el rey – los reyes
la ley - las leyes

Flexiones Irregulares

hombre - mujer
padrino - madrina
toro, buey - vaca
papá - mamá
caballero - dama
caballo – yegua
padre - madre
marido - mujer
yerno - nuera
padrastro - madrastra
carnero - oveja
macho - hembra

Plural de Los Adjetivos

Los adjetivos forman el plural siguiendo las mismas reglas que rigen para los substantivos.
mala - malas
feliz - felices
dulce - dulces
baladi - baladies
fácil - fáciles
cordial - cordiales

Formácion del Femenino

Si el masculino termina en vocal, se cambia ésta por una a; si termina en consonante se agrega una
a.
esposo - esposa
pariente - parienta
tio - tia
huésped - huéspeda
león - leona
aprendiz - aprendiza

Excepciones:

1ª) Terminados en INA


gallo - gallina
héroe - heroína
rey - reina

2ª) Terminados en ESA


abad – abadesa
alcalde - alcaldesa
barón – baronesa
onde - condesa

106
1103362 E-book gerado especialmente para MARCUS ICARO CRUZ DA SILVA
3ª) Terminados em ISA
poeta – poetisa
sacerdote - sacerdotisa

4ª) Terminados em TRIZ


actor – atriz
emperador – emperatriz

ADJETIVOS

O adjetivo é a palavra que funciona como modificador direto do substantivo, qualificando-o. Concorda
sempre com o substantivo que acompanha, sofrendo, assim, variação de gênero, número e grau.

Variação de gênero: La camisa amarilla. (A camisa amarela.)


Variação de número: Los alumnos estudiosos. (Os alunos estudiosos.)
Variação de grau: Victor es más fuerte que Javier. (Victor é mais forte que Javier.)

Classificação dos Adjetivos (Clasificación de los Adjetivos)

Primitivos Derivados
bueno (bom) bondadoso (bondoso)
Simple (Simples) Compuesto (Composto)
fuerte (forte) multicolor (multicor)

Patrios (Pátrios) ou Gentilicios (Gentílicos)


canadiense (canadense), chino (chinês)

Gênero dos Adjetivos (Género de los Adjetivos)

a) Os adjetivos masculinos terminados em o ou e mudam a terminação para a na formação do


feminino.

feo (feio) - fea (feia)


grandote (grandalhão) - grandota (grandalhona)
b) Nos adjetivos masculinos terminados em an, in, on, or e nos gentílicos terminados em consoante,
acrescenta-se -a na formação do feminino.

soñador (sonhador) - soñadora (sonhadora)


inglés (inglês) - inglesa (inglesa)

c) Os adjetivos invariáveis mantêm a mesma forma quando acompanham substantivos masculinos


ou femininos.

Un hombre feliz. (Um homem feliz.) - Una mujer feliz. (Uma mulher feliz.)
hermano menor (irmão menor) - hermana menor (irmã menor)

Apócope

Chama-se apócope a supressão da letra ou da sílaba final em alguns adjetivos.

a) Os adjetivos alguno, bueno, malo, ninguno, primero, postrero, tercero e uno perdem a letra o final
quando precedem um substantivo masculino singular:

Algún chico (algum menino)


Buen hombre (bom homem)
Mal tiempo (mau tempo)
Ningún libro (nenhum livro)
Primer lugar (primeiro lugar)

107
1103362 E-book gerado especialmente para MARCUS ICARO CRUZ DA SILVA
Postrer día (último dia)
Tercer piso (terceiro andar)
Un profesor (um professor)

b) O adjetivo ciento perde a sílaba final to quando precede substantivos plurais, masculinos ou
femininos, mesmo que se interponha um adjetivo:

Cien hombres (cem homens)


Cien mujeres (cem mulheres)
Cien lindas muchachas (cem lindas mulheres)

c) O adjetivo cualquiera perde a letra a final quando precede substantivos singulares, masculinos ou
femininos:

Cualquier libro (qualquer livro)


Cualquier carpeta (qualquer pasta*)

* material de escritório para guardar documentos.

O plural CUALESQUIERA também sofre apócope: cualesquier hombres / cualesquier mujeres.

d) O adjetivo grande perde a sílaba final de quando precede substantivos singulares, masculinos ou
femininos:

Gran chico (grande menino)


Gran chica (grande menina)
e) O adjetivo santo perde a sílaba final to quando precede nomes próprios masculinos de santos,
exceto diante de Domingo, Tomás, Tomé e Toribio:
San Juán
Número dos Adjetivos (Número de los Adjetivos)

Os adjetivos formam plural da mesma forma que os substantivos.

manzana roja (maçã vermelha) - manzanas rojas (maçãs vermelhas)


prueba fácil (prova fácil) - pruebas fáciles (provas fáceis)

Lembre-se:

Simples é a forma plural referente a simple (singular).


Una idea simple (uma ideia simples) - unas ideas simples (umas ideias simples)

PRONOMBRES

Los Pronombres Personales

Los pronombres personales son los que sirven de sujeto en una oración. Ellos son:

Persona Singular Plural


1ª yo nosotros, nosotras
2ª tú vosotros, vosotras
3ª él, ella, Ud. ellos, ellas, Uds.

Ejemplos:
Yo estaba en casa ayer.
Alonso, tú ya has comido bastantes chocolates.
Pedro es español; él es de Sevilla.
Lucía es mi hermana; ella tiene 19 años.
¿Es maestro Ud.?

108
1103362 E-book gerado especialmente para MARCUS ICARO CRUZ DA SILVA
Carlos y yo fuimos al cine anoche; nosotros vimos una película argentina.
Ignacio y Eugenio, ¿sois vosotros de España?
Los chicos jugaban en el jardín, ahora ellos están muy sucios.

Los Pronombres Demostrativos

Los pronombres y adjetivos demostrativos sirven para indicar la relación de proximidad entre los
objetos a que se refieren y los participantes del diálogo. Ejemplo:

Esta ventana necesita reparo → (cerca)


Ése es el sombrero de Juan. → (menos cerca)
Aquélla es mi abuela. → (lejos)

Los Pronombres Demostrativos

MASCULINO FEMENINO NEUTRO


Singular/plural éste/éstos ésta/éstas esto cerca
Singular/plural ése/ésos ésa/ésas eso menos cerca
Singular/plural aquél/aquéllos Aquélla/aquéllas aquello lejos

Nota: Para cada pronombre demostrativo (masculino y femenino) hay


un adjetivo, pero los adjetivos no llevan la tilde (´).

Ejemplo:

Éste (pronombre)
Este (adjetivo)

Funciones: Indican la relación de proximidad entre el objeto al que se refieren y los diversos
participantes del diálogo. Esta proximidad puede ser local, temporal, afectiva.

Ejemplos:

Esta revista que tengo en las manos.


Aquel examen fue más difícil que éste de hoy.
Aquellos soldados fueron un ejemplo de valor.

Concordancia: los pronombres demostrativos siempre concuerdan en género y número con el


nombre al que se refiere.

Ejemplos:

Este coche es nuevo.


Esa escuela está cerrada.

Los Pronombres Posesivos

Recuerda que los adjetivos posesivos vienen seguidos de sustantivos:

Mi(s) livro (s), tu(s) prima(s), su(s) coche(s)

109
1103362 E-book gerado especialmente para MARCUS ICARO CRUZ DA SILVA
Adjetivo Masculino Femenino Masculino Femenino
(mi / mis) Mío Mía Míos Mías
(tu / tus) Tuyo Tuya Tuyos Tuyas
(su / sus) Suyo Suya Suyos Suyas
(nuestro (a) / Nuestros (as) Nuestro Nuestra Nuestros Nuestras
(vuestro (a) / vuestros (as) Vuestro Vuestra Vuestros Vuestras
(su / sus) Suyo Suya Suyos Suyas

Para cada adjetivo possessivo corresponde um pronome possessivo, porém o pronome não vem
seguido de substantivo.

adjetivo adjetivo

Aquél es tu libro y éste es mi libro.

Aquél es tuyo libro y éste es mío libro.

pronombre pronombre
Los complementos directos (CD) e indirectos (CI) corresponden, en portugués, a los objetos diretos
(OD) e indiretos (OI).

Oraciones con CD:

Él escribió esta carta.(CD) = Ele escreveu esta carta.(OD)

Ayer vimos a Maria. (CD) = Ontem vimos a Maria. (OD Preposicionado)


Oraciones con CI:

Él escribió para Juan. (CI)


Ele escreveu para Juan. (OI)
Telefonamos a Lucia. (CI)
Telefonamos para Lucia. (OI)

Los pronombres complemento

Así como los nombres (esta carta, a María, para Juan, a Lucia), también los pronombres pueden
tener la función de complementos directos o indirectos. En este caso se llaman pronombres
complemento. Observa las transformaciones en las oraciones:
Él escribió esta carta (CD). Él la escribió. (CD) = Ele a escreveu.
Ayer vimos a Maria (CD). Ayer la vimos. (CD) = Ontem a vimos.
Él escribió para Juan (CI). Él le escribió. (CI). = Ele lhe escreveu
Telefoneamos a Lucia (CI). Le telefoneamos. (CI). = Lhe telefonamos

Los Pronombres Átonos

Pronombres Pronombres Pronombres tónicos


CD átonos CI átonos con preposición
Yo me me mí, conmigo3
Tú te te Ti, contigo3
Él/usted (hombre) lo/le1 le(se)2 a él/usted
Ella/usted(mujer) la le(se)2 para ella/usted
Nosotros/nosotras nos nos con nosotros (as)
Vosotros/vosotras os os de vosotros (as)
Ellos/ustedes(hombres) los/les1 les(se)2 Ellos/ustedes
Ellas/ustedes (mujer) las les(se) 2 Ellas(ustedes

110
1103362 E-book gerado especialmente para MARCUS ICARO CRUZ DA SILVA
Los Pronombres Tónicos pueden ser usados con las preposiciones:

A / PARA / CON / DE

01- El complemento directo que se refiere a persona (s) masculina (s) se puede sustituir por lo(s) o
le (s).

Ejemplos:

Yo vi a Juan. Yo lo vi. o Yo le vi.


Yo vi a los niños. Yo los vi. o Yo les vi.

02- El cambio de le/les para se sólo ocurre para 3ª persona (singular y plural) y cuando los dos
complementos son pronombres átonos:

Él le dio una flor. Pero Él se la dio.

=a ella los dos complementos son pronombres átonos


03- Las formas conmigo y contigo ya vienen con la preposición con (como ocurre en portugués).

Ejemplos:

Uso de complementos directo:

Él vio a mí. Él me vio.


Él llamó a ti. Él te llamó.
Él vio a Dolores. Él la vio.

Uso de complementos indirecto:


Él escribió una carta para mí. Él me escribió una carta.
Él leyó un poema para ti. Él te leyó un poema.
Él dio un regalo a ella. Él le dio un regalo.

Cuestiones

01- ¿Adjetivo o pronombre? Rellena los huecos con una de las palabras entre paréntesis:
(A) _____ coche es del profesor de matemática. (éste / este)
(B) ¿de quién son _______ libros? (éstos / estos)
(C) ______ es mi prima verónica. (aquélla / aquella)
(D) ¿quién es ______ chica? (ésa / esa)
(E) ¿qué calle es _______? (ésta / esta)

02- Rellena los huecos con demostrativos:


(A) ¿Es _____ el sombrero de tu padre? (cerca)
(B) ______ panadero es muy trabajador. (lejos)
(C) ______ son ingenieros de la empresa. (menos cerca)
(D) _____ peluquera es muy buena. (menos cerca)
(E) _____ son las modistas más famosas de Madrid. (cerca)

03- Contesta a las preguntas:


(A) ¿Dé quién son estos guantes? (yo)
(B) ¿Dé quién es esa casa? (nosotros)
(C) ¿Dé quién es esta revista? (tú)
(D) ¿Dé quién es esa llave? (él)
(E) ¿De quién son estos libros? (vosotros)

04- Rellena las frases con el posesivo correcto:


(A) _____ casa es grande. (mis / mi)
(B) _____ madre se llama rosa. (su/ sus)

111
1103362 E-book gerado especialmente para MARCUS ICARO CRUZ DA SILVA
(C) _____ maestra es ana. (tu / tus)
(D) _____ colegio tiene una biblioteca (nuestro /nuestra)
(E) ______ amigas dicen que _____ ciudad es limpia. (su / sus)

05- Sustituye las palabras en destaque por el pronombre correspondiente:

(A)- Él lee el periódico todos los días.


Él______ lee todos los días.

(B)- Espero la noticia con impaciencia.


______ espero con impaciencia.

(C)- Ellos preguntaron todo al agente.


Ellos______ preguntaron todo.

(D)- Miró a su hijo con cariño.


______ miró con cariño.

(E)- Ella explicó el problema a las niñas.


Ella______ explicó el problema.

(F)- Vi los libros encima de la mesa.


_______ vi encima de la mesa.

(G)- José regaló un disco a nosotros.


José_______ regaló un disco.

(H)- Sus nietos trajeron estos dulces a usted.


Sus nietos_______ trajeron estos dulces.

06- Completa las frases con los pronombres personales:


(A) ¿Quién soy______? (1ª pers., sing.)
(B) _______ sois muy buenos. (2ª pers., pl., masc.)
(C) _______ se llama María. ( 3ª pers., sing., fem.)
(D) ______ , por favor, pueden entrar. (3ª pers., pl.)
(E) ¿ Qué haces _______? (2ª pers., sing)
(F) ________ no somos hermanas. (1ª pers., pl., fem.)

Respuestas:

1- (a. Éste) (b. estos ) (c. Aquélla) (d. esa) ( e. ésta)


2- (a. Èste) ( b. Aquel) (c. Ésos) (d. Esa) (e. Éstas)
3- (a. Son míos) (b. Es nuestra) (c. Es tuya) (d. Es suya) (e. Son vuestros)
4- (a. MI) (b. Su) (c. Tu) (d. Nuestro) (e. Sus / su)
5- a- Lo b- La c- Le d- Lo/Le e- Les f- Los g- Nos h- Le
6- (a- yo) (b- Vosotros) (c- Ella) (d-Ustedes) (e-Tú) (f- Nosotras)

ARTÍCULOS

Los artículos definidos:

SINGULAR PLURAL
MASCULINO EL (o) LOS (os)
FEMENINO LA (a) LAS (as)

CONTRACCIONES: A + EL= AL DE + EL= DEL

112
1103362 E-book gerado especialmente para MARCUS ICARO CRUZ DA SILVA
El artículo siempre precede al nombre.

Ejemplo:
El abogado, La audiencia, Las leyes, Los jueces.

Entre el artículo y el nombre pueden interponer adjetivos o adverbios.

Ejemplo:
El mundialmente conocido abogado, La primera lección, Las nuevas leyes, Los ilustres jueces.

Es incorrecto el uso de artículo delante de nombres de personas o de lugares geográficos, como


ciudades, pueblos, países etc.

Es incorrecto decir: La España, El Pedro, La María, El México, etc.

Pocos nombres de países pueden ser precedidos de artículo, a pesar de no ser obligatorio su
utilización.
Ejemplo:
El Brasil, La Argentina, El Uruguay, El Paraguay El Perú, Los Estados Unidos,
El Canadá, El Ecuador y El Japón.

Algunos nombres de países y ciudades deben llevar, obligatoriamente, el artículo precediéndolos.

Ejemplo:
El Salvador, La India, La Habana, El Cairo, La Coruña, La Rioja.

Puede ponerse precediendo nombres de personas o comarcas si se cualifican los mismos.

Ejemplo:
El gran Napoleón, La buena María, La España antigua, La Francia moderna, etc.

Cuando los apellidos o nombres propios van en plural.

Ejemplo:
Los Fernández, Las Marías, Los Pérez, etc.

Los Artículos Indefinidos

SINGULAR PLURAL
MASCULINO UN (um) UNOS (uns)
FEMENINO UNA (uma) UNAS (umas)

El artículo siempre precede al nombre.

Ejemplo:
Un profesor, Una clase, Unas leyes, Unos jueces.
Entre el artículo y el nombre pueden interponerse adjetivos o adverbios.

Ejemplo:
Un mundialmente conocido abogado; Una interesante mujer; Unas severas leyes; Unos ilustres
jueces.

Es incorrecto el uso de artículo indefinido delante de las palabras otro/s u otra/s.

Ejemplo:
"OTRO ESCRIBANO" y no "un otro escribano"
"OTRO DÍA" y no "un otro día"

113
1103362 E-book gerado especialmente para MARCUS ICARO CRUZ DA SILVA
"Unos" y "Unas" también se utilizan con el sentido de "aproximadamente" para distancias, fechas,
medidas, etc.

Ejemplo:
Mi escuela está a unos cinco kilómetros de aquí.
La casa de la tía de María está a unas tres cuadras del tribunal.
Hace unos seis años que no soy multado.

Cuestiones

01- Ponga el artículo definido:


(A) _____ niño está paseando.
(B) _____ mujer está feliz.
(C) _____ águila está volando.
(D) _____ río es limpio.
(E) _____ hombres están trabajando.
(F) _____ flores son amarillas.

02- Ponga el artículo indefinido:


(A) ______ día iré a España.
(B) ______ señora está en la puerta.
(C) Hace ______ mañana estupenda.
(D) _______ alumnos llegaron tarde.
(E) Bebimos______ cuantas cervezas.
(F) ______ mujeres pasean por el parque.

Respuestas:

01- (a- EL) (b- LA) (c- EL) (d-EL) (e- LOS) (f- LAS)
02- (a- UN) (b- UNA) (c- UNA) (d- UNOS) (e- UNAS) (f- UNAS)

PRONOMBRES

Los Pronombres Personales

Los pronombres personales son los que sirven de sujeto en una oración. Ellos son:

Persona Singular Plural


1ª yo nosotros, nosotras
2ª tú vosotros, vosotras
3ª él, ella, Ud. ellos, ellas, Uds.
Ejemplos:

Yo estaba en casa ayer.


Alonso, tú ya has comido bastantes chocolates.
Pedro es español; él es de Sevilla.
Lucía es mi hermana; ella tiene 19 años.
¿Es maestro Ud.?
Carlos y yo fuimos al cine anoche; nosotros vimos una película argentina.
Ignacio y Eugenio, ¿sois vosotros de España?
Los chicos jugaban en el jardín, ahora ellos están muy sucios.
Los Pronombres Demostrativos

Los pronombres y adjetivos demostrativos sirven para indicar la relación de proximidad entre los
objetos a que se refieren y los participantes del diálogo. Ejemplo:

Esta ventana necesita reparo → (cerca)


Ése es el sombrero de Juan. → (menos cerca)
Aquélla es mi abuela. → (lejos)

114
1103362 E-book gerado especialmente para MARCUS ICARO CRUZ DA SILVA
Los Pronombres Demostrativos

MASCULINO FEMENINO NEUTRO


Singular/plural éste/éstos ésta/éstas esto cerca
Singular/plural ése/ésos ésa/ésas eso menos cerca
Singular/plural aquél/aquéllos Aquélla/aquéllas aquello lejos

Nota: Para cada pronombre demostrativo (masculino y femenino) hay


un adjetivo, pero los adjetivos no llevan la tilde (´).

Ejemplo:

Éste (pronombre)
Este (adjetivo)

Funciones: Indican la relación de proximidad entre el objeto al que se refieren y los diversos
participantes del diálogo. Esta proximidad puede ser local, temporal, afectiva.

Ejemplos:

Esta revista que tengo en las manos.


Aquel examen fue más difícil que éste de hoy.
Aquellos soldados fueron un ejemplo de valor.

Concordancia: los pronombres demostrativos siempre concuerdan en género y número con el


nombre al que se refiere.

Ejemplos:

Este coche es nuevo.


Esa escuela está cerrada.

Los Pronombres Posesivos

Recuerda que los adjetivos posesivos vienen seguidos de sustantivos:

Mi(s) livro (s), tu(s) prima(s), su(s) coche(s)

Adjetivo Masculino Femenino Masculino Femenino


(mi / mis) Mío Mía Míos Mías
(tu / tus) Tuyo Tuya Tuyos Tuyas
(su / sus) Suyo Suya Suyos Suyas
(nuestro (a) / Nuestros (as) Nuestro Nuestra Nuestros Nuestras
(vuestro (a) / vuestros (as) Vuestro Vuestra Vuestros Vuestras
(su / sus) Suyo Suya Suyos Suyas

Para cada adjetivo possessivo corresponde um pronome possessivo, porém o pronome não vem
seguido de substantivo.

adjetivo adjetivo

Aquél es tu libro y éste es mi libro.

115
1103362 E-book gerado especialmente para MARCUS ICARO CRUZ DA SILVA
Aquél es tuyo libro y éste es mío libro.

pronombre pronombre

Los complementos directos (CD) e indirectos (CI) corresponden, en portugués, a los objetos diretos
(OD) e indiretos (OI).
Oraciones con CD:

Él escribió esta carta.(CD) = Ele escreveu esta carta.(OD)

Ayer vimos a Maria. (CD) = Ontem vimos a Maria. (OD Preposicionado)


Oraciones con CI:

Él escribió para Juan. (CI)


Ele escreveu para Juan. (OI)
Telefonamos a Lucia. (CI)
Telefonamos para Lucia. (OI)

Los pronombres complemento

Así como los nombres (esta carta, a María, para Juan, a Lucia), también los pronombres pueden
tener la función de complementos directos o indirectos. En este caso se llaman pronombres
complemento.

Observa las transformaciones en las oraciones:


Él escribió esta carta (CD). Él la escribió. (CD) = Ele a escreveu.
Ayer vimos a Maria (CD). Ayer la vimos. (CD) = Ontem a vimos.
Él escribió para Juan (CI). Él le escribió. (CI). = Ele lhe escreveu
Telefoneamos a Lucia (CI). Le telefoneamos. (CI). = Lhe telefonamos

Los Pronombres Átonos

Pronombres Pronombres Pronombres tónicos


CD átonos CI átonos con preposición
Yo me me mí, conmigo3
Tú te te Ti, contigo3
Él/usted (hombre) lo/le1 le(se)2 a él/usted
Ella/usted(mujer) la le(se)2 para ella/usted
Nosotros/nosotras nos nos con nosotros (as)
Vosotros/vosotras os os de vosotros (as)
Ellos/ustedes(hombres) los/les1 les(se)2 Ellos/ustedes
Ellas/ustedes (mujer) las les(se) 2 Ellas(ustedes

Los Pronombres Tónicos pueden ser usados con las preposiciones:

A / PARA /CON /DE

01- El complemento directo que se refiere a persona (s) masculina (s) se puede sustituir por lo(s) o
le (s).

Ejemplos:

Yo vi a Juan. Yo lo vi. o Yo le vi.


Yo vi a los niños. Yo los vi. o Yo les vi.

02- El cambio de le/les para se sólo ocurre para 3ª persona (singular y plural) y cuando los dos
complementos son pronombres átonos:

116
1103362 E-book gerado especialmente para MARCUS ICARO CRUZ DA SILVA
Él le dio una flor. Pero Él se la dio.

=a ella los dos complementos son pronombres átonos


03- Las formas conmigo y contigo ya vienen con la preposición con (como ocurre en portugués).

Ejemplos:

Uso de complementos directo:

Él vio a mí. Él me vio.


Él llamó a ti. Él te llamó.
Él vio a Dolores. Él la vio.

Uso de complementos indirecto:

Él escribió una carta para mí. Él me escribió una carta.


Él leyó un poema para ti. Él te leyó un poema.
Él dio un regalo a ella. Él le dio un regalo.

Cuestiones

01- ¿Adjetivo o pronombre? Rellena los huecos con una de las palabras entre paréntesis:
(A) _____ coche es del profesor de matemática. (éste / este)
(B) ¿de quién son _______ libros? (éstos / estos)
(C) ______ es mi prima verónica. (aquélla / aquella)
(D) ¿quién es ______ chica? (ésa / esa)
(E) ¿qué calle es _______? (ésta / esta)

02- Rellena los huecos con demostrativos:


(A) ¿Es _____ el sombrero de tu padre? (cerca)
(B) ______ panadero es muy trabajador. (lejos)
(C) ______ son ingenieros de la empresa. (menos cerca)
(D) _____ peluquera es muy buena. (menos cerca)
(E) _____ son las modistas más famosas de Madrid. (cerca)

03- Contesta a las preguntas:


(A) ¿Dé quién son estos guantes? (yo)
(B) ¿Dé quién es esa casa? (nosotros)
(C) ¿Dé quién es esta revista? (tú)
(D) ¿Dé quién es esa llave? (él)
(E) ¿De quién son estos libros? (vosotros)

04- Rellena las frases con el posesivo correcto:


(A) _____ casa es grande. (mis / mi)
(B) _____ madre se llama rosa. (su/ sus)
(C) _____ maestra es ana. (tu / tus)
(D) _____ colegio tiene una biblioteca (nuestro /nuestra)
(E) ______ amigas dicen que _____ ciudad es limpia. (su / sus)

05- Sustituye las palabras en destaque por el pronombre correspondiente:

(A)- Él lee el periódico todos los días.


Él______ lee todos los días.

(B)- Espero la noticia con impaciencia.


______ espero con impaciencia.

(C)- Ellos preguntaron todo al agente.


Ellos______ preguntaron todo.

117
1103362 E-book gerado especialmente para MARCUS ICARO CRUZ DA SILVA
(D)- Miró a su hijo con cariño.
______ miró con cariño.

(E)- Ella explicó el problema a las niñas.


Ella______ explicó el problema.

(F)- Vi los libros encima de la mesa.


_______ vi encima de la mesa.

(G)- José regaló un disco a nosotros.


José_______ regaló un disco.

(H)- Sus nietos trajeron estos dulces a usted.


Sus nietos_______ trajeron estos dulces.

06- Completa las frases con los pronombres personales:


(A) ¿Quién soy______? (1ª pers., sing.)
(B) _______ sois muy buenos. (2ª pers., pl., masc.)
(C) _______ se llama María. ( 3ª pers., sing., fem.)
(D) ______ , por favor, pueden entrar. (3ª pers., pl.)
(E) ¿ Qué haces _______? (2ª pers., sing)
(F) ________ no somos hermanas. (1ª pers., pl., fem.)

Respuestas:

1- (a. Éste) (b. estos ) (c. Aquélla) (d. esa) ( e. ésta)


2- (a. Èste) ( b. Aquel) (c. Ésos) (d. Esa) (e. Éstas)
3- (a. Son míos) (b. Es nuestra) (c. Es tuya) (d. Es suya) (e. Son vuestros)
4- (a. MI) (b. Su) (c. Tu) (d. Nuestro) (e. Sus / su)
5- a- Lo b- La c- Le d- Lo/Le e- Les f- Los g- Nos h- Le
6- (a- yo) (b- Vosotros) (c- Ella) (d-Ustedes) (e-Tú) (f- Nosotras)

A partícula “SE”1

A partícula se, em espanhol assim como em português, desempenha várias funções na estrutura da
língua. Vejamos mais detalhamente quando devemos usá-la na língua espanhola. Aconselho também a
ler a postagem: O uso de sí e si.

Se pode ser um pronome da terceira pessoa do singular ou do plural quando vem antes de lo, las,
los, las. Equivale portanto a le e les. Para sabe mais sobre o assunto, clique aqui.

Enviamos armas a los afganos. *Las les enviamos. *Les las enviamos. Se las enviamos. (Lhas
enviamos.)
*formas incorretas de acordo com as normas gramaticais.

Se, também, é usado como pronombre reflexivo ou recíproco. Neste caso, também, nas terceiras
pessoas do singular e plural.

Se llama Juan. (Chama-se Juan.)


Por favor, siéntense. (Por favor, sentem-se.)
Se, ainda, é usado:

Para formar las oraciones pasivas reflejas. Clique aqui e saiba mais sobre las pasivas reflejas.
Se explicaron los pasos para proteger la Tierra. (Foram explicados os passos para proteger a Terra.)
Nas oraciones impersonales (orações impessoais): De forma geral, a oração é impessoal com se,
quando o verbo está na terceira pessoa do singular e no lugar do sujeito usamos a partícula se. Quando
o verbo vem na terceira pessoa do plural, trata-se de uma pasija refleja.

1
http://www.guiapraticodeespanhol.com.br/2010/08/particula-se-em-espanhol.html

118
1103362 E-book gerado especialmente para MARCUS ICARO CRUZ DA SILVA
Los países donde se habla español (Os países onde fala-se espanhol.)
Com verbos pronominales. Clique aqui e saiba mais sobre verbos pronominales.
Se ou Sé

Fique atento sé (com acento) significa sei ou seja. Leia mais sobre este assunto clicando aqui.

¿Cómo sé si tengo diabetes? (Como sei se tenho diabetes?)


Sé más eficaz. (Seja mais eficaz.)
Já se (sem acento) significa se ou lhe(s), como foi explicado nesta postagem.

Real Madrid se lamenta: operaron a Kaká y no jugará por cuatro meses. (Real Madrid se lamenta:
operaram Kaká e não jogará por quatro meses.)
Quiero trabajar tres sistemas tácticos diferentes y ayer se los expliqué a los jugadores. (Quero
trabalhar três sistemas táticos diferentes e ontem expliquei-os aos jogadores.)
Na última frase, o se desempenha o papel de lhes (referindo-se aos jogadores). Na tradução para o
português, o lhes não aparece porque seria um redundância, ou seja, teria dois objetos indiretos (lhes,
aos jogadores). Em espanhol, contudo esta construção é bastante comum.

A partícula “lo”2

O artigo neutro lo, inexistente em língua portuguesa, é utilizado para substantivar adjetivos e
advérbios.

Lo mejor de todo fue la fiesta. (mejor = melhor - adjetivo)


(O melhor de tudo foi a festa.)

La paz es lo más valioso sentimiento. (más = mais - advérbio)


(A paz é o mais valioso sentimento.)

Cuidado!

O artigo neutro LO é utilizado antes de adjetivo + preposição. Se depois do


adjetivo não tiver preposição, usa-se o artigo definido masculino singular EL.

Lo bonito en un partido es ver goles.


(O bonito em uma partida é ver gols.)

El bello coche de Pablo fue muy caro.


(O belo carro de Pablo foi muito caro.)

Também se utiliza diante do pronome relativo que. Equivale a aquilo que, o que.

Lo que me encanta en ti es tu inteligencia.


(O que me fascina em ti é a tua inteligência.)

¡Atención!

Nunca coloque o artigo neutro "lo" na frente de substantivos masculinos. É


muito comum os brasileiros cometerem esse erro, confundindo lo com o
(artigo masculino, em português). Substantivos masculinos aceitam
somente o artigo el.

Expressões coloquiais com Lo

2
http://www.soespanhol.com.br/conteudo/artigos2.php

119
1103362 E-book gerado especialmente para MARCUS ICARO CRUZ DA SILVA
Emprega-se o artigo neutro lo em diversas expressões coloquiais. Por ele não apresentar variação
para gênero e número, chama-se neutro. Veja alguns exemplos:

Lo + adjetivo + que: intensifica o valor do adjetivo.

No sabes lo complicada que es mi situación en la oficina.


(Você não sabe o quanto é complicada minha situação no escritório.)

Lo + advérbio + que: intensifica o valor do advérbio.

No me di cuenta de lo rápido que pasó este año.


(Não me dei conta do quanto passou rápido este ano.)

Lo + participio + que: intensifica o valor do particípio.

Mira lo roto que está este traje!


(Olha que rasgada que está esta roupa!)

Contração do Artigo (Contracción del Artículo)

A língua espanhola possui apenas dois tipos de contração: al e del.

AL: Preposição a + artigo el

Voy al puerto.

(Vou ao porto.)

DEL: Preposição de + artigo el

Vengo del puerto.


(Venho do porto.)

DISCURSOS DIRETO E INDIRETO - DISCURSOS DIRECTO E INDIRECTO3

A fala de uma pessoa pode ser narrada empregando-se o discurso direto ou o discurso indireto.
O discurso direto é aquele em que se repetem textualmente as palavras do interlocutor.

Llego el lunes.
(Chego na segunda-feira.)

O discurso indireto é aquele em que se relata algo que o interlocutor diz, sem que suas palavras
sejam repetidas textualmente.

Hernandez dice que llega el lunes.


(Hernandez disse que chega na segunda-feira.)

Na passagem do discurso direto ao indireto ocorrem mudanças principalmente nos tempos verbais e
nas pessoas gramaticais. Sofrem alterações também os possessivos, os demonstrativos, os advérbios e
as expressões que indicam referências de tempo e de espaço.

Verbos que introduzem o Discurso Indireto (Verbos que introducen el Discurso Indirecto)

Pablo dijo que saldrá temprano. (decir)


(Pablo disse que sairá cedo.)

3
http://www.soespanhol.com.br/conteudo/discursos.php

120
1103362 E-book gerado especialmente para MARCUS ICARO CRUZ DA SILVA
El panadero nos comentó que la harina es más cara que la leche. (comentar)
(O padeiro nos comentou que a farinha é mais cara que o leite.)

El profesor sugerió a todos los alumnos que estudiasen el contenido. (sugerir)


(O professor sugeriu a todos os alunos que estudassem o conteúdo.)

El médico agregó que además de no recomendar el consumo de chocolate, el paciente tendría que
consumir más ensalada. (agregar/añadir)
(O médico acrescentou que além de não recomendar o consumo de chocolate, o paciente teria que
consumir mais salada.)

Él advertió a las chicas que no se olvidasen de hacer las tareas. (advertir)


(Ele advertiu as meninas que não se esquecessem de fazer as tarefas.)

Correspondência verbal entre os discursos direto e indireto (Correspondencia verbal entre los
discursos directo e indirecto)

O verbo que introduz a frase principal no presente:

Discurso Direto Discurso Indireto


Canto la música. Dice que canta la música.
Cantaba la música. Dice que cantaba la música.
Canté la música. Dice que cantó la música.
He cantado la música. Dice que ha cantado la música.
Cantaré la música. Dice que cantará la música.
Cantaría la música. Dice que cantaría la música.
Había cantado la música. Dice que había cantado la música.
Habré cantado la música. Dice que habrá cantado la música.
Habría cantado la música. Dice que habría cantado la música.
Quiero que cantes la música. Dice que quiere que cantes la música.
Quería que cantaras la música. Dice que quería que cantaras la música.
¡Canta la música! Dice que cantes la música.
No cantes la música. Dice que no cantes la música.

O verbo que introduz a frase principal no passado:

Discurso Direto Discurso Indireto


Canto la música. Dijo que cantaba la música
Cantaba la música. Dijo que cantaba la música.
Canté la música. Dijo había cantado/cantó la música.
He cantado la música. Dijo que había cantado la música.
Cantaré la música. Dijo que cantaría la música.
Cantaría la música. Dijo que cantaría/cantará la música.
Había cantado la música. Dijo que había cantado la música.
Habré cantado la música. Dijo que habría/habrá cantado la música.
Habría cantado la música. Dijo que habría cantado la música.
Quiero que cantes la música. Dijo que quiere que cantaras la música..
Quería que cantaras la música. Dijo que quería que cantaras la música..
¡Canta la música! Dijo que cantaras la música.
No cantes la música. Dijo que no cantaras la música.

121
1103362 E-book gerado especialmente para MARCUS ICARO CRUZ DA SILVA
¡Atención!

Ainda há, além dos discursos direto e indireto, o discurso citado. É aquele em que o
narrador cita literalmente as palavras ou trechos do discurso de outra pessoa. Elas
ressaltam o pouco ou o nenhum envolvimento do eu no discurso do outro. Temos
que destacar o importante papel dos sinais de pontuação nesse discurso: os dois-
pontos, as aspas, os colchetes, o travessão e as reticências.

Referências

http://www.guiapraticodeespanhol.com.br/2010/08/particula-se-em-espanhol.html
http://www.soespanhol.com.br/conteudo

122
1103362 E-book gerado especialmente para MARCUS ICARO CRUZ DA SILVA

Anda mungkin juga menyukai